Pediatrics

¡Supera tus tareas y exámenes ahora con Quizwiz!

genu valgum

"Knock knees" or idiopathic angulation of the knees toward the midlines; age of onset 3-5 years. Characterized by separation of the ankles when standing erect with knees together. LEgs swing laterally with walking or running. Surgical intervention is indicated only if it persists beyond 10 years of age or causes pain; otherwise, manage with observation.

DIC

A disease characterized by both accelerated fibrinogenesis and fibrinolysis. The initiating event is clotting that leads to consumption of procoagulant factors and resultant hemorrhage. Signs include cutaneous and internal organ bleeding. Labs will show thrombocytopenia, prolonged PT and aPTT, reduction in clotting factors, elevated fibrin degradation products (positive D dimer), and fragmented and helmet cells on smear.

whooping cough (Bordetella pertussis)

A disease characterized by severe coughing episodes, often associated with posttussive emesis, and inspiratory "whoop," and phyiscal exam findings of increased capillary pressure (facial petechiae, subconjunctival hemorrhage)

cystic fibrosis

A disease due to an autosomal recessive mutation of the CFTR gene. Patients present with meconium ileus, failure to thrive, and frequent respiratory infections. Sweat will be super salty. In adulthood, patients are infertile (if male), malnourished, and still have repeated pulmonary infections. Diagnose with sweat chloride test (>40 in infants, >60 in other is suggestive).

Reye syndrome

A disease in pediatric patients that presents as rapid onset of encephalopathy, hepatic dysfunction, cerebral edema, and increased ICP. It is exclusively seen in young children treated with apsirin for a viral infection (influenza, varicella).

laryngomalacia

A disease which causes chronic stridor in infants. Caused by a "floppy" supraglottic structure that collapses during inspiration. Usually begins in the neonatal period and is loudest at age 4-8 months. Presents with inspiratory stridor *worse in the supine position* and exacerbated by feeding or URI; prone positioning improves symptoms. Normally it resolves spontaneously by 18 months. Diagnosis is clinical, but can be confirmed with visualization of the larynx by direct laryngoscopy. Findings include an omega-shaped epiglottis and collapse of the supraglottic structures during inspiration.

epiglottitis

A disease which is now extremely rare thanks for the Hib vaccine. Occurs in children 6-12 years. A bacterial or viral infection of the epiglottis. No prodrome but there is a high fever and rapid onset (Within hours). Patient will tripod to open the airway, and will be drooling due to swallowing difficulties. CXR will show thumb print sign, but don't waste time on getting CXR; is suspected, go straight to OR for controlled intubation to visualize swollen epiglottis. Once airway is secured, give abx and patient will rapidly improve.

CVID (common variable immunodeficiency)

A disorder in which B-cell differentiation is abnormal, leading to decreased production of multiple IG classes. Patients are at risk for infections by encapsulated bacteria, Giardia, and enterovirus. Failure to thrive and chronic pulmonary disease are common features. Manage with IVIG to prevent severe infection.

SCID (severe combined immunodeficiency)

A disorder of defective T cell development leading to severe B cell dysfunction. Affected patients present with failure to thrive, recurrent infections, and extremely low lymphocyte concentrations. Lab findings include lymphopenia and hypogammaglobulinemia. Stem cell transplant is the only treatment.

Kallmann syndrome

A disorder of migration of fetal GnRH and olfactory neurons that results in delayed puberty (primary amenorrhea) and anosmia. Patients have hypogonadotropic hypogonadism (low FSH). Genotype will be consistent with phenotype (eg female appearing patients will be 46,XX) and internal organs will appear normal.

renal tubular acidosis

A disorder that results from the inability of the kidney to maintain normal acid-base balance because of defects in bicarbonate conservation or because of defects in the excretion of hydrogen ions. There are both congenital and acquired forms.

Trendelenburg sign

A drooping of the contralateral pelvis that occurs when a patient stands on one foot. There is an associated gait that is waddling in quality, caused by trunk's rocking to compensate for pelvic drooping during stance phase of gait. Due to weakness of gluteus medius and minimus muscles, which are innervated by the superior gluteal nerve.

Alport syndrome

A familial disorder which presents in childhood with recurrent gross hematuria and proteinuria. Sensorineural deafness usually occurs. Electron microscopy findings include alternating areas of thinned and thickened capillary loops with splitting of the GBM.

anaphylaxis

A life-threatening complication of IgE mediated allergic reactions. Can involve hypotension, diarrhea, hives, and airway edema. A diagnosis is clinical and requires involvement of at least two organ systems (and it doesn't need to involve the airway). A confirmed exposure to an allergen isn't needed; treat with *epinephrine 1:1000 IM*, support the airway with *intubation*, and treat *hypotension* with IV fluids and pressors if needed. Adjunt therapy includes H1/H2 blockers and albuterol, which work better than steroids. Make sure to provide an epinephrine pen at discharge and advise staying away from allergic triggers.

craniopharyngioma

A low-grade malignancy derived from remnants of the RAthke pouch. They can cause bitemporal hemianopsia due to optic chiasm compression and endocrinopathies (growth hormone deficiency, diabetes insipidus) due to pituitary stalk compression. Imaging will show a suprasellar calcified mass.

ETEC (enterotoxigenic E coli)

A major cause of traveler's diarrhea, which presents with watery diarrhea. Stool WBCs will be absent and diagnosis can be confirmed with culture. Quinolones or sulfonamides may shorten duration of symptoms, and hydration is essential

hemochromatosis

A major complication of beta thal major caused by increased iron absorption from the intestine and from iron in transfused RBCs. Chelation of iron with deferoxamine promotes iron excretion and may help delay this complication.

late-onset pauciarticular JIA

A male predominant subtype of JIA where age of onset is older than 8, less than 4 joints are involved, and HLA-B27 is positive. Involvement of the hips and sacroiliac joints is common.

atlantoaxial instability

A malformation seen in 10-15% of patients with Down syndrome, which most commonly occurs due to excessive laxity in the posterior transverse ligament, causing increased mobility between the atlas (C1) and axis (C2). Symptoms include torticollis, urinary incontinence, dizziness, vertigo, and diplopia, which progress over several weeks. They result from compression of the spinal cord. On exam, there are UMN findings (leg spasticity, hyperreflexia, Babinski sing, clonus).

prune belly syndrome

A malformation that occurs mostly in males, characterized by a lax, wrinkled abdominal wall, a dilated urinary tract, and intra-abdominal testicular tissue. Additional urinary tract abnormalities include significant renal dysfunction or dysplasia. Oligohydramnios and commonly associated pulmonary complications, such as pulmonary hypoplasia and pneumothorax, are seen. Congenital hip dislocation, clubfeet, and intestinal malrotation with possible secondary volvulus can occur. There does not appear to be a genetic predisposition

neuroblastoma

A malignant tumor of neural crest cells which may arise anywhere along the sympathetic ganglia chain and within the adrenal medulla. The second most common solid tumor in kids after brain tumors. Diagnosed via urine excretion of excessive catecholamines, including *VMA* and *HVA*. Definitive diagnosis by positive bone marrow biopsy plus elevated urine catecholamines, or by results of tissue biopsy. CT or MRI used to assess tumor spread. Skeletal survey or technetium 99m bone scan used to assess for metastasis to bone.

vitamin B2 (riboflavin) deficiency

A malnourished patient with angular cheilitis, stomatitis, glossitis, normocytic normochromic anemia, and seborrheic dermatitis should raise suspicion for what?

caloric irrigation

A maneuver to assess the extent of brainstem injury in a comatose patient when the oculocephalic response is negative or cannot be performed because of possible cervical cord injury. The head is angled at 30 degrees and each auditory canal is irrigated with ice water. In intact response is reflected by eye deviation to the irrigated side (COWS). An abnormal response suggests pontine injury.

*doll's eyes* (oculocephalic maneuver)

A maneuver to assess the extent of brainstem injury in a comatose patient. In normal patients, turning the head of an unconscious patient causes the eyes to look straight ahead and then slowly drift back to the midline position becaus ethe intact vestibular apparatus senses a change in position. In an injured brainstem, movement of the head does not evoke any eye movement; this is termed a negative coulocephalic maneuver.

Cobb angle

A measurement used to define the degree of scoliosis. Measured by drawing a line along the superior aspect of the most angulated vertebrae at the top of the curvature and drawing another line along the inferior aspect of the lowest most angulated vertebrae of the curvature. The angle of the intersection of these lines is what is used.

vaginal diaphragm

A mechanical barrier placed against the cervix and used in combination with spermicide as contraception. It may be inserted as many as 6 hours before intercourse. However, it must be individually fitted by a trained healthcare professional and increases UTI risk.

intrauterine device (IUD)

A method of birth control which either interferes with sperm transport (copper type) or induces endometrial atrophy (progesterone type). Convenient and private, but does not protect against STDs. Can cause uterine bleeding and cramping, and is higher cost.

Alpha Thalassemia

A microcytic anemia that occurs predominantly in Southeast Asians. Due to a genetically mediated deletion of the alpha globin chain. Since there are normally four alpha-globin genes per cell, one gene deletion is silent, two gene deletion is minor with mild anemia, three gene deletion causes severe anemia (Hbb Bart's) and four gene deletion results in fetal hydrops.

iron deficiency

A microcytic anemia with *low serum ferritin* as well as *increased transferrin* and *decreased transferrin saturation.* Reticulocyte count will be normal or increased.

ophthalmoplegic migraine

A migraine headache associated with unilateral ptosis or cranial nerve III palsy

Gilbert syndrome

A mild form of UDP-glucuronyl transferase deficiency in which 50% of the enzyme activity is absent. Mild unconjugated bilirubinemia occurs, associated with stress and poor nutrition.

Enterobius vermicularis (pinworm)

A *helminth infection* in school-age children which typically presents with nocturnal perianal pruritus. Transmission occurs via contact with contaminated bojects (eg bedding or clothing) or unwashed hands after scratching the perianal area. Although adult worms live in the intestine, females migrate distally to deposit eggs onto the perianal skin, primarily at night. In females, the helminths can also spread to the vagina and cause vulvovaginitis.

*Erb palsy:* affects the fifth and sixth cervical nerves; the affected arm cannot be abducted or externally rotated at the shoulder, and the forearm cannot be supinated.

A 1-day-old infant who was born by a difficult forceps delivery is alert and active. She does not move her left arm spontaneously or during a Moro reflex. Rather, she prefers to maintain it internally rotated by her side with the forearm extended and pronated. The rest of her physical examination is normal. What injury has likely occurred?

In *DIC,* there is consumption of fibrinogen; factors II, V, and VIII; and platelets. Therefore, there is prolongation of PT, aPTT, and TT and a decrease in factor VIII level and platelet count. In addition, the titer of fibrin split production is usually increased. D-dimer is a fibrin breakdown product and may also be elevated in DIC

A 10-year-old boy is admitted to the hospital because of bleeding. Pertinent laboratory findings include a platelet count of 50,000/μL, prothrombin time (PT) of 15 seconds (control 11.5 seconds), activated partial thromboplastin time (aPTT) of 51 seconds (control 36 seconds), thrombin time (TT) of 13.7 seconds (control 10.5 seconds), and factor VIII level of 14% (normal 38%-178%). What is the most likely diagnosis?

In an infant who appears otherwise normal, the sudden onset of high fever, together with a marked elevation and shift to the left of the WBC count, suggests *pneumococcal bacteremia.* The incidence of pneumococcal disease producing this picture may be decreasing with the widespread use of a pneumococcal vaccine.

A 14-month-old infant suddenly develops a fever of 40.2°C (104.4°F). Physical examination shows an alert, active infant who drinks milk eagerly. No physical abnormalities are noted. The WBC count is 22,000/μL with 78% polymorphonuclear leukocytes, 18% of which are band forms. What is the diagnosis?

*Fetomaternal transfusion:* the absence of a major blood group incompatibility and the finding of a normal reticulocyte count argue strongly in favor of a recent fetomaternal transfusion, probably at the time of delivery. A Kleihauer-Betke stain for fetal hemoglobin-containing RBCs in the mother's blood would confirm the diagnosis.

A 2950-g (6.5-lb) black baby boy is born at home at term. On arrival at the hospital, he appears pale, but the physical examination is otherwise normal. Laboratory studies reveal the following: mother's blood type A, Rh-positive; baby's blood type O, Rh-positive; hematocrit 38%; and reticulocyte count 5%. What is the likely diagnosis?

*VATER* (or VACTERL) is an association of commonly seen findings of unknown etiology. The patents affected have vertebral defect, anal atresia, cardiac defects, tracheoesophageal fistula, renal/radial defect (or both), and limb abnormalities. Intelligence is normal.

A 3-day-old infant who was found at birth to have anal atresia also has vertebral defects, a VSD, tracheoesophageal fistula, absent left kidney, and shortened arms. What syndrome does this infant likely have?

*Transposition of the great vessels* with an intact ventricular septum presents with early cyanosis, a normal-sized heart (classic "egg on a string" radiographic pattern in one-third of cases), normal or slightly increased pulmonary vascular markings, and an ECG showing right-axis deviation and right ventricular hypertrophy

A 3-day-old infant with a single second heart sound has had progressively deepening cyanosis since birth but no respiratory distress. Chest radiography demonstrates no cardiomegaly and normal pulmonary vasculature. An ECG shows an axis of 120° and right ventricular prominence. What is the likely diagnosis?

*Transient synovitis:* a disorder of unknown etiology, affecting children usually from 2 to 6 years of age. These children usually present with a painful limp. This is a diagnosis of exclusion; septic hip and osteomyelitis must be ruled out. The WBC count and ESR may be normal or slightly elevated. Early aspiration of the joint space may assist in diagnosis. Transient synovitis is a self-limited disorder.

A 3-year-old child refuses to walk, is afebrile, had an upper respiratory tract infection a week ago, has right hip pain with movement, and has a normal WBC count.

*HUS* is most common in children younger than 4 years of age and is characterized by an acute microangiopathic hemolytic anemia, thrombocytopenia from increased platelet utilization, and renal insufficiency from vascular endothelial injury and local fibrin deposition. Ischemic changes result in renal cortical necrosis and damage to other organs such as colon, liver, heart, brain, and adrenal. Laboratory findings associated with hemolytic uremic syndrome include low hemoglobin level, decreased platelet count, hypoalbuminemia, and evidence of hemolysis on peripheral smear (burr cells, helmet cells, schistocytes). Urinalysis reveals hematuria and proteinuria. A marked reduction of renal function leads to oliguria and rising levels of blood urea nitrogen (BUN) and creatinine. Gastrointestinal bleeding and obstruction, ascites, and central nervous system findings such as somnolence, convulsions, and coma can occur. Infection by the verotoxin-producing E coli 0157:H7 has been implicated as a cause of HUS. This organism is epizootic in cattle. Outbreaks associated with undercooked contaminated hamburgers have been reported in several states. Roast beef, cow's milk, and fresh apple cider have been implicated as well. The Coombs test is not positive in this type of hemolytic anemia. *Antibiotic therapy in a patient actively infected with Shiga-toxin-producing E coli may worsen the risk of HUS. Rather, meticulous detail to fluid and electrolyte management is warranted and has resulted in dramatic decreases in the previously reported mortality associated with this disease. *

A 4-year-old boy and his family have recently visited a local amusement park. Several of the family members developed "gastroenteritis" with fever and diarrhea, but the 4-year-old boy's stool was slightly different, as it contained blood. His mother reports that in the past 24 hours he developed pallor and lethargy; she relates that his face looks swollen and that he has been urinating very little. Laboratory evaluation reveals a hematocrit of 28% and a platelet count of 72,000/μL. He has blood and protein in the urine. What is the diagnosis and management?

imperforate anus

A cause of failure to pass meconium and the reason newborn temperatures are not taken anally. Assess with a cross table X-ray on the prone child with with radiopaque perineal marking, which gives relationship between gas bubble and anus. All patients with concerns for this should undergo sacral US and X-ray, VCUG, NG tube passage, echocardiogram. Low lesions can be corrected via dilation or minor surgery and there is a higher chance of maintaining continence. High lesions need a colostomy with future correction.

nonclassic congenital adrenal hyperplasia

A cause of gonadotropin-independent precocious puberty. Patients can have premature secondary sex characteristics (pubic hair, cystic acne), accelerated growth and bone age, and low to normal LH levels.

hepatitis B

A cause of hepatitis associated with a DNA virus. Can be transmitted perinatally or through exposure to infected blood, tattooing needles, IV drug use, or exposure to infected body secretions.

renal papillary necrosis

A cause of isolated hematuria in patients with sickle cell trait

Diamond Blackfan anemia

A cause of macrocytic anemia which will present with craniofacial anomalies and triphalangeal thumbs. Caused by a congenital defect of erythroid progenitor cells, which leads to increased apoptosis of RBCs, resulting in profound anemia. Short stature, cleft palate, and webbed neck are all common findings.

McCune Albright syndrome

A cause of peripheral precocious puberty characterized by bony changes (polyostotic fibrous dysplasia), skin findings (cafe au lait spots), and endocrinopathies (hyperthyroidism, precocious puberty). PAtients often have enlarged gonas but their secretion of sex steroids is independent of the HPGA.

bacterial tracheitis

A cause of stridor characterized by acute inflammation of the trachea. Causes include Staph aureus, Strep, and nontypeable H flu. Disease is abrupt onset with toxicity, high fever, and mucous and pus in the trachea. Give antistaphylococcal antibiotics and airway support to treat.

inspissated bile syndrome

A cause of unconjugated hyperbilirubinemia associated with hemolysis (eg ABO imcompatability) or large hematomas. In these cases, the biliary system becomes overwhelmed by the increased bilirubin load; although unconjugated hyperbilirubinemia predominates early, conjugated hyperbilirubinemia eventually develops as hepatocellular function increases to meet demand.

Sandifer syndrome

A characteristic torticollis with arching of the back in infants, caused by painful esophagitis (GERD)

hydroxyurea

A chemotherapeutic agent that increases hemoglobin F, used as preventive care in sickle cell patients; has been shown to decrease incidence of vasoocclusive crises.

*Juvenile idiopathic arthritis* (JIA, formerly known as juvenile rheumatoid arthritis) frequently causes spindle-shaped swelling of finger joints and can involve unusual joints such as the sternoclavicular joint. Presentation of JIA occurs as either polyarthritis (five or more joints, systemic symptoms not so severe or persistent), pauciarticular (four or fewer joints, lower-extremity joints, extra-articular disease unusual), or systemic disease (severe constitutional disease, systemic symptoms prior to arthritis, rheumatoid rash, high spiking fevers, variable joint involvement, also known as Still's disease). This disorder can be associated with spiking high fevers and diffuse rash.

A child has a 2-week history of spiking fevers, which have been as high as 40°C (104°F). She has spindle-shaped swelling of finger joints and complains of upper sternal pain. When she has fever, the parents note a faint salmon-colored rash that resolves with the resolution of the fever. She has had no conjunctivitis or mucositis, but her heart sounds are muffled and she has increased pulsus paradoxus. What is the diagnosis?

Lymphadenitis, most commonly caused by bacterial infection; Staph aureus is the most common pathogen isolated, followed by group A strep.

A child has a lymph node that becomes tender, erythematous, and enlarged. It is acute and unilateral. The patient is under 5 and nontoxic appearing. What is the diagnosis?

tuberous sclerosis

A child has achromic skin patches and infantile spasms. What is the likely underlying disorder?

*ALL:* additional symptoms may include anorexia, epistaxis, fatigue, testicular pain and swelling, and abdominal pain. Note that a normal CBC does not rule out leukemia.

A child presents with fever and bone and joint pain. They refuse to bear weight. On physical exam, pallor, bruising, hepatosplenomegaly, and lymphadenopathy are noted. CBC shows anemia and thrombocytopenia. What do you suspect?

Sigella Salmonella E coli Yersinia

A child with bloody diarrhea plus fever and history of travel should raise alarms for what pathogens?

*Spondylolisthesis:* a developmental disorder characterized by a forward slip of vertebrae, usually L5 over S1.

A child with slowly developing back pain and palpable "step off" at the lumbosacral area should raise suspicion for what diagnosis?

SCID

A child without any immune system has this disease. They are at risk for every infection. Thus, they become infected with opportunistic organisms. Give prophylactic TMP/SMX and scheduled IVIG. Bone marrow transplant is used for treatment.

duodenal atresia

A child's prenatal ultrasound demonstrates gastric dilation with polyhydramnios. At birth, they have a scaphoid abdomen with epigastric distention. They have a feeding intolerance and vomiting with FTT. What disease is likely?

juvenile idiopathic arthritis

A chronic autoimmune disorder marked by symmetric arthritis of 1 or more joints. There will be no fever or rash. Lab findings reflect systemic inflammation, with elevated CRP/ESR, hyperferritinemia, hypergammaglobulinemia, and thrombocytosis. *Anemia* is commonly seen as a consequence of this chronic inflammation.

atopic dermatitis

A chronic inflammatory dermatitis (eczema) characterized by dry skin and thickening of the skin (lichenificiation). The skin is overly sensitive to many sitmuli that produce pruritus, which leads to scratching, which causes many of the skin manifestations. In infants, truncal and facial areas are involved; extensor surfaces are more involved than flexural surfaces. In older children, flexural surfaces are more involved.

congestive heart failure

A clinical syndrome defined as inadequate oxygen delivery by the myocardium to meet the metabolic demands of the body. In children, it is often the result of congenital heart disease, acquired heart disease, or other disorders. Clinical features include tachypnea, cough, wheezing and rales n exam and pulmonary edema on CXR (evidence of pulmonary congestion. There can also be tachycardia, sweating, pallor, diminished urine output, and enlarged cardiac silhouette. Hepatomegaly and peripheral edema can occur, as can failure to thrive, poor feeding, and exercise intolerance. Cyanosis and shock are the late manifestations.

Dandy-Walker malformation

A combination of an absent or hypoplastic cerebellar vermis and cystic enlargement of the fourth ventricle, which blocks the flow of CSF

Von Willebrand Disease

A common bleeding disorder that causes impaired platelet adhesion; it is the most common cause of heavy, regular menses in adolescents. Platelet counts and PT are normal; aPTT may be either normal or prolonged.

viral myocarditis

A common cause of CHF in older children and adolescents

Chlamydia trachomatis

A common cause of afebrile pneumonia at 1-3 months of age. Symptoms include a *staccato-type* cough, dyspnea, and absence of fever. A history of conjunctivitis after birth may be identified in 50% of patients. Physical exam may demonstrate tachypnea and wheezing. Diagnosis is suggested by eosinophilia and CXR with interstitial infiltrates. Manage with oral erythromycin or azithromycin.

Salmonella

A common cause of diarrhea; spread by poultry, milk, eggs, and exposure to lizards or turtles. Sickle cell patients with this infection may develop bacteremia or osteomyelitis.

hereditary spherocytosis

A common cause of hemolytic anemia in persons of Northern European descent. Typically an autosomal dominant disorder caused by a defect of RBC membrane proteins, most commonly ankyrin, resulting in unstable, round RBCs. These fragile cells get caught in the microcirculation of the spleen; the classic triad of symptoms is *hemolytic anemia, jaundice, and splenomegaly.* Neonates may present with jaundice refractory to standard phototherapy. Labs will show increased MHCH, indirect hyperbilirubinemia, and negative Coombs.

transient synovitis

A common cause of hip pain in children, especially boys age 3-8 years. Cause is unknown but symptoms may develop after mild viral illness. Synovial inflammation leads to pain, decreased ROM, and a limp. On exam, patients are well appearing and afebrile; the affected hip may be flexed, slightly abducted, and externally rotated. Lab shows normal WBC count and CRP/ESR due to lack of infection. Treatment consists of rest and NSAIDs with careful observation.

Meningococcal meningitis (N. meningitidis)

A common cause of meningitis in children over 3 months; it classically presents with meningeal signs as well as a petechial or purpuric rash. It progresses rapidly and causes significant morbidity and mortality, even with appropriate treatment. Treat with ceftriaxone and vancomycin.

intraventricular hemorrhage

A common complication of prematurity which can present with acute neurological changes, a bulging fontanelle, and increasing head circumference. Diagnosis is made with cranial ultrasound.

osteonecrosis (avascular necrosis)

A common complication of sickle cell disease; patients typically report chronic groin, thigh, or buttock pain without warmth or erythema of the joints, leukocytosis, or elevated inflammatory markers.

language disorder

A common developmental disability of childhood; characterized by persistent difficulties in comprehension and production of spoken and written language. It may involve rules, content, or functional use of language. IT interferes with academic functioning and early intervention in the form of therapy is beneficial.

neuroblastoma

A common extracranial solid tumor of childhood, with a median age <2. It arises from neural crest cells and often involves the cervical paravertebral sympathetic chain, leading to Horner syndrome. The "harlequin" sign can be observed in anhidrotic areas.

Lyme arthritis

A common late manifestation of Borrelia burgdorferi infection. Presents as an inflammatory monoarticular or asymmetric oligoarticular arthritis, most commonly in the knee. Synovial fluid shows an inflammatory profile, but gram stain and culture are usually negative.

Ewing sarcoma

A common malignant bone tumor of childhood which typically affects white adolescent boys. The tumor usually occurs in the pelvis or diaphyses of long bones like the femur, and often metastasizes early to the lungs, bone, and bone marrow. IT presents with localized pain and swelling; systemic findings (fever, weight loss, leukocytosis, elevated CRP/ESR) are common. Imaging shows a lamellated periosteal reaction with "onion-skinning" and a "moth-eaten" or mottled appearance with extension into soft tissue.

"growing pains"

A common musculoskeletal complaint in children, with unknown etiology. Diagnosis can be made clinically: features include pain at night that resolves by morning, pain affecting the lower extremities bilaterally, and otherwise normal physical exam and activity. Management includes parental education and reassurance; massage, stretching exercises, and heat can be used.

hearing loss

A common neurologic sequelae of Strep pneumo meningitis due to inflammatory damage of the vestibulocochlear nerve, cochlea, or labyrinth

*Impetigo:* topical antibiotics like *mupirocin* are preferred for localized infection; oral antibiotics (cephalexin, dicloxacillin, clindamycin) are indicated when topical therapy is impractical for widespread impetigo. Bullous impetigo (flaccid bullae containing yellow fluid) would also call for oral antibiotics.

A common pediatric rash typically caused by Staph aureus or Strep pyogenes. Manifests with multiple painful pustules on the face and extremities; the pustules rupture and harden into a characteristic "honey colored" crust.

macrosomia

A common result of fetal hyperglycemia and hyperinsulinemia; can increase the risk of shoulder dystocia, which can be complicated by brachial nerve palsies and clavicle fractures.

transient synovitis

A common self-limited postinfectious response of the hip joint. URI or diarrhea often precede it. It is the most common cause of painful limp in toddlers. Characterized by low grade fever, limp, and irritability. Hip pain may be acute or insidious. WBC and ESR will be normal or only slightly elevated, Treat with NSAIDs, bed rest, observation.

*Tinea versicolor:* a fungal infection of the skin caused by Malassezia yeasts. Diagnosis is usually made clinically, although potassium hydroxide prep demonstrates yeast cells and hyphae ("spaghetti and meatballs"). Topical therapy (selenium sulfide, antifungals) generally result in resolution.

A common skin eruption characterized by light macules on the trunk and upper extremities in adults; in children, the eruption typically involves the face as well. It is most noticieable after sun exposure as the surrounding skin becomes darker.

herpetic gingivostomatitis

A common vesicular oral infection in children caused by HSV; presents with fever, pharyngitis, and clusters of vesicles that generally localize to the anterior oral cavity and lips.

herpangina

A common vesicular oral infection in children caused by coxsackievirus; it presents with fever and pharyngitis as well as gray vesicles that progress to fibrin-coated ulcers on the posterior soft palate, anterior palatine pillars, tonsils, and uvula.

Osgood Schlatter

A common, benign cause of knee pain caused by overuse in young adolescents. During periods of rapid growth, the quadriceps muscles pull on the insertion site of the patellar tendon. Pain occurs at this site, which is located on the apophysis of the tibial tubercle. This traction apophysitis may lead to elevation and chronic avulsion of the tibial tubercle. Patients have progressive pain over the tibial tuberosity that worsens with activites and improves with rest. Exam typically reveals prominence and tenderness over the tibial tubercle. Pain can be reproduced by squatting or extending the knee against resistance.

Mycobacterium avium complex (MAC)

A complication of HIV infection characterized by fever, weight loss, night sweats, abdominal pain, bone marrow suppression, and elevated LFTs; risk is highest when CD4 count falls under 50.

hydrops of the gallbladder

A complication of Kawasaki disease which may occur in 10% of patients and should be considered in patients with acute RUQ abdominal pain.

coronary artery aneurysms

A complication of Kawasaki disease which occurs in 20% of untreated patients, usually around days 7-14 (most commonly in the subacute phase of the disease).

eczema herpeticum

A complication of atopic dermatitis caused by a superimposed HSV infection. An open area exposed to HSV type 1 can develop painful vesicles with an erythematous base that evolve to punched-out erosions with hemorrhagic crusting. Fever and lymphadenopathy are typically present. Begin systemic acyclovir as soon as possible, as this type of infection may be life-threatening.

avascular necrosis (osteonecrosis)

A complication of sickle cell disease common at the humeral and femoral heads. Features include pain that initially occurs with weight bearing bur progresses to pain at rest. Exam shows pain and limited ROM. There will be no warmth or erythema, leukocytosis, or elevated CRP/ESR. X-ray may appear normal early in disease, and MRI should be used to confirm diagnosis.

compartment syndrome

A complication of supracondylar fractures wherein the pressure within the anterior fascial compartment is greater than 30-45 mm Hg, leading to ischemic injury and Volkmann's contracture (flexion deformity of the fingers and wrist). Pallor, pulselessness, paralysis, pain, and paresthesias are late signs. *Pain with passive extension of the fingers* is a big clue.

compartment syndrome

A complication of supracondylar humerus fractures characterized by severe pain, pallor, poikilothermia, paresthesias, and late findings of pulselessness and paralysis. Patients with the predisposing injury should be monitored for development of this complication, particularly as swelling from the injury increases.

toxic megacolon

A complication of ulcerative colitis. Severe inflammation of the colon leads to decreased intestinal motility, disruption of the mucosal barrier which allows bacteria to enter, and colonic dilatation. Patients present with fever, abdominal distention, and septic shock. They are at risk for perforation and hemorrhage.

hereditary fructose intolerance

A condition cause dby fructose 1 phosphate aldolase B deficiency. Begins in infancy, after introduction of fruit juice to the infant's diet. Symptoms include severe hypoglycemia, vomiting, diarrhea, FTT, and seizures. Management includes avoidance of fructose, sucrose, and sorbitol.

Hirschprung's disease

A condition caused by absent ganglion migration to the Meissner and Auerbach plexus in the colon, resulting in no motility, since muscles are unable to relax. X-ray shows a dilated proximal colon and a normal-looking distal colon. Diagnose with contrast enema, which shows a transition zone. Follow up with a biopsy. In some cases, rather than presenting with failure to pass meconium, there will be overflow incontinence with stool eruption after a digital rectal exam. In this case, anorectal manometry is done, which shows increased tone. Also follow this with biopsy.

transient tachypnea of the newborn

A condition caused by delayed resorption and clearance of alveolar fluid in premature newborns; c-section babies are at increased risk. Normally, mature fetal lungs reabsorb liquid late in gestation. Excess pulmonary fluid causes resp distress (tachypnea, grunting, retractions) and hypoxia in newborns within hours of delivery. However, breath sounds are often clear, since fluid is in interstitial spaces and not alveoli themselves. On CXR, lungs will be hyperinflated with fluid in interlobar fissures; this confirms the diagnosis. IT is the self-limited condition that resolves within hours to days; treat with supplemental O2.

premature adrenarche

A condition caused by early activation of adrenal androgens. Typical manifestations include body odor, oily skin, acne, pubic hair, and axillary hair. However, estrogen and testosterone levels remain normal, so there are no other signs of premature puberty or virilization. Bone age is also normal in these patients.Generally benign, but a risk factor for PCOS, DM, and metabolic syndrome, especially in obese patients.

rickets

A condition caused by vitamin D deficiency that results in deficient mineralization of growing bones with a normal bone matrix. More common in exclusively breastfed infants with minimal sunshine exposure. Due to vitamin D deficiency most often. Presents with bowed weight bearing bones, short stature, prominent costochondral junctions, and craniotabes (thinning of the outer skull to create a ping pong ball sensation on palpation).

HUS

A condition characterized by acute renal failure in the presence of microangiopathic hemolytic anemia and thrombocytopenia

meconium ileus

A condition generally seen in cystic fibrosis patients. A collection of meconium too thick and viscous to pass through the bowels creates a blockage; the result of pancreatic insufficiency. Typically located in the ileum. Can cause bilious vomiting or failure to pass meconium. X-ray can show an area of obstruction (with air-fluid levels) with a gas-filled plug. Perform a water-soluble contrast enema to help break down the obstruction. Sometimes surgical intervention is required. Perforation is a complication.

small left colon syndrome

A condition occurring exclusively in infants of diabetic mothers, in which infants present with abdominal distention and failure to pass meconium secondary to the decreased caliber of the left colon.

angioedema

A condition similar to urticaria, but swelling involves the deeper layers of the dermis and mucous membranes (lips, airway ,GI tract). Can be seen along with urticaria, anaphylaxis, or independent of either (ACE-inhibitor induced). Secure the airway with intubation to treat; antihistamines and steroids will likely not be effective.

orthostatic proteinuria

A condition where children, especially athletic individuals, have increased urianry protein excretion while upright but not while supine. A benign condition whose confirmation eliminates the need for further workup. Diagnosed with elevated afternoon TP:CR ratio and a normal first-morning urine TP:CR ratio.

neural tube defects (spina bifida)

A congenital defect caused by genetic syndromes and maternal folate deficiency. Stems from difficulties with fusion of the caudal neural tube. Treated with folate supplementation. Screen during pregnancy; a positive AFP screen prompts US to reveal the defect.

biliary cyst

A dilatation of the biliary tree. Presents with abdominal pain, jaundice (due to obstructive cholestasis), and palpable mass. Older children may have pancreatitis. Diagnosis is generally made by US; ERCP may be needed if obstruction is suspected. Surgical resection is the treatment as it relieves obstruction and reduces chance of malignant transformation.

Pompe disease (GSD type II)

A disease caused by alpha glucosidase deficiency. Should be suspected in any infant with muscular weakness and cardiomegaly. Presents within first 2 weeks of life with flaccid weakness, poor feeding, progressive cardiomegaly, hepatomegaly, and acidosis.

The child in the question likely has *renal vein thrombosis*, a condition more commonly seen in infants who are dehydrated, had birth depression, have polycythemia or were born to diabetic mothers. The child in the case was large for gestational age, a hint that gestational diabetes might have been present and which also can result in polycythemia. Children with renal vein thrombosis can present with history of oliguria and hematuria, and on examination enlarged kidneys may be palpable. Confirmation of the diagnosis is by* renal ultrasound with renal vein Doppler studies. *

A mother arrives to your office in a panic. She had just returned home from the delivery of her third child, who was born by cesarean section because he was large for gestation age. Upon changing his first diaper at home she noticed gross blood in his diaper. On examination, you find a left-sided abdominal mass that was not present in the normal newborn nursery. What is the diagnosis and management?

*Pneumothorax* occurs with a frequency of about 1% to 2% of births, but they are rarely symptomatic. Incidence is higher in infants born with meconium-stained fluid, and the chest radiograph is as described. Transillumination may assist in the diagnosis while awaiting radiograph; immediate treatment for infants with significant distress is with a 23-gauge butterfly needle attached to a stopcock and removal of the air. For those without significant distress and who are not requiring high levels of oxygen, 100% oxygen therapy can assist in "nitrogen washout" of the pneumothorax.

A postterm infant is born at home after a prolonged and difficult labor. The maternal grandmother brings the infant to the hospital at 1 hour of life because of fast breathing. The grandmother notes that the child seemed well for a while, but then developed increased work of breathing. Physical examination reveals an infant in moderate respiratory distress with diminished breath sounds on the left. Chest radiograph reveals the heart to be pushed to the right side and loss of lung markings in the left lung field. What is the diagnosis?

*Holt-Oram syndrome* is characterized by abnormalities in the upper extremities, hypoplastic radii, thumb abnormalities, and cardiac anomalies. Occasionally the pectoralis major muscle is missing in Holt-Oram, and as such it needs to be considered when discussing Poland syndrome.

A newborn has low sloping shoulders, right hand attached at elbow with agenesis of the forearm, cardiac abnormalities, missing chest wall musculature, and a bifid thumb. What is the diagnosis?

*CMV* is the most common sort of congenital infection, with infection estimates ranging from 0.4% to 2.4% of all live births. Many cases are asymptomatic; others may develop cytomegalic inclusion disease, a multiorgan manifestation of disease including intrauterine growth restriction (IUGR), hepatosplenomegaly, jaundice, petechiae or purpura, microcephaly, chorioretinitis, and intracranial calcifications. More than half of infants with this congenital infection develop sensorineural hearing loss.

A newborn has microcephaly, intracranial calcifications, hepatospleno-megaly, and marked hyperbilirubinemia and thrombocytopenia. What is the likely diagnosis?

neonatal sepsis

A newborn infant becomes markedly jaundiced on the second day of life, and a faint petechial eruption, first noted at birth, is now a generalized purpuric rash. Hematologic studies for hemolytic diseases are negative. What is likely occurring?

mastoiditis

A potential complication of acute otitis media where the mastoid air cells become acutely infected; risk is increased with tympanoplasty. If there's acute otitis media and swelling behind the ear or anteriorly rotated ear, prompt surgical eval is needed.

refeeding syndrome

A potentially fatal complication of nutritional rehab in patients with chronic anorexia nervosa. Carb intake stimulates insulin activity, which in turn promotes cellular uptake of phosphorus, potassium, and magnesium. Clinical manifestations include arrhythmias and cardiopulmonary failure.

*Androgen insensitivity syndrome:* nonfunctioning androgen receptors result in peripheral androgen resistance. They is typically primary amenorrhea due to lack of female internal genitalia. Testes are fucntionally normal but are undescended and may be found in the abdomen, labia, or inguinal canal. Patients with cryptorchid testes are at increased risk of testicular cancer and management includes *elective gonadectomy.*

A patient who is genotypically male (46X,Y) but appears phenotypically female should make you think of what disorder?

Lyme arthritis

A patient with acute monoarticular arthritis following months of migratory arthralgias, with synovial fluid showing leukocyte count of 25,000 but negative Gram stain and culture, may have what?

*Wiskott-Aldrich syndrome:* an X-linked recessive defect in the WAS gene, which regulates cytoskeleton remodeling in response to cell signaling. In Wiskott Aldrich syndrome, the actin cytoskeleton in WBCs is abnormal, impaired cell migration and immune response. Significant reduction in platelet volume and size is expected.

A patient with eczema, microthrombocytopenia, and recurrent infections should raise concern for what?

*von Willebrand disease:* a common bleeding disorder caused by deficiency in vWF. PAtients with vWF deficiency have impaired platelet adhesion, as vWF acts as a bridging glycoprotein between platelets and endothelial factors at sites of vascular injury. vWF also serves as a carrier for factor VIII, preventing its degradation by protein C.

A patient with heavy, regular menses, anemia, and normal coagulation studies aside from a mildly elevated PTT likely has what hematologic abnormality?

This patient likely has a temporal *brain abscess.* You should due a contrast-enhanced CT or MRI of the brain to find a ring-enhancing lesion.

A patient with recent otitis media has nocturnal headaches and morning vomiting. How should you investigate?

*X-linked (Bruton) agammaglobulinemia:* a recessive disorder characterized by a defect in tyrosine kinase in B cells, resulting in failed development of bone marrow pre-B cells into mature circulating B cells.

A patient with recurrent sinopulmonary and GI infections, markedly low serum Igs, and few B lymphocytes likely have what disease?

*SCID:* stem cell transplant is the only definitive therapy.

A patient with recurrent, severe infections, failure to thrive, and lymphopenia (low CD19 and CD3) likely has what?

*Wilson disease:* results in copper accumulation in the liver, brian, and cornea. Check ceruloplasmin levels.

A pediatric patient with psychiatric symptoms (mood changes, irritability, bizarre behavior, auditory hallucinations) and neurologic symptoms (slurred speech, drooling, hand tremor) as well as increased LFTs likely has what?

spinal muscular strophy

A peripheral hypotonic disorder caused by anterior horn cell degeneration which presents with hypotonia, weakness, and tongue fasciculations. Weak cry, bell shaped chest, and frog leg posture are all characteristic. Extraocular movements and sensory exam will be normal. It is the second most common hereditary neuromusculaar disorder after DMD. It is autosomal recessive and caused by mutations to SMN1 on chromosome 5. DNA testing is usually diagnositc, and muscle biopsy will show characteristic atrophy of groups of muscle fibers that were innervated by the damaged axons. Treatment is supportive; there is no cure. Survival beyond the first year of life is unusual.

PANDAS

A phenomenon in which patients develop acute onset of OCD symptoms or tic disorders after streptococcal infection

female condom

A polyurethane sheath placed into the vagina to prevent passage of sperm. Protects against STDs, but can cause vaginal irritation or allergy.

splenic sequestration crisis

A pooling of RBCs in the spleen, often seen in sickle cell patients. Presents with signs of severe, acute anemia (eg tachycardia, pallor, and shock) and splenomegaly in young patients. LAb testing will show normocytic anemia, reticulocytosis, and thrombocytopenia.

necrotizing fasciitis

A potentially fatal form of deep cellulitis; patients present with pain and systemic symptoms out of proportion to physical findings. Infection extends into the muscle. Exam may reveal crepitus and hemorrhagic bullae. Cause is polymicrobial and may involve GABHS and anaerobic bacteria. IV antibiotics and surgical debridement are necessary.

viral myocarditis

A potentially lethal disorder of the myocardium most commonly caused by viral infection (eg coxsackie B, adenovirus) in children. A viral prodrome (URI) often precedes illness, then patients develop chest pain and resp distress with an S3 gallop and holosystolic murmur. CXR reveals cardiomegaly.

methemoglobinemia

A rare complication that can occur after exposure to oxidizing agents like dapsone, nitrites, or anesthetics. It results in cyanosis. Lethargy, resp depression, seizures, and death may occur. Iron on hemoglobin is oxidized to the ferric (Fe3+) state and unable to bind oxygen, and the remaining ferrous (Fe2+) sites on hemoglobin have increased affinity for O2, leading to decreased oxygen delivery to peripheral tissues. Patients will have pulse ox readings of ~85% regardless of the true oxygen saturation level. Supplemental O2 does not improve symptoms. Arterial blood gas testing analyzes only unbound arterial oxygen and will display a falsely normal PaO2.

Ataxia-Telangiectasia

A rare disease which presents with telangiectasias, ataxia, poor DNA repair, lymphoma, and leukemia. Associated with sinopulmonary infections and absent IgA. Avoid excessive radiation.

varicella (chickenpox)

A rash caused by VZV. Approximately 2 weeks after exposure to airborne particles, patients with this infection have a brief prodrome of fever and malaise followed by maculopapular rash that rapidly becomes intensely pruritic and vesicular. It involves the trunk, face, and extremities. Generally self-limiting and does not require antiviral therapy.

Meckel's diverticulum

A remnant of the vitelline duct which can contain gastric tissue which causes acid secretion and therefore GI bleeding due to ulcer formation. In a child, it presents with either FOBT+, IDA, or hematochezia. Bleeding is usually painless and intermittent. Diagnosed with a technicium-99 radionucleotide scan and treated surgically

croup

A respiratory illness most common caused by parainfluenza virus; presents in children age 3-36 months. Presents with dry cough, hoarseness, and inspiratory stridor due to upper airway obstruction. Radiographs will show "steeple sign."

child abuse

A retinal hemorrhage in a child is highly suggestive of what?

hydrocele

A scrotal swelling caused by fluid accumulation in the tunica vaginalis adjacent to the testis. Although isolated incidences usually cause no clinical problems and often resolve spontaneously within a few weeks, some are associated with inguinal hernias.

benign rolandic epilepsy

A seizure disorder which involves nocturnal partial seizures with secondary generalization. Autosomal dominant. Seizures occur in the early morning hours when patients are asleep with oral-buccal manifestations (moaning, grunting, pooling of saliva). Seizures spread to face and arm and then generalize into tonic-clonic seizures. EEG shows biphasic spike and sharp wave disturbance in the mid-termporal and central regions. Valproic acid is first-line.

Sydenham chorea

A self-limited autoimmune disorder associated with *rheumatic fever* that presents with chorea (uncontrolled, restless proximal limb movenets) and emotional lability. Patients are unable to sustain protrusion of hte tongue. Gait and cognition are not affected. Diagnosis is presumed with elevated *ASO* or ADB titers which indicate a recent strep infection. Treat with haloperidol, valproic acid, or phenobarbital.

transient hyperammonemia of the newborn

A self-limited disorder that occurs in premature infants within the first 24-48 hours of life. Symptoms include resp distress, alkalosis, vomiting, and lethergy which rapidly progresses to coma. Aggressive treatment of hyperammonemia is required to prevent neurologic sequelae.

pityriasis rosea

A self-limited exanthem that begins with a solitary patch followed by clusters of smaller oval lesions that stretch along skin lines. Management is reassurance alone, though symptomatic relief of pruritus (eg antihistamines, topical corticosteroids) may be indicated.

Todd paralysis

A self-limited, focal weakness or paralysis that occurs after a focal or generalized seizure. It presents in the postictal period with hemiparesis or complete hemiplegia involving extremities on one side of the body. The pathophysiology is unknown. Rule out other causes by CT and MRI if there is no clear history of seizure. Treatment is supportive, and paralysis typically resolves within 36 hours.

*croup* (laryngotracheobronchitis)

A self-limiting illness seen commonly in children 6 months to 3 years. Caused by a parainfluenza virus that causes inflammation of the upper airway. Patients present with a prodrome for 1-2 days before developing a barking, seal-like cough interspersed with inspiratory stridor. Cough will be worse at night. Diagnose with CXR showing *steeple sign* (subglottic narrowing). IF causing significant resp impairment, supplement with O2 and give racemic epi and steroids (usually dexamethasone).

Barrett's esophagus

A sequelae of GERD caused by conversion of normal stratified squamous epithelium of the esophagus into columnar epithelium. Associated with esophageal adenocarcinoma

osteomyelitis

A sequelae of sickle cell due to chronically infarcted bones. Most often due to *Staph aureus*, but with increased incidence of *Salmonella* compared to the general population.

avascular necrosis

A sequelae of sickle cell that resembles a fracture, but is due to chronic vaso-occlusive crisis. Surgery is indicated, but only after 4-6 months of conservative treatment (NSAIDs, rest, crutches).

varicella zoster ("shingles")

A seuelae of varicella infection seen in immunocompromised adults who had chicken pox; reactivation causes an extremely painful prodrome that precedes a rash in the same dermatomal distribution of the pain and never crosses the midlines. Acyclovir may decrease duration and pain.

acute chest syndrome

A sickle cell crisis defined by new pulmonary infiltrate associated with resp symptoms (cough, SOB, chest pain) and hypoxemia. May be severe and cause up to 25% of sickle cell deaths. Causes include infection, sickling, fat embolism, etc. Treat with hydration, oxygen, and pain management.

Eisenmenger syndrome

A situation in which a left-to-right shunt such as a VSD causes pulmonary vessel hypertrophy which over time leads to increased PVR and pulmonary HTN, to the point that PVR exceeds SVR, reversing the left-to-right shunt to a right-to-left shunt.

*Frostbite* is the condition in which tissue is frozen and destroyed. There is initial stinging, followed by aching, culminating in numb areas. Once rewarmed, the area becomes red, blotchy, and painful. Gangrene may develop.

A skier recently rescued from a snowbank following an avalanche complains about his feet. Upon rescue they were whitish yellow and numb, but now they are blotchy and painful. What is the diagnosis?

erysipelas

A skin infection that involves the dermal lymphatics, usually caused by GABHS (Strep pyogenes). Clinical features include tender, erythematous skin with a distinct border; face and scalp are common locations. Diagnosis is by visual inspection and management is by systemic antibiotics. Complications include bacteremia, PSGN, and necrotizing fasciitis.

cellulitis

A skin infection within the dermis, usually caused by Strep pyogenes or Staph aureus. Infection is usually caused by a break in the skin barrier allowing bacteria to gain entry beyond the epidermis. Characterized by erythema, warmth, and tenderness; the infected skin border is indistinct. Diagnosis is by visual inspection and blood cultures are seldom positive. Manage with antibiotics.

patellofemoral syndrome

A slight malalignment of the patella that causes knee pain. Pain is worse with activity or with walking up and down stairs. It is relieved with rest. Physical exam may show patella in a lateral position. Diagnosed by "sunrise view" radiograph of knee. Manage with rest, stretching, and strengthening of the medial quadriceps.

cholesteatoma

A small epithelium-lined sac in the middle ear containing debris. Resembles a discrete, whitish polyp that extends through the tympanic membrane. Can present in children with recurrent otitis media, or in the face of a chronically draining ear. The mass can grow aggressively, leading to CNS complications like facial nerve damage, hearing loss, and intracranial extension. Referral to an otolaryngologist is required; a CT scan of the temporal bones can define the extent of the disease.

retrophayngeal abscess

A soft-tissue infection of the neck due to bacterial infection of the potential space between the pharynx and vertebral fascia. Most often preceded by URI. Occurs most commonly in ages 6 months to 6 years. Symptoms include fever, odynophagia, dysphagia, neck pain with extension, and muffled voice. On lateral x-ray, there is classic widening of the prevertebral space; a CT with contrast confirms the diagnosis.

hemorrhagic disease of the newborn

A special type of vitamin K deficiency. IT may occur early (within 24 hours after birth), within the first week of life (the classic form), or late (1-3 months after birth. Characterized by serious bleeding in the early and late forms, but classic disease generally presents only with cutaneous bleeding, hematemesis, and bleeding from the circumcision site or umbilical cord. CNS bleeding may occasionally occur.

*Turner syndrome:* diagnose with karyotype analysis showing 45,XO. IF karyotype is normal but suspicion remains high, fluorescence in situ hybridization is performed to detect mosaicism that is beyond the resolution of standard karyotyping.

A sporadic chromosomal disorder caused by complete or partial deletion of an X chromosome, resulting in poorly developed "streak ovaries" on pelvic ultrasound. Due to loss of ovarian function, patients with this disorder have low estrogen levels, resulting in primary amenorrhea and absent breast development. FSH and :LH levels will be increased due to low estrogen removing the negative feedback on the HPO axis.

spondylolysis

A stress fracture in the pars interarticularis (bone that connects the superior and inferior articular facets of a vertebral body) secondary to repetitive hyperextension of the spine, which can occur in gymnastics, tennis, or diving. It typically involves the lumbar region. Results in localized back pain which increases with hyperextension. Diagnosed by bone scan and managed with rest and analgesics.

cephalohematoma

A subperiosteal hemorrhage secondary to birth trauma confined and limited by the cranial sutures, usually involving the parietal or occipital bones.

acute promyelocytic leukemia

A subtype of AML in which Auer rods are observed

spasmodic croup

A subtype of croup with acute onset, usually at night; typically recurs and resolves without treatment and is due to a hypersensitivity reaction.

tinea capitis

A superficial dermatophytosis that most commonly occurs in children and immunocompromised patients. It causes a scaly, erythematous patch that can progress to alopecia with inflammation, lymphadenopathy, and scarring. There is often a residual "black dot" noted at points of broken hairs. Oral griseofulvin or terbinafine are the preferred initial treatment.

impetigo

A superficial skin infection involving the upper dermis; usually due to Staph aureus or Strep pyogenes. Features include honey-colored crusts or bullous lesions on the face, espeically around the nares. Fever is generally absent. Infection is easily transmitted. Diagnosis can be made by visual inspection; cultures are not required.

Ross procedure.

A surgical procedure to fix aortic stenosis with insufficiency or aortic stenosis refractory to balloon valvuloplasty. The aortic valve is replaced with the patient's own pulmonary valve. Alternatively, a prosthetic valve can be used.

Fontan procedure

A surgical procedure to treat tricuspid atresia, in which flow from the inferior vena cava is directed into the pulmonary arteries, usually by means of an extracardiac conduit or intra-atrial baffle or tunnel.The SVC is usually anastomosed to the right pulmonary artery as well. The net result is systemic venous return directed to the pulmonary artery.

Henoch Schonlein Purpura

A systemic *IgA mediated vasculitis* involving the skin, joints, GI tract, and kidneys. The median age of onset is *5 years.* Males are more commonly affected. Often preceded by a *viral syndrome or URI.* Most patients recover within 4 weeks, although it recurs at least once in 50% of patients.

*Frostnip* is manifest by small, firm, white, cold patches of skin in exposed areas; treatment is rewarming the areas before they become numb.

A teen, just back from a skiing trip, has blistering and peeling of several areas on her face; she reports the lesions started as firm, cold, white areas that felt stinging at the time and are now more sensitive than the surrounding skin. What is the likely diagnosis?

*Ectopic pregnancy:* occurs when a developing blastocyst implants outside the uterus. Evaluation includes urine pregnancy test and transvaginal ultrasound.

A teenage female patient with a history of abnormal vaginal discharge suspicious for STD presents with vaginal bleeding and pelvic pain. What should you suspect?

failure to thrive

A term used to describe a growth rate less than expected for a child; of particular concern when child's wieght *crosses two major percentile bars on the NHS charts.* Weight is generally affected before length, which in turn is usually affected before head circumference.

Adam's forward bending test

A test for scoliosis where, as the patient bends forward, a hump, representing posterior displacement of the curved spine, appears. Pain is absent.

This toddler likely has *constipation.* Behavioral modifications to treat includes increasing water and fiber intake, limiting cow's milk to <24 oz a day, and sitting on the toilet after each meal. If these interventions are not helpful, oral laxatives (PEG or mineral oil) should be initiated and titrated to produce daily soft bowel movements. Enemas should be used only if oral laxative therapy does not relieve severe constipation.

A toddler who is going through dietary changes, toilet training, and school initiation presents with straining, hard stools, and anal fissures. What is the likely diagnosis and treatment?

scarlet fever

A toxin-mediated bacterial illness that results in a rash; may develop during any GABHS infection. The exanthem begins on the trunk and moves peripherally. Skin is erythematous with tiny skin-colored papules and has the texture of sandpaper. The rash blanches with pressure. Petechiae are often localized within skin creases in a linear distribution ("Pastia's lines"). Desquamation occurs as infection resolves.

toxic shock syndrome (TSS)

A toxin-mediated illness characterized by fever, shock, desquamating skin rash, and multiorgan dysfunction. Most commonly associated with Staph aureus and GABHS. Classically associated with tampon use.

air enema

A treatment for intussusception in children which is both diagnostic and therapeutic

exchange transfusion

A treatment for rapidly rising bilirubin levels secondary to hemolytic disease

Trisomy 13 (Patau Syndrome)

A trisomy associated with holoprosencephaly, severe mental retardation, microphthalmia, and cleft lip/palate.

Rh hemolytic disease

A type of alloimmune hemolytic disease which occurs when an Rh- mother produces antibodies to the Rh antigen on her Rh+ fetus. In subsequent pregnancies, antibodies pass from mother to second fetus, causing hemolysis that presents as severe jaundice, anemia, hepatosplenomegaly, and hydrops fetalis. A direct Coombs test is strongly positive.

purulent pericarditis

A type of pericarditis caused by bacterial infection, either primary or disseminated from pneumonia or meningitis. Staph aureus and Strep penumo are the most common agents. Patients with this disease have a high incidence of constrictive pericarditis owing to the intense inflammatory response.

premature thelarche

A type of precocious puberty where there is visible or palpable breast tissue only, with no other secondary sex characteristics. No pubic hair should be apparent. This is very common and benign and usually presents in the first two years of life, usually because there is transient release of low levels of estrogen. No workup is necessary and no treatment is indicated.

Lymphocytic thyroiditis

A typical organ-specific autoimmune disease characterized by lymphocytic infiltration of the thyroid gland, with or without goiter. It is the most common cause of juvenile hypothyroidism, peaking in adolescence, and affecting as many as 1% of schoolchildren. The condition is 4 to 7 times more prevalent in girls than in boys and may persist for many years without symptoms. Patients are initially euthyroid (with the occasional child having elevated TSH levels), but, with the eventual atrophy of the gland, they become hypothyroid (decreased triiodothyronine [T3] and T4, elevated TSH). Spontaneous remission can occur in one-third of the affected adolescents.

labial adhesions

A usually benign condition in which the labia minora are fused. Fusion ranges from a small area to the entire area between the clitoris and the fourchette. It is most common in young girls who are in the low estrogen state of preadolescence. Sometimes urine pooling can cause an increased risk of UTI. Treatment can be merely observation, as the condition should resolve with the estrogenization that occurs with puberty. Nightly application of an estrogen cream for a week resolves this condition in the majority of patients. However, as recurrence is common, patients must be instructed to apply daily petrolatum for a month or two after separation.

juvenile chronic myelogenous leukemia (JMML)

A variant of CML which occurs predominantely in infants and children younger than 2. Sometimes characterized by abnormalities of chromosome 7 or 8. the Philadelphia chromosome is absent. Presents with fever, chronic eczemalike facial rash, suppurative lymphadenopathy, petechiae and purpura, and moderate leukocytosis, anemia, and thrombocytopenia. Treatment is bone marrow transplant.

type III RTA

A variant of type I renal tubular acidosis complicated to proximal tubular bicarb wasting during infancy; treat with large doses of oral alkali.

non-Hodgkin's lymphoma

A very aggressive male predominant cancer. Associated with immunodeficiency states (HIV, Wiskott Aldrich, ataxia telangiectasia, EBV). Indicence increases after 5 years. Three subtypes.

atopic dermatitis

A very common condition characterized by pruritus, erythema, and scaly lesions on the skin. Pathogenesis involves epidermal dysfunction due to improper synthesis of stratum corneum components. Excessive bathing, dry environments, stress, overheating, and irritating detergents can be triggers. Lesions usually begin with pruritus and evolve to erythematous papules and scaly plaques. Infants typically have a distribution in the face, scalp, and extensor surfaces; in older children and adults, lesions can also be seen in flexural creases.

varicella ("chickenpox")

A viral infectious rash with a vague prodrome. Rash starts on trunk and heads with following outward spread to extremities. There are vesicles with an erythematous base in different stages. Scarring and secondary infections are common complications.

Transmission of *HSV* from mother to newborn can happen in utero, intrapartum, and postnatally. Intrapartum transmission is most common. Infants born vaginally to a mother with a primary genital herpes infection are at highest risk for disease, with up to a 50% possibility of perinatal transmission; the risk to a baby born to a mother with a recurrent HSV infection is much lower. About half the infants of congenital HSV are born to mothers who are unaware of their infection. Infants can display isolated CNS involvement, isolated cutaneous infection, or systemic generalized infection. Treatment usually is with acyclovir; even with therapy, morbidity is high in infants with CNS involvement.

A week-old infant presents with fever and focal seizure. What congenital infection might cause this?

leukocoria

A white pupil, caused by either cataract or retinoblastoma usually

*West Syndrome:* also known as infantile spasm. Syndrome confirmed by interictal EEG showing hypsarrhythmia. Treatment is *ACTH* which can help with spasms, normalizing EEG. Almost all kids with infantile spasm will have some sort of intellectual disability, 50% of which are severe.

A young child (under 1 year) has symmetric jerking of the head, trunk, or extremities without fever. What should you consider?

hearing impairment

A young infant with inconsistent response to physician's questions, trouble listening and following directions, and lack of progression of language skills raises concerns for what problem?

epiglottitis

Acute inflammation and edema of the epiglottis, arytenoids, and aryepiglottic folds; most common in children 2-7 years old. Hib used to be the most common cause, but due to immunization it is now rare, and *GABHS*, Strep pneumo, and Staph species cause the majority of cases now.

Friedreich ataxia

Adolescents often present with this disease with *progressive gait ataxia* and other neurologic manifestations such as *dysarthria* and loss of vibration and position sense. It is an autosomal recessive disorder caused by a GAA expansion resulting in loss of function mutation in the frataxin gene. The neurologic symptoms are a result of degeneration of the spinocerebellar and posterior column tracts. Other clinical features include *hypertrophic cardiomyopathy*, DM, and scoliosis. the most common cause of death is *cardiac dysfunction* from CHF or arrhythmia as a sequelae of the hypertrophic caridomyopathy.

The child in the question likely has *Kasabach-Merritt phenomenon*, which is seen with large vascular anomalies (ie, Kaposiform hemangioendothelioma and tufted angioma). Platelet and RBC sequestration within the vascular tumor causes peripheral thrombocytopenia, coagulopathy, and microangiopathic hemolytic anemia. Treatment options include corticosteroids, α-interferon, and vincristine. Surgery frequently results in excessive bleeding.

After being delivered following a benign gestation, a newborn infant is noted to have a platelet count of 35,000/μL, decreased fibrinogen, and elevated fibrin spilt products. On examination, you note a large cutaneous hemangioma on the abdomen that is purple and firm. What is the diagnosis?

Get a *contrast enema*, which can identify both meconium ileus and Hirschprung disease

After pneumoperitoneum has been exluded by abdominal x-ray, what is the best next step in working up an infant with bilious emesis who has not passed meconium?

generalized seizure

After this type of event, children often have a phase of sleepiness and confusion for minutes to hours.

Chlamydia trachomatis Neisseria gonorrhoeae

All sexually active women under age 24 should be screened for what?

congenital hypothyroidism phenylketonuria galactosemia

All states screen for what congenital conditions? Note that all the screen conditions are treatable and if not detected early can lead to irreversible brain injury

*Marfan syndrome* is a genetic disorder transmitted as an autosomal dominant trait with variable expression. People with this disorder typically have tall stature, arachnodactyly, subluxation of the lens, dilatation of the aorta, and dissecting aneurysm. Mental retardation is not a part of this syndrome. Vascular complications can be serious if not identified early; aortic dissection can lead to sudden death.

An 11-year-old boy is the tallest child in his sixth grade class. He is thin, has a high arched palate, mild scoliosis, and joint hyperextensibility. He is an honor student, and enjoys basketball but must use prescription sports goggles, so he can see the ball on the court. What is the likely diagnosis?

The child in the question likely has *cyclic neutropenia*, an autosomal dominantly inherited condition where the neutropenia occurs about every 3 weeks resulting in the signs and symptoms in the question and lasts about 3 to 5 days. It is often treated with recombinant human granulocyte colonystimulating factor (rhG-CSF).

An 11-year-old child arrives to your hospital with a history of recurrent episodes of oral ulcers, fever, pharyngitis, and lymphadenopathy. These fever episodes, which also have been noted in other family members, occur about every 21 days and some of the episodes have been associated with pneumonia and sepsis. What is the likely diagnosis?

atopic dermatitis

An IgE mediated skin condition typically seen in younger children. Appears as scaly skin on extensor surfaces (in infants/young children) or flexor surfaces (older children or adults). Skin can be pruritic and become secondarily infected if severely excoriated. Causes related to environmental expsoure or food ingesion. Use emollients and moisturizers as baseline therapy. Topical steroids can be used as first line for exacerbations.

The clavicle is the most commonly fractured bone in the delivery process. While some fractures are identified at birth by finding crepitus on physical examination of the shoulder, others may not be identified until callus formation is noted at about a week of age. Clavicular fracture may happen in any delivery, although there is higher risk with large-for-gestational-age infants. Reassurance and supportive care can be given.

An LGA baby has a mass palpated on the right shoulder. What is the probable cause?

Guillain Barre syndrome

An LP showing albuminocytologic dissociation (increased CSF protein in the absence of elevated cell count) is evidence of which disease?

Gaucher disease

An LSD caused by glucocerebrosidase deficiency; the most common gangliosidosis. Autosomal recessive. Features include hepatoplenomegaly, thrombocytopenia, Erlenmeyer flask distal femur. Manage with enzyme replacement therapy.

Niemann Pick disease

An LSD caused by sphingomyelinase deficiency. Presents by age 6 months with progressive neurodegeneration, ataxia, seizures, hepatosplenomegaly, and cherry red macula.

Norwalk virus

An RNA virus spread by the fecal-oral route and linked to utbreaks of gastroenteritis in all age groups, particularly in close dpopulations (daycares, schools, cruises). Presents with *prominent vomiting* and diarrhea. Duration of illness is 48-72 hours. Diagnosis is clinical and management is supportive.

Wiskott-Aldrich

An X-linked (in boys) immunodeficiency with infections, thrombocytopenia, and eczema. IgE and IgA increased. Patients may need bone marrow but rarely survive to adulthood. Treat with IVIG, splenectomy, supportive treatment.

Fragile X syndrome

An X-linked disorder caused by CGG repeats. Anticipation occurs. It is most severe in males as it is X-linked. It is the *most common inherited cause of mental retardation.* It also features large ears, macrocephaly, large tests, and commonly ADHD.

hemophilia B (Christmas disease)

An X-linked disorder causing a deficiency in factor IX; it has similar features to hemophilia A. Prolonged aPTT is characteristic. Low factor IX is seen. PT and platelet count are normal. Manage with factor IX replacement.

Wiskott Aldrich

An X-linked disorder characterized by combined immunodeficiency, eczema, and congenital thrombocytopenia with small platelets. Caused by mutation of a gene on the short arm of the X chromosome.

Wiskott Aldrich syndrome

An X-linked disorder characterized by thrombocytopenia with unusually small platelets, eczema, and defects in T and B cell immunity.

Duchenne muscular dystrophy

An X-linked myopathy characterized by myofiber degeneration. Onset between 2-5 years. Caused by a deletion in the gene for a cytoskeletal protein that associates with actin; absence causes weakness and rupture of the plasma membrane, leading to muscle injury and degeneration. Eventually, muscle is infiltrated with lymphocytes and replaced with fibroblasts and lipid deposits.

Lesch-Nyhan syndrome

An X-linked recessive disorder caused by deficiency of HGPRT, an enzyme involved in purine metabolism. This deficiency results in the accumulation of hypoxanthine and uric acid in the urine, serum, and CNS. Presentation begins in early infancy with delayed milestones and hypotonia; by age 3, they can develop dystonia and chorea as well as spasticity and hyperreflexia. A distinct finding is self-mutilation (biting of the fingers, banging of the head). Can lead later in life to gouty arthritis and obstructive nephropathy.

Hunter syndrome

An X-linked recessive mucopolysaccharidosis with no corneal clouding. Features include hepatosplenomegaly, hearing loss, stiff contracted joints, and dysostosis multiplex.

gastroschisis

An abdominal wall defect lateral to the umbilical cord insertion site with herniation of uncovered bowel; an isolated defect with no associated conditions. Can result in oligohydramnios.

developmental dysplasia of the hip

An abnormal development of the femoral head and acetabulum; will present in newborns as asymmetric gluteal, thigh, or inguinal creases and apparent leg-length discrepancy. It may also present with palpable clunk on Barlow and Ortolani maneuvers, but these maneuvers may be negative or equivocal. It should be evaluated with *hip ultrasound.* Treatment is with a Pavlik harness.

sickle cell

An abnormal variant of hemoglobin caused by a valine substitution for glutamine at the 6th position. Autosomal recessive (one copy of the abnormal hemoglobin makes you a carrier). Results in cells which warp under the influence of hypoxemia, acidosis, or other stressors, deforming into rigid cells that can't pass through capillaries. The end result is hypoxemia and hemolysis.

acute renal failure

An abrupt decrease in the ability to excrete nitrogenous wastes. Symptoms vary but can include lethargy, nausea, vomiting, resp distress, HTN, and seizures. Oliguria can occur (<1 mL/kg/hr urine output) but it may also be nonoliguric (normal urine output).

hydrocele

An accumulation of fluid in the tunica vaginalis. Small ones usually resolve spontaneously in the first year of life. Larger ones or those that have a variable fluid level with time will likely need surgical repair.

Addison's disease

An acquired adrenal insufficiency that results from autoimmune destruction f the adrenal cortex by lymphocytic infiltration. Antibodies to the adrenal gland may be detected, and there is often other associated endocrinopathies (Hashimoto's thyroiditis, type I diabetes). Diagnosed by an ACTH stimulation test, which measures adrenal cortisol reserve by comparing baseline cortisol level with cortisol level 1 hour after ACTH injection. Normally, cortisol level doubles in response to ACTH stimulation; a blunted response usually indicates some type of primary adrenal insufficiency.

aplastic crisis

An acute drop in hemoglobin accompanied by a low reticulocyte count (<1%) without splenomegaly. Common in sickle cell patients. Anemia is typically severe (<6 g/dL) and patients may have pallor, fatigue, and a functional systolic murmur due eto hyperdynamic blood flow. WBCs and platelet counts will be normal. Most commonly caused by parvovirus B19, which infects and destroys RBC precursors.

meconium aspiration syndrome

An acute resp disorder caused by aspiration of meconium into the airways of the fetus or neonate. Newborns will present with resp distress. CXR reveals increased lung volume with diffuse patchy areas of atelectasis and parenchymal infiltrates alternating with hyperinflation. Pneumothorax or pneumomediastinum may occur. Prevention by suctioning is paramount.

pertussis ("whooping cough")

An acute resp infection caused by Bordetella pertussis. Infants under 6 months are most at risk.

Bell palsy

An acute, unilateral facial nerve palsy that begins about 2 weeks after a *viral infection.* Although the exact pathophysiology is unknown, reactivation of herpes simplex or varicella-zoster virus seems to be the most common cause; demyelination through an autoimmune process or allergic inflammation may also play a part in some cases. On the affected side, the upper and lower face are typically paretic, the mouth droops, and the patient cannot close the eye. Treatment consists of maintaining moisture to the affected eye (especially at night) to prevent keratitis. Complete, *spontaneous resolution* occurs in about 85% of cases, 10% of cases have mild residual disease, and about 5% of cases do not resolve.

peritonsillar abscess

An adolescent with hot potato voice and visible tonsillar bulge which displaces the uvula to one side likely has this disease.

*Slipped capital femoral epiphysis* is a disease of unknown etiology and occurs typically in adolescents; the disorder is most common among obese boys with delayed skeletal maturation or in thin, tall adolescents having recently enjoyed a growth spurt. Onset of this disorder is frequently gradual; pain referred to the knee in 20% of cases can mask the hip pathology.

An afebrile, obese 14-year-old boy has developed pain at the right knee and a limp. What diagnosis do you suspect?

ABO hemolytic disease

An alloimmune hemolytic anemia that occurs when an O-type mother has a fetus type A, B, or AB. The mother produces antibodies to either the A or B blood group antigen that then pass to the fetus, causign hemolysis with resultant jaundice. A direct Coombs is weakly positive. This can occur in the first pregnancy.

malrotation

An anatomic abnormality of intestinal rotation that allows the midgut to twist around the superior mesenteric vessels (ie volvulus). This may obstruct and infarct the bowel. Has a 2:1 male predominance. Associated wtih small bowel atresia, Hirschprung's, and intussusception.

relative polycythemia

An apparent increase in RBC mass caused by a decrease in plasma volume. Most commonly caused by dehydration.

PID

An ascending STD in which pathogens from the cervix spread to the uterus and fallopian tubes. Can be caused by chlamydia, gonorrhea, or other microbes. Presents with lower abdominal pain and tenderness, cervical motion tenderness, and adnexal tenderness. There should also be one of the following: fever, leukocytosis, inflammatory pelvic mass on exam or US, elevated ESR or CRP, and lab evidence of infection.

systemic-onset juvenile idiopathic arthritis

An auto-inflammatory disease in children characterized by long-standing daily fever, arthritis of 1+ joints, and a characteristic pink macular rash; lab findings include leukocytosis, thrombocytosis, elevated ESR/CRP, and anemia.

Acute disseminated encephalomyelitis (ADEM)

An autoimmune demyelinating disease seen in children less than 10 years of age. It may follow many different types of infections, including upper respiratory tract infections, varicella, mycoplasma, herpes simplex virus, rubella, rubeola, and mumps; it may also follow immunizations. An MRI will show multiple "white" plaques scattered across the brain representing areas of demyelination. Treatment is with high dose steroids.

Graves' disease (diffuse toxic goiter)

An autoimmune disorder characterized by autonomous production of excessive thyroid hormone mediated by a TSH look-alike IgG antibody (TSI). More common in females. Manage with PTU or methimazole and thyroidectomy or radioactive iodine if those fail.

celiac disease (gluten-sensitive enteropathy)

An autoimmune disorder characterized by intolerance to gluten (found in *wheat, rye , barley, and oats*), which results in mucosal damage. The primary symptoms are diarrhea, vomiting, bloating, anorexia, and sometimes FTT. Abdominal pain and large, foul-smelling stools are also common.

Hashimoto's disease (chronic lymphocytic thyroiditis)

An autoimmune disorder characterized by lymphocytic infiltration of the thyroid gland, resulting in varying degress of follicular fibrosis and atrophy. More common in girls. Presents with goiter, short stature, transient hyperthyroidism. Treat with L-thyroxine.

myasthenia gravis

An autoimmune disorder that presents with progressive weakness or diplopia. Caused by antibodies against the ACh receptor at neuromuscular junctions. The neonatal form causes transient weakness in the newborn period due to maternal transfer of AChR antibodies. A juvenile form presents in childhood and mostly affects girls.

HCM

An autosomal dominant cardiac defect characterized by LVH in the absence of any systemic or cardiac disease known to cause the hypertrophy. It causes poor LV filling and can cause a left ventricular outflow tract (LVOT) obstruction. It is the *most common cause of sudden death in young athletes.* Symptoms may be absent until syncope or sudden death occurs, or may include chest pain and exercise intolerance. Physical exam shows a classic harsh systolic ejection murmur at the apex that is accentuated with physiologc maneuvers that reduce LV volume (Valsalva or standing).

hereditary elliptocytosis

An autosomal dominant cause of normocytic anemia due to a defect in spectrin structure; majority of patients are aysmptomatic though they may have jaundice at birth. Elliptical RBCs are seen on smear. Treat with splenectomy if hemolysis is a concern; no treatment necessary in absence of hemolysis

Marfan syndrome

An autosomal dominant connective tissue disorder due to a gene defect on chromosome 15 that codes for *fibrillin*. Features include tall stature with long extremities, joint laxity, chest wall deformities, and scoliosis or kyphosis. A decreased upper to lower segment ratio is suggestive. There is also commonly lens subluxation and retinal detachment, as well as aortic root dilatation, mitral valve prolapse, and aortic regurgitation. Complications include sudden death as a result of aortic dissection.

osteogenesis imperfecta

An autosomal dominant connective tissue disorder that presents with frequent fractures, joint hypermobility, and dentinogenesis imperfecta. Additional features include blue sclerae, hearing loss, and short stature.

myotonic muscular dystrophy

An autosomal dominant disorder characterized by grip myotonia (delayed muscle relaxation), facial weakness, foot drop, dysphagia, and cardiac conduction anomalies. Other problems include cataracts, testicular atrophy, infertility, and baldness.

hereditary hemorrhagic telangiectasia

An autosomal dominant disorder characterized by locally dilated and tortuous veins and capillaries of the skin and mucous membranes.

Alagille syndrome

An autosomal dominant disorder characterized by paucity of intrahepatic bile ducts and multiorgan involvement. Presents with features of cholestatic liver disease, debilitating *pruritus*, unusual facies (broad forehead, deepset and wide spaced eyes, saddle nose, large ears), pulmonary outflow obstruction, renal disease, posteiror embryotoxon, musculoskeletal anomelies, growth failure, pancreatic insufficiency, and hypercholesterolemia. Diagnosis is clinical and management is supportive.

congenital myotonic dystrophy

An autosomal dominant muscle disorder that presents in the newborn period with weakness and hypotonia. It is a trinucleotide repeat disorder with autosomal dominant inheritance and variable penetrance. Transmission is through the affected mother in over 90% of cases. Antenatal history may reveal polyhydramnios caused by poor swallowing in utero; neonatal history often includes feeding and respiratory problems.

Tay Sachs disease

An autosomal recessive LSD caused by hexosaminidase A deficiency; incidence is higher in Ashkenazi Jews. Presents with hyperacusis (increased sensitivity to sound), macrocephaly, cherry red macula, progressive blindness, seizures, and severe developmental delay.

tyrosinemia type I

An autosomal recessive amino acid metabolism disorder. Characterized by episodes of peripheral neuropathy, chronic liver disease, odor of cabbage or rotting fish, and renal tubular dysfunction. Diagnosed by succinylacetone in the urine. Dietary phenylalanine, tyrosine, and NTBC must be restricted. Liver transplant is used for management.

PKU

An autosomal recessive amino acid metabolism disorder. Presents with developmental delay, mousy or musty odor, mental retardation, eczema, light eyes and hair. Diagnosed with increased phenylalanine to tyrosine ratio in the serum and treated with a phenylalanine restricted diet.

maple syrup urine disease

An autosomal recessive amino acid metabolism disorder. Presents with progressive vomiting and poor feeding, lethargy, sweet odor in the urine, and episodes of hypoglycemia and severe acidosis. Diagnosed by increased serum and urine branched chain amino acids. Managed with dietary protein restriction.

pyruvate kinase deficiency

An autosomal recessive cause of normocytic anemia that results in decreased production of pyruvate kinase isoenzyme, leading to ATP depletion and decreased RBC survival. Presents with pallor, jaundice, and splenomegaly. KErnicterus has been recorded. Blood smear will show *polychromatic RBCs.* Diagnose by finding decreased PK activity in RBCs. Manage with transfusions (and splenectomy for severe disease).

Schwachman Diamond syndrome

An autosomal recessive condition characterized by decreased neutrophil chemotaxis, cyclic neutropenia, and pancreatic exocrine insufficiency. Patients present with recurrent soft tissue infection, chronic diarrhea, and failure to thrive.

Fanconi anemia

An autosomal recessive congenital aplastic anemia with mean age of bone marrow failure onset of 7 years. Typical presentation is with ecchymosis and petechiae. Skeletal abnormalities, inlcuding short stature and absence of hypoplasia of the thumb and radius, are present in almost all patients. In addition, skin hyperpigmentation and renal abnormalities are seen. Treat with transfusions of RBCs and platelets as needed and ultimately bone marrow transplant.

congenital adrenal hyperplasia

An autosomal recessive congenital enzyme deficiency in the adrenal cortex which causes primary adrenal insufficiency of childhood. The most common cause of ambiguous genitalia when no gonads are palpable.

*Homocystinuria:* diagnosed by finding increased methionine in the urine and plasma, or by a positive urinary cyanide nitroprusside test. Management includes a methionine restricted diet, aspirin to decreased thromboembolism risk, and B9 and B6 supplementation.

An autosomal recessive defect in amino acid metabolism. Presents with marfanoid body habitus without arachnodactyly, downward lens subluxation (as opposed to upward lens subluxation in Marfan's), hypercoagulable states, CV abnormalities like mitral or aortic regurg (no aortic dilatation, in contrast to Marfan's), scoliosis, and developmental delay.

Wilson's disease

An autosomal recessive defect in copper excretion that causes copper deposition in the liver, brain, eyes, and heart. Kayser-Fleischer rings can be seen in hte peripheral cornea; neurologic findings and hepatic dysfunction are common. Diagnosed by decreased serum deruloplasmin, elevated serum and urine copper, and copper deposition in hepatocytes obtained by liver biopsy. Manage by avoiding copper-containing food (nuts, liver, shellfish, chocolate), chelation therapy with oral penicillamine and zinc salts, and sometimes liver transplant.

cystinuria

An autosomal recessive disorder caused by a defect in renal reabsorption of cystine, lysine, arginine, and ornithine. Leads to kidney stones. Features include UTIs, dysuria, abdominal or back pain, urgency, and urinary frequency.

Hartnup disease

An autosomal recessive disorder caused by a defect in the transport of neutral amino acids; most patients are asymptomatic but some may present with intermittent ataxia, photosensitive rash, mental retardation, and emotional lability.

galactosemia

An autosomal recessive disorder caused by galactose-1-phsophate uridyltransferase deficiency; should be suspected in any newborn with hepatomegaly and hypoglycemia. Clinical features begin after newborn feeds on cow's milk or breast milk (which contain galactose) and include vomiting, diarrhea, FTT, hepatomegaly, cataracts with characteristic oil droplet appearance, and renal tubular acidosis. Diagnosed via nonglucose reducing substance in urine and confirmed with enzyme deficiency in RBCs; manage with galactose free diet (soy or elemental formulas). Prognosis is good if treated early; if not treated, mental retardation and sepsis are common sequelae. Nearly all females suffer from ovarian failure if not treated.

von Gierke disease (GSD type I)

An autosomal recessive disorder caused by glucose 6 phosphatase deficiency. Presents with persistent hypoglycemia, hepatomegaly, metabolic acidosis, hypertriglyceridemia, and enlarged kidneys. Manage with a high-complex carb diet. Patients at risk for HCC.

ataxia telangiectasia

An autosomal recessive disorder characterized by combined immunodeficiency, cerebellar ataxia, oculocutaneous telangiectasias, and predisposition to malignancy. Results from a mutation of a gene on the long arm of *chromosome 11.* The gene product is involved in cell cycle control, DNA recombination, and cellular responses to DNA damage.

Bernard Soulier syndrome

An autosomal recessive disorder characterized by decreased platelet adhesion as a result of absence of platelet membrane glycoproteins. Severe hemorrhage may occur and large unusual platelets are seen on blood smear.

Glanzmann's thrombasthenia

An autosomal recessive disorder characterized by diminished ability of platelets to aggregate and form a clot as a result of deficient adhesive glycoprotein IIb/IIIa on the platelet cell membrane.

thrombocytopenia absent radius syndrome (TAR)

An autosomal recessive disorder characterized by thrombocytopenia and limb abnormalities, especially absence of the radius (note that the thumb is present, in contrast to Fanconi anemia). Cardiac and renal disease may be present. Thrombocytopenia improves in the second or third year of life.

Chediak-Higashi

An autosomal recessive disorder leading to indiscriminate lyososmal fusion. It will present with albinism, neuropathy, and neutropenia. Look for giant granules in neutrophils. Infections typically involve mucous membranes and skin (Staph aureus). Treat infections aggressively.

homocystinuria

An autosomal recessive disorder that results from errors in methionine metabolism secondary to cystathionine synthase deficiency. Patients will have a Marfanoid body habitus (tall stature, long thin limbs, joint hyperlaxity, ectopia lentis, chest deformities) as well as fair hair and eyes, development delay, and an increased risk for CVAs. Diagnosis made based on elevated homocysteine and methionine levels; treatment includes vitamine B6, folate, and B12 to lower homocysteine levels. In addition, antiplatelets or anticoagulation should be given to prevent CVAs.

homocystinuria

An autosomal recessive metabolic disease caused by deficiencies of cystathionine β-synthase, methylenetetrahydrofolate reductase, or the coenzyme for N5-methyltetrahydrofolate methyltransferase. Manifestations include poor growth, arachnodactyly, osteoporosis, dislocated lenses, and mental retardation. In addition, thromboembolic phenomena may be seen in the pulmonary and systemic arteries and particularly in the cerebral vasculature; vascular occlusive disease is, in turn, one of the many causes of acute infantile hemiplegia.

*pigmented gallstones* (cholecystectomy expected to occur early)

An early complication of sickle cell disease due to constant hemolysis and elevated bilirubin

infantile spasm (West syndrome)

An epilepsy syndrome with age of onset 3-8 months. Rare in children older than 2 years. *Tuberous sclerosis* is the most commonly identified cause. Presents as brief myoclonic jerks lasting 102 seconds each, occurring in clusters of 5-10 seizures over 3-5 minutes. The jerks consist of sudden arm extension or head and trunk flexion (jackknife seizures). EEG shows characteristic hypsarrhythmia pattern. Treat with ACTH.

Prader-Willi syndrome

An example of genomic imprinting caused by absence of a region on the paternally derived chromosome 15. Features include almond eyes, fishlike mouth, hypotonia, short stature, mental retardation, and hypogonadism. There is failure to thrive in the first year of life followed by obesity as a result of hyperphagia later in childhood.

primary polycythemia (polycythemia vera)

An extremely rare cause of polycythemia during childhood. A malignancy involving the RBC precursor.

trachoma

An eye disease due to infection with Chlamydia trachoamtis serotypes A, B, and C. Presents with follicular conjunctivitis and inflammation. There is often concomitant nasopharyngeal infection (rhinorrhea, pharyngitis). Repeated or chronic infection causes inversion of the eyelashes and scarring of the cornea

*Myocarditis:* the findings of pallor, dyspnea, tachypnea, tachycardia, and cardiomegaly are common in congestive heart failure regardless of the cause. The constellation of findings in the question suggests myocarditis as the etiology of this patient's condition. The most common causes of myocarditis include adenovirus and coxsackievirus B, although many other viruses can cause this condition.

An ill-appearing 9-week-old baby girl is brought to the emergency room. She is pale and dyspneic with a respiratory rate of 80 breaths per minute. Heart rate is 195 beats per minute, heart sounds are distant, a gallop is heard, and she has cardiomegaly on x-ray. An echocardiogram demonstrates poor ventricular function, dilated ventricles, and dilation of the left atrium. An ECG shows ventricular depolarization complexes that have low voltage. What is the likely diagnosis?

scarlet fever

An illness caused by group A strep that develops most commonly in children who have strep pharyngitis. Certain strains of Strep pyogenes produce erythrogenic exotoxins responsible for the characteristic rash of this disease. Fever, sore throat, and headache typically precede the rash, which usually spreads across the trunk, groin, and axillae. The rash has a characteristic "sandpaper" finely papular texture and is most pronounced in the skin hands. As the illness resolves, desquamation of the rash results in peeling of the hands and feet. An erythematous pharynx with tonsillar exudates, palatal petechiae, and a strawberry tongue are also classic findings.

acute rheumatic fever

An immune-mediated consequence of untreated strep pharyngitis. Presents with carditis (friction rub, prolonged PR interval, diffuse ST elevations), chorea (continuous, rapid, irregular jerks), and elevated ESR. There is also erythema marginatum, subcutaneous nodules, and migratory arthritis.

Hyper-IgE (Job) Syndrome

An immunodeficiency with severely elevated IgE levels and peripheral eosinophilia. Recurrent "Cold" abscesses (Staph, H flu, Strep pneumo) are typically seen. Eczema, retained primary teeth, fractures, and post-infectious pneumatoceles are all common findings.

secondary polycythemia

An increase in RBCs relative to total blood volume caused by increased EPO production. Increased EPO production can either be appropriate (chronic hypoxemia from cyanotic congenital heart disease, pulm disease, or high altitiudes) or inappropriate (tumors of the kidney, hydronephrosis). Clinical features include *ruddy facial complexion* without hepatosplenomegaly. Labs show elevated Hgb and Hct but normal platelet and WBC counts. EPO levels are high as well.

IGF-1

An indirect test for growth hormone deficiency; random growth hormone levels should not be measured, as most growth hormone is released during sleep.

heroin withdrawal syndrome

An infant born at 35 weeks' gestation to a mother with no prenatal care is noted to be jittery and irritable, and is having difficulty feeding. You note coarse tremors on examination. The nurses report a high-pitched cry and note several episodes of diarrhea and emesis. What should you suspect?

Thrombocytopenia-absent radius (TAR) syndrome is diagnosed in the newborn who demonstrates profound thrombocytopenia, bilateral absence of radius, and abnormally shaped thumbs. Cardiac lesions include TOF and ASD. About 40% of patients die in the newborn period as a result of low-platelet-induced bleeding.

An infant in the neonatal intensive care unit (NICU) has shortened lower distal arms but with thumbs. Her platelet count is profoundly low. What is the diagnosis and associated congenital heart defect?

The infant described in the question has alkaptonuria (alcaptonuria is an alternate spelling), an autosomal recessive disorder caused by a deficiency of homogentisic acid oxidase. The diagnosis is made in infants when their urine turns dark brown or black on exposure to air because of the oxidation of homogentisic acid. Affected children are asymptomatic. In adults, ochronosis—the deposition of a bluish pigment in cartilage and fibrous tissue—develops; symptoms of arthritis may appear later. No specific treatment is available for patients who have alkaptonuria, although supplemental ascorbic acid may delay the onset of the disorder and reduce clinical symptoms.

An infant is brought to a hospital because her wet diapers turn black when they are exposed to air. Physical examination is normal. Urine is positive both for reducing substance and when tested with ferric chloride. What is the diagnosis?

*Primary pulmonary hypoplasia* due to oligohydramnios (Potter sequence) includes a dysmorphic child (widely spaced eyes, low set ears, broad nose, receding chin, limb abnormalities) and bilateral renal agenesis. These infants have immediate respiratory distress; the condition is not compatible with life.

An infant of uncertain but seemingly term dates is born via emergent cesarean section for nonreassuring heart tones; the obstetrician has noted little or no amniotic fluid. The infant is small, has abnormally shaped limbs, and an unusual facies. The child has immediate respiratory distress. A chest radiograph reveals a poorly developed chest with little lung tissue. What is the likely diagnosis?

TGA

An infant presents with central cyanosis, a quiet precordium, and a single S2 with no murmur. What do you suspect?

*Congestive heart failure* can result from a number of causes, including congenital heart disease, Kawasaki disease, metabolic cardiomyopathies, arrhythmias, and viral myocarditis. The case presented is typical of an infant with heart failure. Such infants are weak, diaphoretic, have poor weight gain, and may be tachypneic with retractions. Lung findings can include crackles, wheezes, or both. Prompt evaluation by a pediatric cardiologist is imperative.

An infant previously diagnosed with a large muscular VSD comes to the office with complaints from the mother of fatigue and poor feeding over the past month. You note the child has not gained weight since the previous visit 2 months ago. The child is apathetic, tachypneic, and has wheezes and crackles on lung auscultation.

Bacterial meningitis; *Strep pneumo and N meningitidis* are the most common causes of bacterial meningitis in children age >1 month. Confirm diagnosis with LP; follow LP with prompt administration of empiric antibiotics. Third-gen cephs (ceftriaxone, cefotaxime) are effective against most strains of Strep pneumo and N meningitidis. Vanc is also given due to resistant strains of Strep pneumo. Whenever possible, do LP before giving antibiotics so you don't sterilize the CSF.

An infant which presents with fever, vomiting, and nuchal rigidity (crying with neck flexion) raises concern for what disease?

Give *naloxone*, as the meperidine given to the mother is an opioid which has likely crossed the placenta.

An infant who appears to be of normal size is noted to be lethargic and somewhat limp on the warmer after birth. The mother is 28 years old, and this is her fourth delivery. The pregnancy was uncomplicated, with normal fetal monitoring prior to delivery. Labor was rapid, with local anesthesia and intravenous meperidine (Demerol) administered for maternal pain control. What should you do to rapidly improve the infant's condition?

intussusception

An infant with episodic crying, emesis, bloody sticky stool, and lethargy should raise concern for what?

*Galactosemia:* a metabolic disorder caused by galactose-1-phosphate uridyl transferase deficiency, leading to elevated blood levels of galactose. They are also at increased risk for E coli neonatal sepsis. Early diagnosis and treatment by elimination of galactose from the diet are mandatory.

An infant with failure to thrive, bilateral cataracts, jaundice, vomiting, hepatomegaly, convulsions, and hypoglycemia may be suffering from what?

herpangina

An infection by *coxsackie virus* that typically affects young children in the late summer or early fall. Patients have *grey vesicles* that progress to fibrin-coated *ulcers*. The oral enanthem is located in the *oropharynx* on the posterior soft palate, anterior palatine pillars, tonsils, and uvula. There is no associated rash.

measles (rubeola)

An infection by an RNA virus of the paramyxoviridae family. Highly infectious. Clinical features include a transient enanthem (rash on mucous membrane) of *Koplick spots* (small gray papules on the buccal mucosa) and exanthem (rash on skin) that begins around the neck and ears and spreads downward. *Cough, conjunctivitis, and coryza* are classic symptoms.

preseptal cellulitis

An infection located anterior to the orbital septum that most commonly results from a break in the periorbital skin (eg abrasion, insect bite) which allows cutaneous flora like Staph aureus or Strep pyogenes to enter the preseptal space.

otitis media

An infection of the middle ear caused by respiratory bugs. Characterized by unilateral ear pain (seen as child tugging on ear, since there is relief of pain with pulling pinna). Diagnosis confirmed by pneumatic insufflation (puff of air revealinga tense immobile membrane) and visualization of a moderately bulging tympanic membrane.

otitis media

An infection of the middle ear fluid common in young children. Most commonly affects children ages 6-18 months and those around age 5 (age of school initiation). It often follows an upper respiratory infection, as nasal congestion contributes to Eustachian tube inflammation. The cardinal findings are decreased mobility on pneumatic insufflation and bulging of the tympanic membrane. Fever, hearing loss, and tympanic membrane erythema are all common findings.

sinusitis

An infection of the nose and sinuses which presents with purulent nasal discharge and sinus tenderness. Radiographs will show air-fluid levels and opacification, but don't get imaging, as it is expensive and unecessary. If it has lasted over 10 days, presume bacterial infection. Patients with severe symptoms (high fever, facial pain for 3-4 days, acutely worsening symptoms), initiate therapy with amoxicillin-clavulonate. Don't use azithromycin!

infectious mononucleosis

An infectious disease caused by EBV, transmitted primarily through saliva and infecting B lymphocytes. Presents with fever, malaise, fatigue, pharyngitis, posterior cervical lymphadenopathy, hepatosplenomegaly, macular rash. Symptoms resolve in weeks to months. CBC demonstrates atypical lymphocytes.

malaria

An infectious disease endemic to the tropics and subtropics. It is trasmitted through the bite of female Anopheles mosquitos. Clinical features include cyclical fevers which correlate to RBC rupture and subsequent parasitemia. Chills, vomiting, headache, and abdominal pain may also occur. Diagnosed by identifying the parasite on peripheral blood smear.

osteomyelitis

An infectious disease most commonly caused by Staph aureus spreading hematogenously or directly inoculating bone. Presents with bone pain and swelling. X-ray may be falsely negative early, so MRI is best radiographic test, and bone biopsy is best for organism identification. If toxic, treat before biopsy, and if nontoxic, get biopsy before starting antibiotics.

hand foot mouth disease

An infectious rash caused by coxsackie A virus. Has a vague prodrome but may present with oral pain. There are vesicles on an erythematous base primarily on the hands, feet, and mouth. Can also involve the buttocks.

erythema infectiosum ("slapped cheek")

An infectious rash caused by parvovirus B19. Usually has a nonspecific prodrome. Rash is isolated to the face bilaterally. There may, however, be some spread to the trunk or limbs, which will be lacy or reticular in appearance. In a healthy baby, it is benign and will resolve spontaneously. If children with sickle cell, anemia, or heavy metal intoxication, this may precipitate an aplastic crisis.

dermatomyositis

An inflammatory condition of muscle which results in progressive muscle weakness with characteristic skin findings. Affected individuals are usually 5-14 years of age, and females are more likely to be affected. Presents with consititutional symptoms, heliotrope rash, Gottron's papules, and proximal muscle weakness, mostly of the hip girdle and legs. Treat with corticosteroids.

bronchiolitis

An inflammatory disorder of the small airways caused by viral infection, usually *RSV*, in children under 2. It doesn't respond to beta agonists like asthma does, because it is caused by partial airway obstruction from sloughed epithelial cells, not bronchospasm. Most studies will be normal and CXR may show inflammation of bronchioles and hyperinflation. Diagnose with rapid antigen testing from a nasopharyngeal swab. However, the main priority is not diagnosing but deciding whether to hospitalize. Any child with an *SpO2 under 90, prematurity, age under 3 months, cardiopulm abnormality, or immunodeficiency* should be hospitalized. In the hospital, put on isolation, give O2, and IV fluids/feeds if necessary.

Fanconi anemia

An inherited DNA repair defect which results in bone marrow failure. Patients will present with signs of pancytopenia, short stature, hypo- or hyperpigmented macules, abnormal hypoplastic thumbs, and genitourinary malformations. Lab findings will include pancytopenia, positive chromosomal breakage testing; treatment is through hematopoietic stem cell transplant.

Klumpke's palsy

An injury of the lower brachial plexus (c7 and C8 nerve roots). Caused by upward traction on the arm. Clinical features include claw hand owing to unopposed finger flexion and decreased ability to extend the elbow and flex the wrist. Horner's syndrome may also be present if the sympathetic fibers of the first thoracic nerve have also been damaged.

stridor

An inspiratory sound of the upper airway which indicates a partial obstruction somewhere near the trachea or larynx. There are many causes, most commonly croup, epiglottitis, bacterial tracheitis, retropharyngeal abscess, and peritonsillar abscess. Inspect visually and do a CXR to get the diagnosis.

femoral anteversion

An inward angulation of the femur; the most common cause of in-toeing in children older than 2. Feet and patella point medially; hips are able to internally rotate more than normal. Child prefers to sit in a W position. Managed with observation only; prognosis is excellent.

A child who has a subacute disorder of the CNS that produces cranial nerve abnormalities (especially of cranial nerve VII and the lower bulbar nerves), long-tract signs, unsteady gait secondary to spasticity, and some behavioral changes is most likely to have a *pontine glioma.* These diffuse tumors are difficult to treat.

An irritable 6-year-old child has a somewhat unsteady but nonspecific gait. Physical examination reveals a very mild left facial weakness, brisk stretch reflexes in all four extremities, bilateral extensor plantar responses (Babinski reflex), and mild hypertonicity of the left upper and lower extremities; there is no muscular weakness. What is the most likely diagnosis?

septic arthritis of the hip

An orthopedic emergency characterized by bacterial infection of the hip. Results in a limp. Staph aureus and strep pyogenes are the most common organisms. N. gonorrhoeae may cause it in teenagers. Characterized by fever, irritability, limp, pain with movement of joint. Hip is held in flexion, abduction, and external rotation. Erythema, swelling, and asymmetry may be present. WBCs, ESR, and CRP will be elevated. Blood culture is positive in 30-50% of cases.

slipped capital femoral epiphysis

An orthopedic emergency which occurs in adolescents who are either obese or in a growth spurt. They'll complain of hip or knee pain, sudden onset. Get a frog-leg position x-ray to confirm; treat with surgery.

The patient in this question, especially with the pulmonary findings, may have Hodgkin disease. In underdeveloped countries, the peak incidence of Hodgkin disease is in children under 10 years of age; however, in developed countries, the peak incidence occurs in late adolescence and young adulthood. Systemic symptoms of Hodgkin disease include fever, night sweats, malaise, weight loss, and pruritus. Although a biopsy of the node may prove to be necessary at some point, the first step would involve a CXR, which may show mediastinal mass suspicious for Hodgkin disease. Depending on the results of the radiograph, a biopsy of a node may be indicated, especially if the question of Hodgkin (or other malignancy) remained high.

An otherwise healthy 17-year-old boy complains of swollen glands in his neck and groin for the past 6 months and an increasing cough over the previous 2 weeks. He also reports some fevers, especially at night, and possibly some weight loss. On examination, you notice that he has nontender cervical, supraclavicular, axillary, and inguinal nodes, no hepatosplenomegaly, and otherwise looks to be fairly healthy. What should you do?

Beckwith Wiedemann syndrome

An overgrowth disorder characterized by a predisposition to neoplasms. Most patients have an alteration of chromosome 11p15, which includes genes that encode insulin-like growth factor 2. At birth, findings include macrosomia, macroglossia, hemihyperplasia, and medial abdominal wall defects like umbilical hernia or omphalocele. Newborns must be monitored closely for hypoglycemia, but this problem is usually transient and older asymptoamtic patients usually do not require ongoing glucose monitoring. However, they are at increased risk of Wilms tumor and hepatoblastoma and screening abodminal ultrasound and AFP levels should be obtained every 3 months from birth to age 4 years. Continue to get an abodminal ultrasound every 3 months from age 4-8 years, and after 8 years get a renal ultrasound.

*Epiglottitis:* although the incidence of epiglottitis has been drastically reduced due to widespread Hib vaccination, Hib remains the most common cause. Acute management focuses on intubation and antibiotics.

An uncommon but potentially fatal infection that presents with acute onset fever with dysphagia, drooling, and respiratory distress. Patients may hyperextend the neck and maintain a "tripod position" to maximize airway diameter when significant airway swelling is present.

membranous nephropathy

An uncommon cause of nephrotic syndrome in children which occurs in those infected by *hepatitis B.* It will present with edema, hypoalbuminemia, and elevated urine protein

rumination

An uncommon disorder in which patients will regurgitate, rechew, and reswallow food. Initially described in infants with mental retardation and institutionalized patients, it may also be present in otherwise normal children who have problem relationships with their caregivers. One hypothesis suggests the regurgitation behavior begins as a self-stimulus, with infants putting their hands in their mouth and gagging themselves. The behavior is reinforced with the attention they receive for the regurgitation. It may also be seen in adolescents, and may be misdiagnosed as bulimia. While some studies have suggested an association, bulimic patients do not typically reswallow their food. Current information suggests that neither antireflux nor psychiatric medications have any beneficial effects on these patients. Instead, treatment must be focused on removing the attention paid by the caregivers to the behavior, and reinforcing proper eating behaviors.

Erb's palsy

An upper brachial plexus injury involving the C5 and C6 nerve roots. The most common brachial plexus injury, most often involving the right arm. Clinical features include a *flaccid arm* and *asymmetric Moro reflex.* The arm is held in internal rotation with the elbwo extended, forearm pronated, and the wrist and fingers held in flexion (the "waiter's tip" position).

X-linked (Bruton's) Agammaglobulinemia (XLA)

An x-linked disorder (seen only in males) of B-cell deficiency. Presents with recurrent sinusitis, otitis, and pneumonia. All Ig levels will be deficient. Flow cytometry will show absence of B cells, and you can confirm this with genetic testing for the BTK gene. Patients will need scheduled IVIG.

*Hemophilia A:* treatment includes prevention of trauma and replacement of factor VIII. Desmopressin acetate (DDAVP) may cause release of stored factor VIII from patient's own cells and may be useful in mild hemophilia.

An x-linked disorder seen often in Caucasian males. Presents with hemarthroses (involving the knees, elbows, and ankles most commonly) and deep soft-tissue bleeding. Severe forms run the risk of spontaneous bleeding, while moderate forms have bleeding only with trauma.

Menkes kinky-hair disease

An x-linked recessive disorder caused by abnormal copper transport. Affected patients have low serum copper, and clinical features develop in the first few months of life and include myoclonic seizures, pale friable hair, optic nerve atrophy, severe mental retardation, progressive neurologic degeneraiton, and early death. Typical hair findings, low serum ceruloplasmin, and low serum copper make the diagnosis.

renal dysgenesis oligohydramnios

ACE inhibitors will cause what deformity in the fetus if used by a pregnant woman?

*Meconium aspiration* is the likely diagnosis here. Infants who are postdates (more than 42 weeks' gestation) and show evidence of chronic placental insufficiency (low birth weight for gestational age and wasted appearance) have a higher than average chance of being asphyxiated, and passage of meconium into the amniotic fluid places these infants at risk for meconium aspiration. Intubation and suction of the trachea as well as supplemental O2, are indicated.

At 43 weeks' gestation, a long, thin infant is delivered. The infant is apneic, limp, pale, bradycardic, and covered with "pea soup" amniotic fluid. What is the diagnosis and management?

This patient has a physiologic discharge related to estrogen withdrawal. Examination can exclude vaginal trauma, and the history makes sexual abuse unlikely. Imaging is not indicated at this point. Parents should be reassured that it will resolve in a few weeks.

At the 2-week check-up of a term female infant, the mother reports a grayish and sometimes bloody vaginal discharge since birth. The infant's mother and grandmother are the only caretakers. Examination of the external genitalia reveals an intact hymen with a thin grayish mucous discharge. What is the likely diagnosis?

between 3-6 months

At what age do the primitive reflexes disappear?

4-12 years

At what ages are growing pains common?

Moro: 3 months Rooting: 4 months Palmar: 6 months Babinski: 1 year

At what ages does each primitive reflex disappear?

This is likely *clonidine poisoning* and should be treated by *naloxone*. Atropine can also be used.

Aunt Mary is helping her family move to a new apartment. During the confusion, 3-year-old Jimmy is noted to become lethargic. The contents of Aunt Mary's purse are strewn about on the floor. In the ER, the lethargic Jimmy is found to have miosis, bradycardia, and hypotension. He develops respiratory depression that progresses to apnea, and has to be intubated. What should you give Jimmy?

Legg-Calve-Perthe disease

Avascular necrosis of the hip in children around six. There will be insidious onsent of knee pain and an antalgic gait (spend less time of painful leg). Diagnose with x-ray, treat with cast.

sideroblastic anemia

Anemia characterized by ring sideroblasts in the bone marrow. They result from accumulation of iron in the mitochondria of RBC precursors. MAy be inherited or may be acquired as a result of drugs or toxins (isoniazid, alcohol, lead poisoning, chloramphenicol).

cryptorchidism

Another term for undescended testes. May be associated with inguinal hernia, GU malformations, hypospadias, and genetic syndromes. In most males with this condition, the testes descend spontaneously before 12 months of age. Testes that do not descend by this age are predisposed to future malignancy.

persistent pulmonary hypertension of the newborn

Any condition associated with low blood flow to the lungs after birth. Perinatal asphyxia is one of the most common etiologies. Increased PVR leads to right-to-left shunt through the foramen ovale, with resultant hypoxemia. Clinical features range from cyanosis to respiratory failure. Prevention of hypoxemia is the cornerstone of therapy (oxygen); mechanical ventilation must be started early if O2 alone is not enough. In severe cases, ECMO is needed.

primary apnea

Apnea in the first minutes of life from any perinatal insult, which responds quickly to stimulation (rubbing with towel under back). The mouth then nose should be suctioned of secretions.

Secondary apnea

Apnea in the first minutes of life which presents with gagging respirations following primary apnea and then a second period of apnea. Bagging (via mask or tube) is needed to overcome this; stimulation will not work. IF pulse is less than 100 or there are respiratory difficulties after primary apnea, PPV is indicated.

thought to be secondary to an incompletely developed respiratory center

Apneic episodes, characterized by an absence of respirations for more than 20 seconds with bradycardia and cyanosis, are thought to be caused by what in otherwise well premature infants?

Initiated by the SRY gene located on the short arm of the Y chromosome; by week 9 of gestation, the SRY gene differentiates the gonads into fetal testes, which produce testosterone. Testosterone made by Leydig cells in the testes stimulates development of the Wolffian ducts, and anti-mullerian hormone made by fetal Sertoli cells inhibits development of mullerian structures; this is how the internal male genitalia is formed. The conversion of testosterone to DHT by 5alpha-reductase is responsible for penis enlargement, scrotal fusion, and masculinization of the external genitalia.

By default, the genitals are female; how does male sexual differentiation occur?

SLE

Children born to mothers with this disease are at increased risk for congenital third degree AV block

HIV

Children can get this disease vertically transmitted from mother, or horizontally transmitted from sex and needle drugs (the later is teenagers). Diagnose by *DNA PCR* if under 18 months since maternal antibodies will give false positives, or *ELISA* if over 18 months. All patients should receive HAART regardless of CD4 count, and prophylaxis cutoffs are the same as in adults once 6 years of age.

MMR (vaccine strain measles)

Children may experience fever and a maculopapular rash a week after receiving this vaccine, due to infection with vaccine-strain live-attenuated pathogens which cause a mild form of disease. No treatment is needed as the vaccine-strain illness is minor and self-limiting. However, patients with vaccine strain illness should avoid contact with immunocompromised individuals until the rash has resolved, since it is still transmissible to others.

PCP

Children of HIV positive mtohers should be given prophylactic treatment for what infection starting at 6 weeks of age?

*Intussusception,* because there is localized bowel wall inflammation that can act as a lead point for the intestines to telescope into the adjacent bowel.

Children with Henoch Schonlein purpura are at risk for what abdominal complication?

*Intussusception* due to intestinal edema and bleeding acting as a lead point for intestinal telescoping

Children with Henoch-Schonlein purpura are at risk of what complication?

splenic rupture

Children with infectious mono with splenomegaly should be restricted from ocntact sports until the spleen has returned to normal size due to risk of what complication?

ARPKD

Children with this disease often have a maternal history of oligohydramnios leading to pulmonary hpyoplasia. Kidneys are greatly enlarged and cystic. Severe HTN and liver involvement is common. Ultimately all patients require renal transplant.

bacterial sinusitis

Children with this disease present with persistent nasal drainage, congestion, and cough; symptoms last 10-30 days without improvement. There must be ill appearance, high fever (over 102.2 F) and purulent nasal drainage for at least 3 days. The most common predisposing factor is a preceding viral URI.

*McCune-Albright syndrome:* due to mutation in the GNAS gene which results in constant G protein activation and overproduction of pituitary hormones. Can also lead to thyrotoxicosis (TSH), acromegaly (GH), and Cushing syndrome (ACTH).

Children with this disease will present with precocious puberty, recurrent fractures due to polyostotic fibrous dysplasia, and irregular cafe au lait macules confined to one side of the body.

DKA

Children with this problem will present with hyperglycemia and anion gap metabolic acidosis. It is often preceded by infection. Presenting symptoms include polyuria, polydipsia, abdominal pain, and fatigue; on exam, patients are often dehydrated and breathing rapidly.

selective mutism

Children with this problem will refuse to speak in specific social situations, such as school, but will engage normally in other situations, such as at home. IT is considered an anxiety disorder and is frequently comorbid with social anxiety disorder. Treatment includes CBT, family therapy, and SSRIs.

absence seizure

Children with this seizure disorder may be diagnosed with ADHD. There is no post-ictal period. Child will see skip-phase conversations, with teacher jumping from topic to topic from their perspective. Confirm with EEG and treat with ethosuximide. Kids usually outgrow these.

common migraine

Bifrontal headache without an aura or diffuse throbbing headache of only a few hours' duration. As with adults, the headaches can be terminated with vomiting or sleep. Abortive medications such as appropriate doses of ibuprofen taken less than 30 minutes after the onset of symptoms can be highly effective. Family history is frequently positive.

*Necrotizing enterocolitis:*pathogenesis involves inflammation and necrosis of intestinal mucosa with invasion of gas-producing bacteria. Premature and SGA babies are particularly vulnerable due to decreased bowel motility, increased intestinal permeability, and immature host defenses.

Bilious emesis, hypothermia, abdominal distention, leukocytosis, and metabolic acidosis in a newborn should raise suspicion for what?

malrotation of the gut with acute volvulus

Bilious vomiting and sudden onset of abdominal pain in an otherwise healthy infant is the classic presentation for what disease?

*basal ganglia* hippocampus some brainstem nuclei

Bilirubin most frequently localizes to what areas of the brain as kernicterus?

hyphema

Blood within the anterior chamber, most commonly caused by blunt trauma. Causes impaired vision. A blood-aqueous fluid level may be seen as blood settles.

Gray-Turner sign

Bluish discoloration of the flanks which can occur in acute pancreatitis; caused by blood tracking along fascial planes.

Cullen sign

Bluish discoloration of the periumbilical area which can occur in acute pancreatitis; caused by blood tracking along fascial planes.

orbital floor fracture ("blow-out fracture")

Blunt trauma to the eye can fracture the orbital floor, which is thin and easy to fracture, resulting in this lesion. Orbital fat and the inferior rectus can become entrapped, leading to diplopia, strabismus, and backward displacement of the globe into the orbit. Numbness of the cheek and upper teeth can occur. Give oral antibiotics to prevent contamination. Surgical repair is indicated normally.

tet spells

Episodes in infants with Tet of Fallot characterized by sudden cyanosis and decreased murmur intensity. The trigger may be any maneuver that decreases arterial oxygen saturation, including crying, exercise, tachycardia, vasodilation, or volume depletion. The child typically becomes irritable and cries, resulting in a cycle of increasing cyanosis. Alterations in consciousness and hyperpnea may occur as a result of severe hypoxia and acidosis.

brain development

Essential fatty acids play an important role in what process in infants?

Fever (+1) Lymphadenopathy (+1) Tonsillar exudates (+1) Absent cough (+1) Less than 14 (+1) Over 44 (-1) 0-1: no further testing 2-3: perform rapid testing 4-5: empiric antibiotics

Define the Centor criteria for diagnosing Strep pharyngitis

*Mild:* <60 seconds, no or improving headache, no amnesia; send home with no CT. *Severe:* >60 seconds, headache present or worsening, amnesia present; get a CT and observe in house.

Define the difference between mild and severe consussion, recognizing that in real life this is more of a spectrum but on exams it will be one or the other.

1st: epidermis only, pain + erythema 2nd: into dermis, pain, erythema, blisters 3rd: through dermis, white + painless

Define the different degrees of burns

transient tachypnea of the newborn (TTN)

Delayed clearance of fetal lung fluid, mainly seen in c-sections. Presents with a term infant that has grunting and tachypnea with a hyperexpanded and "wet" appearing CXR. Treat with supplemental O2 or CPAP intubation if needed.

42 weeks or more from the first day of the LMP

Delivery which occurs during what time period is considered post-term

less than 37 weeks from first day of LMP

Delivery which occurs in what time period is considered premature?

*Primary AIHA* is generally idiopathic, with no underlying cause identified. Viral infections may be causal. *Secondary AIHA* is associated with underlying diseases like lymphoma, SLE, or immunodeficiency.

Describe the difference between primary and secondary autoimmune hemolytic anemia

1) preadolescent 2) elevation of breast and nipple as small projections 3) enlargement of breast; no separation of areola and breast; areola enalrges 4) areola and nipple project to form secondary mound above level of breast 5) only nipple projects; areola usually recedes to contour of breast

Describe the five Tanner stages of female breasts

1) preadolescent, no pubic hair 2) sparse, long, downy hair along labia 3) darker, coarser, curlier hair 4) coarse, curly, adult-type hair covering symphysis pubis 5) adult-type hair spread onto medial surface of thighs

Describe the five Tanner stages of female genitalia

1) preadolescent, no pubic hair, prepubertal testes. 2) testes larger; sparse, long, downy pubic hair. 3) testes further enlarged; penis length enlarges; darker, coarser, curlier hair. 4) darkening of scrotal skin; penis length and width increase; glans develops; coarse and curly pubic hair extending over symphysis pubis. 5) testes and penis adult in size and shape; adult-type pubic hair that spreads to medial surface of thighs.

Describe the five Tanner stages of male genitalia

There is a loss of the normal charge and size selective glomerular barrier to filtration of plasma proteins, leading to excessive urinary protein losses leading to hypoporteinemia. Reduced plasma oncotic pressure idnuces increased hepatic production of plasma lipoproteins, leading to hypercholesterolemia.

Describe the pathophysiology of nephrotic syndrome

Early localized disease is the first stage, with erythema migrans and constitutional symptoms like fever, headache, myalgias, fatigue, arthralgias, and lymphadenopathy. Early disseminated disease is the second stage, with multiple erythema migrans lesions which are smaller than the initial lesion, facial nerve palsy, aseptic meningitis, and ecephalitis. Carditis is rare but usually presents as heart block or myocarditis. Late disease is characterized by arthritis primarily.

Describe the stages of Lyme disease

*Unconjugated:* lipid soluble, crosses BBB to cause kernicterus, can't be excreted in the urine *Conjugated:* water soluble, can't cross BBB so doesn't result in kernicterus, excreted in urine

Describe the two types of bilirubin

C diff

Diarrhea in patients with recent antibiotic use should make you think of infection with what organism?

caput succedaneum

Diffuse edema or swelling of the soft tissue of the scalp that crosses the cranial sutures and usually the midline. Usually seen in the newborn.

nasolacrimal duct obstruction

Failure of complete canalization of the lacrimal system that results in obstruction to tear outflow. Obsturction typically occurs distally, at Hasner's valve. Cause is unknown. Features include watery eye with increased tear lake, matted eyelashes, mucus in hte medial canthal area, and bilateral involvement. MAnage with observation. Nasolacrimal massage may help. NLD probing is curative and is performed in those between 6 and 12 months of age.

Klippel Feil syndrome

Failure of normal vertebral segmentation that results in relative fusion of the involved vertebrae. Usually occurs in the cervical spine but can occur elsewhere. Associated with congenital torticollis, GU anomalies, congenital heart disease, hearing loss, and Sprengel's deformity.

localized candidal infection (thrush) in the mouth, which will present as white patches on the buccal mucosa

Failure to rinse your mouth after administering inhaled corticosteroids can result in what problem?

jejunoileal atresia

GI obstruction caused by mesenteric vascular accident during fetal life. Bilious emesis and abdominal distention occur within the first few days of life. Abdominal XR will reveal air-fluid levels, and contrast studies will show the abnormality.

Obtain amniotic fluid by amniocentesis; a lecithin to sphingomyelin ratio (*L:S ratio*) greater than *2:1* and the presence of *phosphatidylglycerol* (a minor phospholipid in surfactant) are indicators of fetal lung maturity.

How do you determine fetal lung maturity?

number = age (year) X 3

How do you determine how many cubes a child should be able to stack?

Diagnosis is made by neutrophil function testing, either dihydrorhdamine 123 or nitroblue tetrazolium testing.

How do you diagnose CGD?

All infants born to HIV-infected mothers have transplacentally acquired maternal antibody that may persist 18-24 months. Thus, *HIV specific DNA PCR* should be performed at birth and monthly until 4 monhts of age to detect perinatally infected infants. NEgative HIV DNA PCR at 4 months is consistent with an infant who has not been infected.

How do you diagnose HIV in infants of HIV+ mothers?

*ASO titer* is positive in 90% of children after strep pharyngitits but only 50% of patients with impeteigo. *ABD titer* is reliably positive after resp or skin infections. *Renal biopsy* is indicated only in serious cases; it shows mesangial proliferation and increased mesangial matrix.

How do you diagnose PSGN?

abdominal ultrasound elective cholecystectomy

How do you diagnose and manage cholecystitis, respectively?

Usually the diagnosis is clinical as the patient should undergo an appendectomy before getting an ultrasound or CT to prevent perforation. However, abdominal ultrasound and CT often do aid in diagnosis.

How do you diagnose appendicits?

*PFTs* showing a decreased FEV1/FVC that shows significant improvement after bronchodilators. Alternatively, you can see a normal FEV1/FVC which is made worse by methacholine challenge.

How do you diagnose asthma?

Suspect it when you see elevated LFTs, hypergammaglobulinemia, and circulating autoantibodies. *Liver biopsy* is generally performed to evaluate for cirrhosis, to grade disease activity, and to exclude other diagnoses.

How do you diagnose autoimmune hepatitis?

Diagnosis must be rapid to affect outcome. Abdominal US, radionucleotide imaging, and liver biopsy are performed in rapid sequence to rule out other causes of cholestasis, and then *intraoperative cholangiogram with laparotomy* to examine the biliary tree cofnirms the diagnosis.

How do you diagnose biliary atresia?

*Serum IgA endomysial and serum tissue transglutaminase antibody testing* are both sensitive and specific screens for celiac disease, except in IgA deficient patients. A clinical response to removal of gluten from the diet, with weight gain and resolution of symptoms, helps confirm the diagnosis. Small bowel biopsy is the gold standard and will show short flat villi, deep crypts, and vacuolated epithelium with lymphocytes.

How do you diagnose celiac disease?

Diagnosis can be confirmed with eosin-5-maleimide binding and acidified glycerol lysis tests.

How do you diagnose hereditary spherocytosis, which presents with Coombs-negative hemolytic anemia, jaundice, and splenomegaly, as well as reticulocytosis, hyperbilirubinemia, spherocytosis, and family history of anemia?

*Monospot* is a first-line test in diagnosing EBV mono; it measure presence of heterophile antibody, which agglutinates sheep RBCs. In children under 4, EBV antibody titers are the preferred diagnostic method, since monospot is less sensitive in this population in whom antibodies do not reliably form. Test viral capsid antigen (VCA), early antigen (EA) and EBV nuclear antigen (EBNA).

How do you diagnose infectious mononucleosis?

hydrogen breath test

How do you diagnose lactose intolerance?

Imaging with a *bone scan or MRI* detects it a few days after symptoms onset. A *plain x ray is not a good initial study* as it will be normal early in the infection. After 10-14 days of infection, a plain x ray begins to reveal elevation of the periosteum. Labs will show elevated WBC, ESR, CRP.

How do you diagnose osteomyelitis?

Diagnosis of sickle cell disease is now usually made at birth through state newborn screening programs. Hemoglobin electrophoresis is a highly sensitive and specific test that demonstrates HgS and HgF in the newborn with sickle cell disease.

How do you diagnose sickle cell?

Random blood glucose above 200 mg/dL with polyuria, polydipsia, weight loss, or nocturia

How do you diagnose type 1 diabetes?

quantitative assay for vWf antigen and activity (*ristocetin cofactor assay*) are diagnostic.

How do you diagnose von Willebrand disease?

Treat with IVF, oxygen, and IV pain control. Find an infection if there is one and treat it. Crisis is over when patient is no longer in pain, sickles are gone, and bili/retic return to baseline.

How do you treat a sickle crisis, where ischemia leads to vaso-occlusive crisis and tremendous amounts of pain? Recall retic count and bili will be higher than the patient's baseline when this occurs, and sickled cells will be seen on smear.

1) *allergen avoidance* 2) *intranasal steroids* (most effective) 3) second-gen antihistamines 4) intranasal antihistamines

How do you treat allergic rhinitis?

Treatment includes avoidance of specific food allergens. Inhaled or systemic steroids have been helpful.

How do you treat eosinophilic esophagitis?

Permethrin 1% cream rinse (Nix) can be used. 1:1 vinegar to water rinse can be used for nit removal. You should also wash all clothing and bedding in very hot water and replace all commonly used brushes.

How do you treat head lice (Pediculosis capitis) in small children?

Hydrocephalus requires surgical placement of a *ventriculoperitoneal shunt* to divert the flow of CSF. Complications of ventriculoperitoneal shunts include shunt infection and shunt obstruction.

How do you treat hydrocephalus?

Treatment is generally reserved for symptomatic patients; beta blockers or CCBs reduce the LVOT obstruction and improve diastolic compliance. Surgical myomectomy, antiarrhythmic medications, and dual-chamber pacing may also be indicated. *Participation in competitive athletic sports should be prohibited.*

How do you treat hypertrophic cardiomyopathy?

oral ferrous sulfate

How do you treat iron deficiency anemia?

Abortive treatment includes sumatriptan; for prophylactic treatment, propanolol is the drug of choice.

How do you treat migraines?

Anticoagulate with LMWH or warfarin

How do you treat patients with SLE and thrombosis secondary to antiphospholipid antibodies

heparin FFP warfarin

How do you treat protein C deficiency?

Clear the fecal mass and use stool softeners for 3-6 months; also use behavioral modification.

How do you treat retentive encopresis, which is inappropriate passage of feces due to chronic constipation and overflow incontinence, where a radiograph demonstrates a dilated, stool-filled colon?

incision and drainage if fluctuance is present

How do you treat retropharyngeal abscesses?

First, you must eradicate GABHS infection; use *benzathine penicillin IM injection one dose* or penicillin orally for 10 days. Control inflammation with *NSAIDs* and, if patients has severe cardiac involvement, corticosteroids.

How do you treat rheumatic fever?

IV bolus of isotonic fluid (normal saline)

How do you treat severe dehydration in children?

IV anticonvulsants, such as a short-acting benzo like *lorazepam* or *diazepam* followed by a loading dose of eitehr *phenobarbital* or *phenytoin.*

How do you treat status epilepticus?

It depends on the grade. Grade I VUR is reflux of urine into an undilated ureter. Reflux into the ureter and collecting system without dilatation is called grade II. Grade III lesions have dilatation of the ureter and collecting system without blunting of the calyces. Grade IV lesions are characterized by blunting of the calyces, and grade V lesions demonstrate even more dilatation and tortuosity of the ureter. While somewhat controversial, low-grade lesions (grade I and grade II) usually are conservatively managed with close observation, daily low-dose antibiotics, and urinalyses and cultures every 3 to 4 months. Grade V lesions (and some grade IV lesions) require surgical reimplantation of a ureter if the findings persist. Lesions in between these two extremes are treatment dilemmas.

How do you treat vesicoureteral reflux?

Risk of testicular cancer is decreased post orchiopexy, but overall remains elevated compared to the general population.

How does orchiopexy, performed before age 1, affect the risk of testicular cancer in children with cryptorchidism?

It increases systemic vascular resistances, increases pulmonary blood flow, and improves hypoxemia.

How does placement of patients in a knee-chest position during a "tet" spell help improve symptoms?

It helps determine whether cyanosis is caused by cardiac or respiratory disease. In patients with cardiac disease, giving 100% O2 increases the PaO2 only slightly, and levels above 150 mm Hg are unusual. In infants with lung disease, the PaO2 usually increases considerably when 100% O2 is given, often reaching levels greater than 150 mm Hg.

How is the 100% oxygen test used to help evaluate neonates with cyanosis.

Differences include age of onset (ADEM is usually seen in < 10-year olds), the presence of systemic findings like fever and emesis, and the lack of progression in the lesions once identified characterizes ADEM.

How is the presentation of ADEM different than MS?

Exclusive breastfeeding until 6 months and then continuation of breastfeeding along with introduction of solid foods until infant is 1 year old.

How long does the AAP recommend breastfeeding for?

200 words by age 2; uses 2 word sentences

How many words should a child have by age 2?

1000 words by age 3; uses complex sentences and prepositions

How many words should a child have by age 3?

>10%

How much of a person's body surface area needs to be burned in order for them to be admitted to a burn ward and have fluids replaced?

hip ultrasound

How should investiage suspected developmental dysplasia of the hip (eg if you note asymmetric gluteal/thigh/inguinal creases or apparent leg length discrepancy)?

Preterm infants should receive routine immunizations according to *chronologic age* rather than age correct for gestation. In other words, being preterm does not affect the normal vaccination schedule.

How should you administer vaccines to preterm infants?

For patients still in diapers, straight catheterization of the urethra should be performed to obtain a sterile urine specimen. Clean catch specimens are unreliable in these patients. However, once the patient is out of diapers, a mid-stream clean-catch is appropriate testing.

How should you collect urine specimens from pediatric patients in whom you suspect UTI?

1) *oral penicillin VK*, or 2) single dose IM benzathine penicillin, or 3) erythromycin (if penicillin allergy)

How should you treat Strep pharyngitis?

Avoid electrolyte derangements and meds that block potassium channels; beta blockers with pacemaker placement can prevent cardiac arrest.

How should you treat a child with congenital prolonged QT interval?

Duonebs (albuterol and ipratropium) with IV steroids; then transition to orals. Treatment may require escalating therapy up to intubation. Hospital admission is determined based on need for supplemental O2 and peak flow prior to and after bronchodilator therapy in the ED.

How should you treat acute asthma exacerbations?

These children should receive postexposure prophylaxis with the *varicella vaccine.* At-risk patient who cannot get the vaccine should get the varicella Ig instead, but under normal cirucmstances, the vaccine is sufficient and Ig is not indicated.

How should you treat an asymptomatic, healthy patient over 1 year who has been exposed to varicella and has not received their up to date immunizations for varicella?

*Macrolides* are the antibiotic of choice for pertussis treatment and post-exposure prophylaxis; all close contacts should be given a macrolide regardless of age, immunization status, or symptoms.

How should you treat close contacts of those infected with Bordetella pertussis?

If meconium is present but the infant is vigorous (pulse over 100, good respiration and muscle tone) then *simple suctioning* of the *mouth then nose* is sufficient. If the infant has poor tone or resp effort or pulse under 100, endotracheal suctioning with passage and withdrawal of an endotracheal tube should be performed. If the infant develops a dangerously low pulse, the focus should shift to delivering ventilatory support.

How should you treat meconium passed in utero, which puts the infant at risk of *meconium aspiration syndrome*?

Give passive immunization with hep B immune globulin andf active immunization with HBV vaccine within 12 hours of birth.

How should you treat newborns of HBV positive mothers?

typical = amoxicillin atypical = azithromycin

How should you treat pneumonia caused by typical pathogens vs atypical?

Pubertal gynecomastia is seen in up to 2/3 of adolescent boys during mid to late puberty; no workup or treatment is necessary as it usually resolves within a few months to 2 years.

How should you work up gynecomastia in a male undergoing puberty?

Legg Calve Perthes disease

Idiopathic avascular necrosis of the femoral head. Children have a slightly painful limp with decreased internal rotation and abduction of the hip. Diagnosed with AP and frog leg lateral radiographs showing increased density in the affect femoral head or a crescentic subchondral fracture in the femoral head, termed the "crescent sign."

neonatal hepatitis

Idiopathic hepatic inflammation during the neonatal period. A diagnosis of exclusion and the most common cause of cholestasis in the newborn. Male predisposition. Symptoms range from transient jaundice to liver failure. Jaundice and hepatomegaly are the presenting symptoms in 50% of patients, in the first week of life. Disease is generally self limited, with full recovery during infancy in 70% of patients. Diagnosis is clinical and management is supportive with supplementation of fat soluble vitamins. TPN may be needed if growth remains problematic.

Legg-Calve-Perthes disease

Idiopathic osteonecrosis of the femoral head; classically affects boys age 3-12, with peak incidence at age 6. Patients have an antalgic gait (avoid weight bearing on affected side) and dull chronic lower extremity pain of insidious onset. The pain may affect the hip directly or be referred to the thigh or knee. Initial x-rays may be normal, so persistent symptoms should prompt repeat imaging, which may show slcerosis of the femoral head with flattening and fragmentation. As disease progresses, exam may reveals limited internal rotation and abduction of the hip.

Only consent from one parent is necessary to proceed with treating the minor, especially when the decision is clearly in the child's best interest.

If a child has divorced parents, but only one of the parents gives consent for the child to get vaccines, what should you do?

*Upper GI series* like a barium swallow should be performed; it is the fastest and most accurate method of diagnosing malrotation with midgut volvulus.

If a newborn presents wtih bilious vomiting and gasless abdomen suggestive of volvulus secondary to malrotation, what is the best way to definitively diagnose?

Vaccination with the *conjugated S pneumoniae vaccine* decreases incidence of invasive infections caused by S pneumo. Pneumococcal vaccination plus *twice daily penicillin prophylaxis* can prevent almost all cases of penumococcal spesis in patients with sickle cell anemia.

If a patient has sickle cell anemia (with a *high reticulocyte count, previous admissions for pain and hematuria, and AA race*), what prophylaxis must you give to prevent cases of all pneumococcal sepsis? Recall that sickle cell patients become functionally hyposplenic due to autoinfarction and are more susceptible to infection with encapsulated organisms (*S pneumoniae, H influenzae, N meningitidis*).

Immediately attempt *intraosseus access*, which requires less skill than central line placement, and is also safer and faster than central lines. It provides a cannula large enough to deliver fluids and meds rapidly and to get blood samples for lab testing. The most common site is the proximal tibia, but any large bone can be used.

If a patient needs fluid resuscitation and you can't get IV access, what should your next step be?

get a CXR to rule out pneumothorax

If a patient with apparent subcutaneous emphysema presents with severe coughing paroxysms, what should your next step be?

*Carotid artery dissection* can occur with injury to the internal carotid artery, which is located directly lateral and posterior to the tonsillar pillars. Dissection can lead to meningismus and features of ischemic stroke (hemiparesis, facial droop, aphasia). Confirm diagnosis of carotid artery dissection by CT or MR angiography.

If a pediatric patient has symptoms of hemiparesis and motor aphasia after injury to the posterior pharynx, what should you suspect?

A nonspecific vulvovaginitis is common in this age group, often caused by chemical irritants such as bubble baths or by poor hygiene. Mothers should be counseled to use only cotton underwear for young children, stop bubble baths (or at least splash fresh, clean water in the vaginal area at the end of a bath), and reemphasize wiping front to back after urination or bowel movements.

If a young child has nonspecific vaginal itching with no other risk factors or complaints (eg no indications of sexual abuse or foreign bodies in the vagina), what advice would you give the mother?

Confirm with *venous lead* measurements if a screening capillary lead level is >5 micrograms/dL

If an asymptomatic child has an elevated lead level on capillary (fingerstick) blood specimens, what is the proper next step?

get a brain MRI to evaluate for hypothalamic or pituitary tumor; if a CNS tumor is excluded, primary treatment for idiopathic central precocious puberty is GnRH agonist therapy

If someone presents with precocious puberty and elevated LH, what should your next step be?

right-to-left shunt from congenital heart defects (TAPVR, Tet of Fallot, truncus arteriosus, TGA, tricuspid atresia); this is called the *100% oxygen challenge test.*

If you give 100% oxygen to a patient with central cyanosis and it fails to raise the PaO2 significantly, what diagnosis does this suggest?

Give more IM epinephrine, since additional doses may be required for refractory symptoms.

If you give IM epinephrine for suspected anaphylaxis but the symptoms recur minutes later, what should your next step be?

Unlike in adults, all negative rapid strep test children must be confirmed to be infected with strep by *throat culture* and if that is positive, you can treat with amoxicillin or penicillin.

If you have a high Centor criteria (5) but rapid strep testing is negative, what should be your next step?

Get an echo to determine the location and size of the defect and rule out other defects. Most small VSDs close spontaneously and require no treatment.

If you here a harsh holosystolic murmur in an infant and suspect VSD, what should be your workup?

meningitis

In infants, this infectious disease presents with *bulging fontanelles* and extremes of behavior (irritability, coma). Often caused by Neisseria in this age group with additional symptoms of rash, fever. Rapidly progressive. Diagnose with CSF and blood cultures and give antibiotics if either grows something. Don't delay antibiotics to obtain an LP or imaging.

*Decrease:* maneuvers that increase preload or afterload (squatting, leg raise, hand grip) increase LV cavity size and decrease outflow obstruction, decreasing murmur intensity. *Increase:* maneuvers that decrease LV cavity size by decreasing preload (Valsalva, abrupt standing, amyl nitrite) cause worsening of LV outflow tract obstruction and increase murmur intensity.

In patients with HCM, what movements decrease and increase the intensity of the murmur, respectively?

Waterhouse-Friderichsen syndrome

In patients with meningococcemia, this disease is characterized by sudden vasomotor collapse and skin rash due to adrenal hemorrhage; occurs due to fulminant meningococcemia. The condition carries an almost 100% mortality.

endotracheal intubation mechanical ventilation

In patients with severe asthma who remain unresponsive to maximal medical therapy and have signs of impending resp arrests (persistent hypoxemia, worsening hypercarbia, resp acidosis), what should your next steps be?

sickle cell disease

In patients with this disease, constant low grade hemolysis results in chronic anemia (baseline hemoglobin between 7 and 9), increased baseline bilirubin, and increased baseline reticulocyte count. Expect to see a *low haptoglobin, elevated LDH, elevated bili, and elevated retic count.*

ulcerative colitis (rectum only) pancolitis (entire colon)

In this IBD, infrlammation is diffuse, limited to the mucosa, and localized to the colon. It begins in the rectum and extends proximally in a contiguous fashion. Can present with rectal bleeding, diarrhea, and abdominal pain in mild disease, tenesmus, systemic symptoms, fever, weight loss, and anemia in moderate disease, and hypoalbuminemia, leukocytosis, and increased stool frequency in severe disease. Extraintestinal manifestations are less common during childhood.

Blount disease

In this condition, genu varum (bowleggedness, which is normally a common finding in infants that resolves spontaneously by 2 years) persists past age 2. It is characterized by an abnormality in the medial aspect of the proximal tibial epiphysis. Radiographically there is a prominent step abnormality with beaking and calcification at the proximal tibial epiphysis. Aggressive treatment is essential, as the disease can be rapidly progressive and lead to permanent growth disturbances. Bracing can be effective up to the age of 3; later correction may require surgery.

Hyper-IgM Syndrome

In this defect of humoral immunity, there are low levels of IgA and IgG but a normal to high IgM. This is because class-switching doesn't occur. Treat with scheduled IVIG.

bacterial pneumonia

In this disease, fever, cough, and dyspnea typically occur without preceding URI symptoms. Physical exam may demonstrate rales, tachypnea, decreased breath sounds, and evidence of resp distress. Diagnosis suggested by WBC > 20,000 with a neutrophil predominance and lobar consolidation on CXR. Manage with abx and supportive care.

ataxia telangiectasia

In this disease, quantitative measurement of serum IG reveals IgE and IGA deficiency the majority of the time.

congenital torticollis

In this disorder, the head is tilted toward the affected side with the chin pointed away from the contracture. Decreased range of motion and stiffness are noted when stretching. Treatment includes stretching exercises to relieve the muscle contracture.

bacterial

In this form of conjunctivitis in an older child, discharge is purulent, itching is minimal, preauricular adenopathy is absent, and labs show PMNs and organisms on gram stain.

allergic conjunctivitis

In this form of conjunctivitis in an older child, discharge is watery or mucoid, itching is severe, and eosinophils are seen on conjunctival scraping

viral

In this form of conjunctivitis in older children, discharge is watery, itching is minimal, preauricular adenopathy is common, and no organisms are seen on gram stain.

chronic granulomatous disease

In this immunodeficiency, macrophages can eat but not kill catalase positive organisms. Associated with chronic skin or organ abscesses. Pathogens include Staph, Aspergillus, and Serratia. Confirm with negative nitro blue test that reveals absent respiratory burst. Organisms that produce their own H2O2 can be killed. Treat with prophylactic TMP/SMX and itraconazole. This will ultimately require bone marrow transplant for cure.

leukocyte adhesion deficiency

In this immunodeficiency, neutrophils can't adhere or get out of blood vessels. Thus, there is no pus despite a massive leukocytosis and high fever. Most common infection locations are pulm, GI, genital, skin. Delayed cord separation is an early sign often seen in vignettes. Get a bone marrow transplant.

GERD

In this pathologic state, there is inappropriate transient lower esophageal sphincter relaxation, allowing excessive gastric material to enter the esophagus and even the oropharynx. It results in inflammation over time.

Congenital anomalies Neonatal asphyxia Meconium aspiration syndrome

Incidence of what is higher in SGA babies?

hydrocephalus

Increased CSF pressure within the ventricles, resulting from either blockage of cSF blow, decreased CSF absorption, or increased CSF production

congenital glaucoma

Increased intraocular pressure soon after birth. Normal pressure in infants is 10-15 mmHg, so the definition of this disease is pressure exceeding 30 mmHg. Most cases are autosomal dominant. Features include tearing, photophobia, enlarged cornea, corneal clouding, and dull red reflex. Usually bilateral. Manage with surgery to open outflow channels.

benign transient proteinuria

Increased urinary protein excretion that shows up as proteinuria on labs. Not intrinsically pathologic and associated with vigorous exercise, fever, dehydration, and CHF.

Neonatal thyrotoxicosis resulting in *heart failure.* Other symptoms of neonatal thyrotoxicosis include tachycardia and tachypnea, irritability and hyperactivity, low birth weight with microcephaly, severe vomiting and diarrhea, thrombocytopenia, jaundice, and hepatosplenomegaly.

Infants born to mothers with hyperthyroidism are most at risk for what complication?

prolonged use of baby bottles that contain high concentrations of sugars, which promotes dental disease

Infants with extensive maxillary tooth decay, especially frontal, and posterior maxillary and mandibular tooth decay, likely have what etiology to blame?

seizure full fontanelle temporal lobe hemorrhage no intracranial calcifications

Infants with herpes encephalitis from vertical transmission usually present with what symptoms?

Congenital Torticollis

Infants with this abnormality will present with a neck mass, ipsilateral head tilt, and contralateral chin deviation; it is a postural deformity in the SCM is tight and contracted, likely due to intrauterine crowding. There is often positional plagiocephaly that occurs when the infant chronically lies down on its preferred head side; that is, there is flattening of the head on that side as well as anterior displacement of the ear and forehead.

*Rickets:* caused by vitamin D deficiency or calcium deficiency, with the former being more common. People with dark skin are at increased risk as the primary source of vitamin D is sunlight. Breast milk alone does not have adequate vitamin D, so infants who do not get fortified baby food or formula should get vitamin D supplementation of 400 IU daily to prevent rickets.

Infants with this disease present with costochondral joint hypertrophy, genu varum (femoral and tibial bowing), a large anterior fontanel, and craniotabes.

diskitis

Infection (usually by Staph aureus) or inflammation of the intervertebral disc. Begins with URI symptoms following by back pain with tenderness over the involved disc. Fever is sometimes present. Children refuse to flex the spine. ESR is elevated. Diagnosed clinically, though MRI and bone scan can confirm. Treat with bed rest; use antibiotics in patients thought to have infection.

pneumonia

Infection and inflammation of lung parenchyma

cysticercosis

Infection by Taenia solium, the pork tapeworm, through gthe fecal oral route. There are no symptoms until the tapeworm encysts in muscle or brain tissue. If encysting in brain, seizures are generally the presenting symptom. Diagnosed by head CT or MRI showing a solitary parenchymal cyst or single or multiple calcifications.

visceral larva migrans

Infection by Toxocara canis or cati can cause this disease. Mostly occurs due to ingestion of eggs in contaminated soil or dog fur; larvae are released from eggs and migrate through tissue. Causes fever, eosinophilia, leukocytosis, hepatomegaly. If migration to eyes occurs, retinal granulomas can happen. Treat with albendazole, steroids for ocular involvement.

toxoplasmosis

Infection by the intracellular parasite T. gondii. Transmission occurs through direct contact with cat feces, ingestion of undercooked meat contaminated with cysts, transplacental passage, or organ transplantation. Most patients are asymptomatic of have a mono-like illness with malaise, fever, sore throat, myalgias, and lymphadenopathy. Immunosuppression may trigger reactivation of the disease with more severe symptoms. It commonly presents with focal seizures in this setting.

amebiasis

Infection by the protozoan parasite Entamoeba histolytica via infestion of cysts in contaminated food or water. Symptoms range from mild colitis to severe dysentery with cramping abdominal pain, tenesmus, and diarrhea.

otitis externa (OE)

Infection of the external auditory canal, commonly by P aeruginosa, Staph aureus, or Candida albicans. Can also develop in a patient with a perforated tympanic membrane secondary to acute otitis media. Presents with pain, itching, and drainage from the ear; systemic symptoms are usually absent. Diagnosed based on physical exam finding of erythema and edema of the external auditory canal.

*Phechromocytoma:* you should measure urine VMA and HVA and serum metanephrine. In adults, the episodes of hypertension are more paroxysmal than in children, where the hypertension is more sustained. While it is an unusual diagnosis in children, pheochromocytoma must be considered in the evaluation of a patient with hypertension who has intermittent symptoms described. Pheochromocytoma can be associated with tuberous sclerosis, Sturge-Weber syndrome, ataxia-telangiectasia, and it can be inherited as an autosomal dominant trait.

During a routine well-child examination, a 12-year-old girl reports that she has occasional headache, "racing heart," abdominal pain, and dizziness. Her mother states that she has witnessed one of the episodes, which occurred during an outing at the mall, and reported the child to be pale and to have sweating as well. Other than some hypertension, she has a normal physical examination. What is the likely diagnosis?

A natal tooth is a fairly unusual finding, occuring in about one of every two to three thousand live births. The tooth is usually a primary mandibular incisor, but can be a supernumerary tooth as well. Although it may be uncomfortable for the mother to breast-feed an infant with a natal tooth, the primary concern is whether the tooth is loose, as an unstable natal tooth may fall out and be aspirated by the infant. *Extraction by a pediatric dentist* is the usual course when the tooth is loose; however, extraction can lead to abnormal spacing of the remaining teeth as they erupt.

During the initial examination of a term newborn infant, you notice a fully erupted mandibular incisor. It appears normal in shape and position, but is very loose. What should you do?

omphalocele

Congenital extrusion of the bowel in the midline, covered with a membrane. Commonly associated with chromosomal abnormalities like Beckwith-Wiedemann syndrome. To treat, cover viscera in a sterile bag and place saline-soaked gauze over extruded contents to prevent desiccation and infection. Place an NG tube to keep bowel decompressed. Often requires placement of a covered silo to allow extruded contents to gradually re-enter the abdomen.

gastroschisis

Congenital extrusion of the bowel to the right of midline, without a membrane. Susceptible to twisting and infection and "angry looking." To treat, cover viscera in a sterile bag and place saline-soaked gauze over extruded contents to prevent desiccation and infection. Place an NG tube to keep bowel decompressed. Often requires placement of a covered silo to allow extruded contents to gradually re-enter the abdomen.

vitamin C (ascorbic acid)

Excessive ingestion of this substance may result in false negative results for hematuria on urine dipstick, which detects the presence of hemoglobin or myoglobin in the urine.

vitamin D iron (if preterm)

Exclusively breastfed infants should be given what supplemental nutrients in the first months of life?

1) *duodenal atresia:* polyhydramnios in utero, bilious vomiting in neonate. CXR shows double bubble, no distal air. Link with Down syndrome. 2) *annular pancreas:* you'll see double bubble with distal air; the pancreas is squeezing the duodenum. Also linked with Down syndrome 3) *malrotation:* incorrect alignment of the intestines due to developmental failure. An upper GI series can show abrupt cutoffs in the GI tracts. 4) *intestinal atresa:* double bubble and multiple air fluid levels; often caused by maternal cocaine or tobacco use, linked to vascular accidents in utero

List four diagnoses on your differential with bilious vomiting

Growth hormone deficiency Hypothyroidism Hypercortisolism Turner syndrome

List four endocrinopathies that can cause short stature in pediatric patients

1) Persistent fisting beyond 3 months is often the earliest sign of neuromotor problems. 2) Early rolling over, pulling to a stand instead of sitting, and persistent toe walking may all indicate spasticity. 3) Spontaneous postures like scissoring indicate motor abnormalities. 4) Early hand dominance (before 18 months) may be a sign of weakness of the opposite upper extremity.

List four red flags in motor development

infections fava beans sulfa drugs salicylates antimalarials

List some oxidizing triggers of hemolysis in patients with G6PD deficiency

1) non weight bearing 2) ESR over 40 3) fever > 38 C 4) WBC > 12,000

List the *Kocher criteria*, recalling that there is increasing probability of septic joint with each positive finding. Confirm diagnosis of septic hip with x-ray and joint aspiration with Gram stain and culture; treat by draining and starting antibiotics.

1) colicky abdominal pain 2) GI bleeding with bloody stool 3) increased intussusception risk

List the GI manifestations of Henoch Schonlein purpura

Joints (migratory arthritis) O (carditis) Nodules (subcutaneous) Erythema marginatum Sydenham chorea

List the clinical features of acute rheumatic fever

*Maternal:* more rapid uterine involution and decreased postpartum bleeding; faster return to prepartum weight; improved child spacing; improved maternal-infant bond; reduced risk of breast and ovarian cancer. *Infant:* improved immunity (prevention of infectious diseases like otitis media, gastroenteritis, respiratory ilness, UTI), improved GI function, decreased risk of childhood cancer, type I diabetes, and necrotizing enterocolitis

List the maternal and infant benefits of breastfeeding

*Maternal:* active untreated Tb, HIV infection, herpetic breast lesions, varicella infection <5 days prior to delivery, chemotherapy, abuse of drugs or alcohol *Infant:* galactosemia

List the maternal and infant contraindications to breastfeeding

*Catarrhal:* 1-2 weeks, mild cough and rhinitis. *Paroxysmal:* 2-6 weeks, cough with inspiratory whoop, posttussive emesis. *Convalescent:* weeks to months, symptoms gradually resolve

List the three clinical phases of pertussis infection

*Noncommunicating hydrocephalus* refers to enlarged ventricles caused by obstruction of CSF flow through the ventricular system (eg aqueductal stenosis) *Communicating hydrocephalus* refers to enlarged ventricles as a result of increased CSF production (eg tumors) or decreased absorption of CSF (eg bacterial meningitis) *Hydrocephalus ex vacuo* is not true hydrocephalus, but rather a term used to describe ventricular enlargement caused by brain atrophy

List the three different types of hydrocephalus

*Catarrhal stage:* lasts 1-2 weeks. Characterized by URI symptoms (rhinorrhea, nasal congestion, conjunctival redness, low grade fever). *Paroxysmal stage:* lasts 2-4 weeks. Characterized by fits of forceful coughing, post-tussive vomiting, cyanosis, apnea, and choking. Between fits, children appear well and are afebrile. *Convalescent phase:* lasts weeks to months. A recovery stage in which the paroxysmal cough continues but becomes less frequent and less severe over time.

List the three stages of whooping cough

*Chemical:* occurs within 24 hours. Bilateral. Caused by silver nitrate; stop giving silver nitrate. *Gonorrhea:* occurs in days 2-7. Purulent and bilateral. Check for systemic illness. Treat with ceftriaxone IM. *Chlamydia:* occurs on day 5-14. Usually watery then purulent and bloody, unilateral then bilateral. Can turn into pneumonia. Treat with erythromycin PO. Don't use topical abx.

List three common causes of conjunctivitis in the newborn

*Classic salt-wasting CAH:* both mineralocorticoid and glucocorticoid paths are affected, resulting in both cortisol and aldosterone deficiency. Girls present with ambiguous genitalia, failure to thrive, vomiting, and electrolyte abnormalities. *Simple virilizing CAH:* only the glucocorticoid path in affected, resulting in cortisol deficiency; girls present with ambiguous genitalia, and boys present later in life with tall stature, advanced bone age, pubic hair, and penile enlargement (precocious puberty). *Nonclassic CAH:* late onset with very mild cortisol deficiency and no mineralocorticoid involvement. Girls present with premature adrenarche, clitoromegaly, acne, rapid growth, hirsutism, and infertility. Boys present with premature adrenarche, rapid growth, and premature acne.

List three different subtypes of congenital adrenal hyperplasia caused by *21 hydroxylase deficiency.*

*Ostium primum:* occurs in the lower portion of the atrial septum. A cleft in the anterior mitral valve leaflet may also be present and may cause mitral regurgitation. Common in Down syndrome. *Ostium secundum:* in the middle portion of the atrial septum. The most common type of ASD overall. *Sinus venosus:* defect high in the septum near the junction of the right atrium and SVC. The right pulmonary veins usually drain into the right atrium or SVC instead of into the left atrium as they do normally.

List three different types of atrial septal defects

cetirizine fexfenadine loratadine

List three second-gen H1 antihistamines

*Hypospadias:* describes the urethral meatus located on the ventral surface of the penis, along the shaft. Not associated with an increased incidence of associated urinary malformations. *Epispadias:* describes the urethral meatus located on the dorsal surface of the penis; often associated with bladder extrophy.

List two abnormalities of the location of the urethral meatus found in newborns

primary dysmenorrhea

Lower abdominal cramping with menses in the absence of other pathology. A common complaint in up to 90% of young women. Onset is generally during adolescence and symptoms may improve with age. Cramping during the first few days of menses is caused by uterine contractions triggered by prostaglandin release from the sloughing endometrium. The pain diminishes as the lining of the endometrium thins. There will be a normal physical exam; NSAIDs are first-line treatment, then prostaglandin synthetase inhibitors which decrease prostaglandin production.

*Airborne:* the most effective way to prevent spread in the hospital setting is through airborne precautions.

Measles virus is highly contagious through what route?

congenital syphilis

Most untreated infants with this disease will develop skin lesions, typically an infiltrative, maculopapular peeling rash that is most prominent on the face, palms, and soles. Involvement of the nasal mucous membranes causes rhinitis with a resultant serous, and occasionally purulent, blood-tinged discharge (snuffles). This, as well as scrapings from the skin lesions, contains abundant viable treponemes. Hepatosplenomegaly and lymphadenopathy are common, and early jaundice is a manifestation. Liver function tests are elevated; hemolytic anemia and thrombocytopenia are common. Infants may have a saddle nose

folic acid

Mothers should take what supplement prenatally to prevent spina bifida?

fibrillin-1

Mutation of what gene results in *Marfan syndrome*, which presents with tall stature, long thin extremities, arachnodactyly, joint hypermobility, upward lens dislocation, and aortic root dilation?

*Emergency phase:* the goal is to restore intravascular volume to ensure perfusion of vital organs. At is the same for all patients regardless of initial sodium levels: 20 mL/kg boluses of IV NS or LR. *Repletion phase:* the goal is more gradual correction of patient's water and electrolyte deficits. For hyponatremic or isonatremic dehydration, replete over 24 hours; hypernatremic dehydration should be more slow, over 48 hours, to minimize risk of cerebral edema.

Name the two phases of parenteral rehydration

1) *Prespeech:* 0-10 months. Expressive language is cooing and babblings. Receptive language characterized by increasing ability to localize sounds. 1) *Naming period:* 10-18 months. Characterized by infant's understanding that people have names and objects have labels. 3) *Word combination period:* 18-24 months. Typically, children begin to combine words 6-8 months after their first word.

Name three broad stages of speech development in an infant

1) Foramen magnum stenosis may lead to *hydrocephalus* or *cord compression*; head circumference must be monitored closely, because sudden infant death can occur as a consequence. 2) *OSA* and respiratory compromise may also occur from foramen magnum narrowing and upper airway obstruction. 3) Orthopedic problems like *genu varum* and back pain.

Name three complications of achondroplasia

bacteremia PSGN SSSS

Name three possible complications of impetigo

*Lymphoblastic lymphoma:* T-cell origin, histologically similar to lymphoblast of ALL. Commonly presents with anterior mediastinal mass, and patient may develop SVC syndrome or airway obstruction as a result. *Burkitt's lymphoma:* the most common lymphoma of childhood. B-cell origin. In Africa, where it is endemic, it presents with jaw masses. *Large cell lymphoma:* B cell origin. Commonly presents as enlargement of lymphoid tissue in the tonsils, adenoids, or Peyer's patches.

Name three subtypes of NHL

1) *spastic diplegia:* involves the lower extremities more than the upper extremities or face 2) *spastic hemiplegia:* characterized by unilateral spastic motor weakness 3) *spastic quadriplegia:* characterized by motor involvement of head, neck, and all limbs.

Name three subtypes of spastic cerebral palsy, where there is increased tone.

*Hematochezia* is bright red blood passed per rectum, usually idnicating a lower GI source or significant rapid bleed from an upper lesion. *Melena* is dark tarry stool and often indicates an upper GI bleed.

Name two different appearances of blood in the stool and their interpretation

*Marasmus:* the most common energy depletion state, characterized by near starvation from both protein and nonprotein deficiencies. the patient is typically very thin from loss of muscle and body fat. *Kwashiorkor:* less common, seen in parts of the world in which starches are the main dietary staple. A protein deficient state characterized by generalized edema, abdominal distention, changes in skin pigmentation, and thin sparse hair.

Name two different states of malnutrition

EBV (Burkitt's lymphoma) HIV (Kaposi's sarcoma)

Name two infectious organisms associated with childhood cancers

aseptic meningitis orchitis

Name two serious complications of mumps

coarctation of the aorta

Narrowing of the aortic arch, just below the origin of the left subclavian artery and typically at or just proximal to the ductus arteriosus. It may be a discrete narrowing (hourglass) or a long-segment obstruction. The narrowed segment obstructs flow from the proximal to the distal aorta. Blood pressure may be elevated in the upper extremities and low in the lower extremities.

pulmonary stenosis

Narrowing of the pulmonary valve which results in increased right ventricular pressure and reduced right ventricular output. May present in the neonate as cyanosis due to right-to-left shunting if severe, but for mild to moderate flow obstruction, symptoms are absent. Manage with balloon valvuloplasty for symptomatic infants.

congenital heart block

Neonates born to mothers with SLE might have what cardiac complication secondary to transplacental passage of maternal antibodies?

aortic stenosis

Neonates with this severe congenital cardiac defect can appear normal at birth but develop CHF symptoms at 12-24 hours as the PDA closes and all systemic flow must be ejected through the aortic valve, which is not possible. It can also present later in life with exercise intolerance, chest pain, syncope, and even sudden death.

pulmonic stenosis

Noonan syndrome is associated with what heart defect?

Howell-Jolly bodies

Nuclear remnants of RBCs that, in normal patients, are removed by the spleen. In patients with sickle cell disease, where the spleen is not functioning, they are expected findings on peripheral smear.

isoimmune thrombocytopenia

Occurs when a mother produces antibodies against her fetus' platelets as a result of sensitization to an antigen that her own platelets lack. The mother's platelet count is normal.

Tet of Fallot

On physical exam of a patient with a congenital heart defect, you note increased right ventricular impulse and a systolic ejection murmur representing pulmonary stenosis. The patient is cyanotic. What do you suspect?

dactylitis (hand-foot syndrome)

One of the earliest manifestations of vaso-occlusion in sickle cell disease. Occlusion of blood vessels supplying the metacarpals and metatarsals occurs secondary to polymerization of sickled hemoglobin; it presents in infatns 6 months to 4 years with acute onset of pain and symmetric swelling of hands and feet. Low grade fever is sometimes also present. Initial radiographs reveal only soft tissue swelling, although in recurrent episodes they can reveal a mottled appearance of bone. Treatment involves hydration and pain control, and then address the underlying SCD.

vesicoureteral reflux

One of the most common abnormalities which results in recurrent UTIs in infants and children. Patients with this abnormality have urinary reflux from the bladder into the kidney, causing dilation of the ureters and kidneys (hydronephrosis). Diagnosis is made by contrast voiding cystourethrogram.

UTIs

One of the most common bacterial infections in children ,especially in uncircumcised boys. Caused by bacteria ascending through the urethra.

intussusception

One of the most common causes of intestinal obstruction in children ages 6-36 months. The ileocolic junction is most frequently involved, with invagination of the ileum into the colon. Preceding viral infections (eg gastroenteritis) are thought to play a role in inflaming lymphatic tissue (eg Peyer patches) which can serve as lead points for the telescoping. Meckel's diverticulum, polyps, and hematomas are other common lead points. Classically, this will present in periodic pain (as telescoping is intermittent) associated with *drawing the legs up toward the abdomen.* Emesis may follow episodes of abdominal pain; it is typically nonbilious but becomes bilious as obstruction persists. Ongoing obstruction can cause mucosal ischemia leading to *Currant jelly stools.*

Mycoplasma pneumoniae

One of the most common causes of penumonia in older children and adolescents. Symptoms include low grade fever, chills, nonproductive cough, headache, pharyngitits, and malaise. The cough may last 3-4 weeks. Lung exam may demonstrate widespread rales, and exam findings are often worse than expected by history. Positive cold agglutinins are suggestive of the diagnosis but not specific; definitive diagnosis is by elevation of serum IgM titers specific for this organism. Treat with oral erythromycin.

radial head subluxation (nursemaid's elbow)

One of the most common elbow injuries in children 1-5 years old. It occurs when the child is lifted or pulled by the hand or arm, causing axial traction on the forearm, which in turn causes the radial head to slip through parts of the annular ligament. Physical exam should show no swelling or deformity, although there may be mild tenderness. The affected child will keep their arm in a pronated position and will resist supination and will cry out in pain at attempts at supination. Forearm hyperpronation or supination plus flexion usually reduces the annular ligament, with an audible click.

Removing firearms from the home, or locking unloaded firearms and ammunition in separate containers.

One of the most important ways to decrease suicide risk in adolescents, especially males.

testicular torsion

Orchiopexy reduces the risk of what complication in children with cryptorchidism?

age 5

Parents can be reassured that bedwetting is normal before what age?

neonatal immune mediated thrombocytopenia

Passive autoimmune thrombocytopenia which can occur when a mother has ITP and antibodies against her platelets cross the placenta and destroy the fetus' platelets.

*Chronic granulomatous disease:* caused by a gene defect in the NADPH oxidase enzyme complex, leading to an inability to form hydrogen peroxide and impaired intracellular killing in phagocytes. Diagnosis made by oxidative burst testing and confirmed by gene testing. Give patients prophylaxis with Bactrim and itraconazole.

Pateints with this disease will have recurrent skin and soft tissue abscesses caused by a variety of catalase positive organisms (Staph aureus, Serratia, Burkholderia, Aspergillus, Nocardia). Other common infections include suppurative adenitis and osteomyelitis.

17-hydroxyprogesterone (17-OHP)

Patients with 21-hydroxylase deficiency will have increased levels of what?

diffuse pruritic maculopapular rash (amoxicillin associated rash)

Patients with EBV infection (infectious mono) who are misdiagnosed with GABHS pharyngitis and prescribed amoxicillin often develop this complication a week after starting the antibiotic; this is not an allergic reaction but is idiosyncratic.

ADHD OCD

Patients with Tourette syndrome often have high rates of what comorbid conditions?

osteoporotic fractures

Patients with Turner syndrome are at increased risk of what due to estrogen deficiency from ovarian dysgenesis?

congenital lymphedema

Patients with Turner syndrome often have this abnormality, which results in accumulation of protein-rich interstitial fluid in the hands, feet, and neck. It is due to lymphatic network dysgenesis. It presents at birth with nonpitting edema due to high protein content in the interstitial fluid in the hands and feet.

IM benzathin penicillin G (give every 4 weeks)

Patients with a history of rheumatic fever are at high risk for recurrence and progression of rheumatic heart disease with repeated episodes of GAS pharyngitis. As such, these patients should receive what prophylactic medication to prevent recurrent Strep pharyngitis?

*Fat soluble vitamin (A,D,E, and K)* due to poor absorption from pancreatic insufficiency; vitamin K deficiency often leads to easy bruising, mucosal bleeding, and prolonged PT.

Patients with cystic fibrosis are at risk for deficiency of what vitamins?

head CT without contrast

Patients with mild TBI with repeated vomiting, headache, and LOC should undergo what kind of testing?

sickle cell disease

Patients with priapism likely have what underlying disorder?

beta thalassemia major

Patients with this anemia present with marked hepatosplenomegaly, and, if untreated, bone marrow hyperplasia resulting in characteristic facies with frontal bossing, maxillary hyperplasia with prominent cheekbones, and skull deformities. Delayed growth and puberty may also be present. Labs will show target cells and poikilocytes on smear, elevated unconjugated bilirubin, elevated iron ,and elevated LDH. Electrophoresis shows low HgA and elevated HgF. Managed with lifelong transfusions and often splenectomy.

B12 deficiency

Patients with this condition present with symptoms of anemia, anorexia, a smooth red tongue, and neurologic manifestations (ataxia, hyporeflexia, positive Babinski response).

tricuspid atresia

Patients with this congenital heart defect often present with ECG showing right atrial enlargement, left axis deviation, and left ventricular hypertrophy. This is the *only cause of central cyanosis* in the newborn period that results in LAD and LVH.

sickle cell anemia

Patients with this disease have chronic ischemia which causes autoinfarction of the spleen, making them prone to infection, especially by encapsulated organisms. They should receive pneumococcal vaccinations as well as prophylactic penicillin.

hyperthyroidism

Patients with this disease may have lid lag and exophthalmos, tachycardia, palpitations, warm flushed skin, nervousness, tremor, and delayed menarche.

polysplenia syndrome

Patients with this disease will have biliary atresia, bilobed lungs, abdominal heterotaxia, and stius ambiguous. Biliary atesia presents earlier than normal and progresses more rapidly.

autoimmune hepatitis

Patients with this disease will present with acute hepatitis mimicking viral hepatitis 50% of the time, with the other 50% presenting with chronic liver disease. Jaundice is usually mild to moderate. Fatigue, anorexia, arthritis, rash, nephritis, and vasculitis may also be present.

CVID

Patients with this disorder are at risk for infections by encapsulated bacteria, Giardia, and etnerovirus. Failure to thrive and chronic pulmonary diseases are common features. Manage with *IVIG* to prevent severe infection.

cyclic vomiting syndrome

Patients with this disorder have a recurrent, predictable pattern of acute and frequent vomiting that resolves spontaneously with no symptoms between episodes. Growth, exam, and eating pattern will all be normal, as this is a diagnosis of exclusion. Pathogenesis is unknown, but most patients have a personal or family history of migraine. Patients typically have regular, predictable symptom intervals and are asymptomatic between episodes.

Beckwith Wiedemann syndrome

Patients with this disorder present with hemihypertrophy, macroglossia, visceromegaly, and Wilms tumor.

Cornelia de Lange syndrome (aka Brachmann de Lange syndrome)

Patients with this genetic disorder will have a single eyebrow (synophrys), long, curly eyelashes, microcephaly, thin down-turned upper lip, and micrognathia. There will also be infantile hypertonia, mental retardation, small hands and feet, cardiac defects, autistic features, self-destructive tendencies, and lack of facial expression.

extrapyramidal cerebral palsy (athetoid cerebral palsy)

Patients with this subtype of cerebral palsy have problems modulating control of the face, trunk, and extremities, often writhing. Significant oral motor involvement often occurs.

DiGeorge syndrome

Patients with this syndrome will present with micrognathia, wide-spaced eyes, low-set ears, and absent thymic shadow.

toxic megacolon colon cancer

Patients with ulcerative colitis are at increased risk for what complications?

postpericardiotomy syndrome

Pericardial effusion which occurs within days to months of cardiac surgery. Life-threatening fluid accumulation is characterized by distant heart sounds, hypotension, and distended jugular veins (Beck's triad). It requires drainage.

postpericardiotomy syndrome

Pericarditis as a result of opening of the pericardium during surgery; the cause is unknown but thought to be autoimmune response to a concomitant viral infection

SCID

Persistent lymphompenia (<1,500 lymphocytes), decreased T cell by flow cytometry, severe hypogammaglobulinemia, and depressed T cellresponses to antigens are all signs of what diagnosis?

coxsackie

Pharyngitis plus a rash on the palms and soles points to infection by what?

adenovirus

Pharyngitis plus conjunctivitis points to infection by what?

Riboflavin (B2)

Photophobia, blurred vision, burning and itching of eyes, poor growth, and cheilosis are symptoms of what vitamin deficiency?

*Hypotonia:* may be caused by systemic pathology (sepsis, electrolyte abnormalities, hepatic or renal encephalopathy) or by neural pathology.

Physical findings of a weak cry, decreased spontaneous movement, and a frog-leg posture in a newborn would indicate what?

parapnuemonic pleural effusion

Pleural inflammation secondary to bacterial pneumonia that leads to leakage of exudative fluid into the pleural space. Patients may present with fever and cough, and exam usually reveals focal rales, decreased or absent breath sounds, or dullness to percussion over the affected area. CXR will confirm the diagnosis with presence of pleural fluid (obscuring costphrenic angle, layering fluid on lateral decubitus film). If mild, oral antibiotics and close outpatient follow-up are appropriate.

amblyopia

Poor vision caused by abnormal visual stimulation that results in abnormal visual development. The most common cause of decreased vision during childhood. Can be secondary to eye misalignment (strabismus), opacification of the lens (cataract), or any other pathologic condition that causes abnormal retinal stimulation.

long QT syndrome

Prolongation of the QT interval; increases risk of lethal ventricular arrhythmias known as *torsades de pointes.* Usually presents with syncope or sudden cardiac arrest. Diagnosis made by ECG showing QT interval (QTc) greater than 0.44 seconds. Treat with beta blocker to reduce symptoms.

*Migraines:* migraine without aura is hte most common form in children, though migraine with aura or migraine equivalent (other symptoms without headache itself) may also occur.

Prolonged (>1 hour) headaches with age of onset younger than 5 years in 20% of patients. Autosomal dominant. Due to changes in cerebral blood flow secondary to release of serotonin, substance P, and vasoactive intestinal peptide. The headache is throbbing and unilateral, starting with the supraorbital area and radiating to the occiput. Nausea, vomiting, and visual disturbances, along with photophobia and phonophobia, are common. Symptoms are improved by sleep but OTC analgesics are ineffective.

hypochloremic, hypokalemic metabolic alkalosis

Prolonged vomiting produces what electrolyte abnormalities?

hypochloremic, hypokalemic metabolic alkalosis

Protracted vomiting, as seen in pyloric stenosis, will produce what metabolic derangement?

glomerular bleeding, which usually occurs in acute or active glomerulonephritis

RBC casts are diagnostic of what?

Howell-Jolly bodies

RBC nuclear remnants which are typically removed by the spleen; if they are seen on peripheral smear, their presence strongly suggests asplenia or functional hyposplenia

proximal RTA (type II)

Renal tubular acidosis due to impaired bicarb reabsorption by the proximal renal rubular cells. Can be an isolated defect, due to intoxication by heavy metals or gentamicin, or associated with Fanconi syndrome. Presents with vomiting, growth failure, acidosis, and muscle weakness. Treat with large doses of oral alkali.

rickets (vitamin D deficiency)

Intestinal absorption of calcium and phosphorus is diminished in this disease. Transient hypocalcemia stimulates the secretion of parathyroid hormone and the mobilization of calcium and phosphorus from bone; enhanced parathyroid hormone activity leads to phosphaturia and diminished excretion of calcium. In children with nutritional rickets, the concentration of serum calcium usually is normal and the phosphate level is low. Increased serum alkaline phosphatase is a common finding. The excretion of calcium in the urine is increased only after therapy with vitamin D has been instituted.

cutaneous larva migrans

Intradermal migration of hookworms due to contact with contaminated feces or soil. Presents as migrating, pruritic, serpiginous, erythematous skin tracks. Self-limited and resolves without treatment in most cases.

Tensilon test

Intravenous injection of edrophonium chloride, a rapid acting cholinesterase inhibitor, which can produce transient improvement of ptosis and diagnose myasthenia gravis.

vitamin B6

Irritability, convulsions, and hypochromic anemia are symptoms of what vitamin deficiency?

Bilateral microtia or anotia Congenital heart disease CNS abnormalities

Isotretinoin will cause what deformity in the fetus if used by a pregnant woman?

Life-threatening CV sequelae, especially *coronary artery aneurysms.* The dilated arteries are prone to thrombotic occlusion and consequent myocardial ischemia and death. Echo should be performed at time of diagnosis and repeated 6-8 weeks later to assess for change; aspirin and IVIG should be started to decrease risk of complications.

Kawasaki disease patients who are untreated are at risk for what complication?

respiratory distress syndrome (RDS)

Respiratory distress caused by *lack of surfactant*, most frequently seen in *preterm* infants. Incidence higher in white males. Risk increases with low L:S ratio, a mother with previous infant with RDS, a mother with DM, neonatal hypothermia, and neonatal asphyxia. A CXR will show diffuse atelectasis with increased density in both lungs and fine, granular, ground-glass opacity. This is diagnostic. Small airways will be filled with air and are clearly seen surrounded by the increased density of the pulmonary field, creating air bronchograms.

Strep pneumo Haemophilus influenzae N meningitidis

Sickle cell patients are at risk of infection and sepsis from which organisms due to functional asplenia?

slipped capital femoral epiphysis

Slipping of the femoral head off the femoral neck. Usually occurs during adolescence in obese males or patients with hypothyroidism. Patients have a painful limp with pain in the groin, hip, or knee. Diagnosed with AP and frog leg lateral radiographs of the pelvis.

Libman Sacks endocarditis

Sterile valvular vegetations found in patients with SLE

malaria

Suspect this disease in any patient with a recent history of travel who presents with febrile paroxysms. Order blood smears for diagnosis. Recall that sickle cell trait confers some protection and demonstrates decreased morbidity,.

amniotocele (dacryocele)

Swelling of the nasolacrimal sac. Cause is accumulation of fluid as a result of nasolacrimal duct obstruction. Features include bluish swelling in the medial canthal area, with possible infection. Treat with local massage and, if infection present, IV antibiotics and urgent NLD probing.

trisomy 18

Rocker bottom feet is associated with what congenital condition?

*Noonan syndrome* "the male Turner syndrome"

This genetic disorder maps to chromosome 12. It features short stature and shield chest, short webbed neck and low hairline, widely spaced eyes, low set ears, epicenathal skin folds, and downslanting palpebral fissures. Cardiac defects include right sided heart lesions, most commonly pulmonary valve stenosis. Mental retardation is common.

Williams syndrome

This genetic disorder presents with a "cocktail party personality." It is caused by a deletion on chromosome 7, including a gene for elastin. Autosomal dominant. Other features include elfin facies, mental retardation, supravalvular aortic stenosis, idiopathic hypercalcemia, and connective tissue abnormalities (including hoarse voice and hernias).

osteogenesis imperfecta (OI)

This genetic disorder results from mutations that caused production of abnormal type I collagen. Features include blue sclerae, fragile bones resulting in frequent fractures, genu valgum, scoliosis or kyphosis, joint laxity, and osteopenia. There is also yellow or gray-blue teeth and easy bruisability. Complications include early conductive hearing loss and skeletal deformities as a result of fractures.

mitral stenosis

This heart murmur, with a *loud first heart sound* and a *mid diastolic rumble*, is almost always due to rheumatic fever, which is a complication of GABHS infection.

Necator americanus Ancylostoma duodenale (hookworms)

This helminth infection is acquired percutaneously through a bare foot; larvae migrate to the lungs and are coughed up and then swallowed. Rash and pruritus at site of penetration is common. They cause IDA with fatigue, pallor, failure to thrive. Treat with albendazole and screen close contacts; iron supplementation is advised.

Strongyloides stercoralis

This helminth, which is transmitted percutaneously from the soles of bare feet, causes transient pruritic papules at the site of penetration, pneumonitis, GI symptoms, and eosinophilia. Treat with ivermectin or albendazole.

leukocyte adhesion deficiency

This immune system disease will present with recurrent skin and mucosal bacterial infections (eg omphalitis, periodontitis) with no pus (lack of neutrophils at inflammations site) and poor wound healing. There is often a history of delayed umbilical cord separation (>21 days) and labs show marked peripheral leukocytosis with neutrophilia.

Hyper-IgM syndrome

This immunodeficiency is due to a defect in the CD40 ligand and is characterized by high IgM, low IgG and IgA, and normal lymphocyte populations

toxoplasmosis

This infection can be transmitted vertically from mother to child. It can be acquired in the mother by ingestion of cat feces, either directly or indirectly via contaminated soil or produce. It can also be acquired by ingestion of raw or undercooked meat from infected animals. Infants with the infection are asympatomatic at birth but experience chorioretinitis in adulthood along with macrocephaly, hydrocephalus, and diffuse intracerebral calcifications. Treatment is pyrimethamine, sulfadiazine, and folate for a year.

croup

This infection involves the larynx and trachea; it usually is caused by parainfluenza or respiratory syncytial viruses. The usual age range for presentation is 6 months to 6 years. Symptoms include low-grade fever, barking cough, and hoarse, inspiratory stridor without wheezing. The pharynx can be normal or slightly red and the lungs are usually clear. In children in severe respiratory distress, prolonged dyspnea can progress to physical exhaustion and fatal respiratory failure.

hepatitis A

This infection is caused by a picornavirus. It is transmitted fecal-orally and is the most common hepatitis virus causing infection. There is usually a 2-6 week incubation period followed by jaundice. Older children are more likely to be completely asymptomatic. Chronic infection does not occur. Diagnosis is based on serology; elevated IgM indicates recent infection and elevated IgG confers lifelong immunity. Management is supportive.

bacterial tracheitis

This infection is seen in children 5-7 years old. It resembles croup, but patients will be more ill-appearing and will not respond to racemic epinephrine. Subglottic narrowing may be seen on CXR. If suspicious, get a tracheal culture, visualize purulence, and treat with abx. Causative pathogens are usually Staph aureus, GAS, and anaerobles, along with Strep pneumo, Moraxella, H flu.

bronchiolitis

This infection tends to be most serious in children under 2, who have lower respiratory involvement. It is caused by RSV. It presents with wheezing and crackles with a waxing/waning course that peaks on days 5-7 of illness. Infants under 2 months are at high risk of developing *apnea* and resp failure as a complication. In addition, they tend to develop recurrent wheezing throughout childhood.

*HSV encephalitis:* diagnose with HSV-PCR and treat with acyclovir as empiric coverage.

This infection will present with a *bloody LP* and abnormal *hippocampus*.

otitis externa

This infection, commonly caused by Pseudomonas, presents as unilateral ear pain with pain on palpation of the pinna. Caused by frequent contact with water ("swimmer's ear"). Can also be caused by Staph aureus. On physical exam, an angry erythematous canal can be seen. It usually improves spontaneously; treat with topical antibiotics and topical steroids. Oral antibiotics are only needed if there is severe disease.

cat scratch disease

This infectious disease is caused by the Gram-negative bacteria Bartonella henselae. It presents with regional lymphadenopathy distal to and after a cat or kitten scratch. Fever may also occur.

GABHS pharyngitis

This infectious disease is usually seen in children 5-15 years old in the winter and spring. It is characterized by exudates on the tonsils, petechiae on the soft palate, strawberry tongue, and enlarged tender anterior cervical lymph nodes, as well as fever. Some patients may have a rash.

tuberculosis (TB)

This infectious disease presents with fever, chills, weight loss, cough, and night sweats. There can also be extrapulmonary manifestations, most commonly cervical lymphadenitis or meningitis. Radiographs will reveal hilar or mediastinal lymphadenopathy. There will also be a Ghon complex, a small parenchymal infiltrate with enlarged hilar lymph nodes.

roseola

This infectious rash is caused by HHV-7. There is a prodrome of high fever that breaks as the rash starts. It is a macular rash that begins on the trunk and spreads to the face. Febrile seizures may result.

measles

This infectious rash is caused by a paramyxovirus. There's an obvious prodrome of low grade fever. Characterized by *cough, coryza, conjunctivitis, and Koplik spots*. Rash starts on the face and spread to the body. The rash both spreads and clears from head to toe.

*rubella* ("German measles")

This infectious rash is caused by a togavirus. The rash itself resembles measles (starts on the face, spreads down). However, the rash tends to be fainter and spreads quicker. There is a prodrome of tender generalized (periorbital, postauricular) lymphadenopathy. There are Forchheimer spots (red enanthem) on the palate which can be seen.

*Supracondylar fracture:* it is an orthopedic emergency if the fracture is displaced and angulated because of the risk of neurovascular injury and compartment syndrome.Pain with passive extension of the fingers is suggestive of compartment syndrome.

This injury occurs when a child falls onto an outstretched arm or elbow. Presents with point tenderness, swelling, and deformity of the elbow. Diagnosed by radiographs.

Nursemaid's elbow (subluxation of the radial head)

This injury occurs with upward force on the arm, such as pulling a toddler up by the hand. This causes the radial head to slip out of the annular ligament which normally keeps it in place. Most often occurs in children under 6, who have slender radial heads. Patients present with sudden onset pain and an elbow held flexed with no swelling. Child is unwilling to use the affected arm, but hand function is normal. Treat by simultaneously flexing the elbow and supinating the hand.

short bowel syndrome

This is a lesion of the gut in patients who have required surgical resection of the small intestine resulting in an inadequate absorptive surface area. There will be carb and fat malnutrition and compromised bowel function with steatorrhea, dehydration, hyponatremia, and hypokalemia. If the ileum is resected, vitamin B12 deficiency can occur over time.

staphylococcal scalded skin syndrome

This is a rash seen most commonly in children under 5. The rash is preceded by fever, irritability, erythema, and extraordinary tenderness of the skin. Circumoral erythema; crusting of the eyes, mouth, and nose; and blisters on the skin can develop. Intraoral mucosal surfaces are not affected. Peeling of the epidermis in response to mild shearing forces (Nikolsky sign) leaves the patient susceptible to problems similar to those of a burn injury, including infection and fluid and electrolyte imbalance. Cultures of the bullae are negative, but the source site or blood often may be positive. Treatment includes antibiotics (to cover resistant S aureus) and localized skin care.

serial transcranial Doppler ultrasound or magnetic resonance angiography

This is recommended beginning at 2 years of age for sickle cells patients to identify patients at risk for stroke

Selective IgA Deficiency

This is the most common primary immune deficiency. It is usually asympatomatic, though recurrent sinopulmonary and GI infections are characteristic, and it is associated with autoimmune disease (eg celiac disease) and atopy (asthma, eczema). *Anaphylaxis occurs during blood transfusions.*

peak expiratory flow rates

This is used to monitor asthma symptoms at home, and used to guide changes in medication or for patients to know when to seek acute medical care.

developmental dysplasia of the hip (DDH)

This lesion occurs when the acetabulum is abnormally flat, leading to easy dislocation of the femur head. More common in girls. Risk factors include breech presentation, family history, and oligohydramnios. Barlow and Ortolani maneuver will be positive.

HCC

This liver tumor may occur in both young children and in adolescents; it is associated with chronic active hep B infection, biliary atresia, glycogen storage disease tpye 1, alpha 1 antitrypsin deficiency, and hereditary tyrosinemia.

*antiphospholipid antibodies* (eg positive lupus anticoagulant or anticardiolipin antibodies)

This marker, when present in SLE patients, reflects an increased risk of thrombotic events

octreotide (vasopressin)

This medication can be given to vasoconstrict esophageal varices.

IV prostaglandin E (PGE)

This medication if given urgently to infants with coarctation of the aorta to improve circulation to the lower body by opening the ductus arteriosus.

indomethacin

This medicine is used in premature infants to close a PDA medically. Recall that PDAs may also be closed surgically by coil embolization, video-assisted thoracoscopic surgery, and ligation in a thoracotomy.

sickle cell disease

This normocytic anemia is caused by a mutation of hemoglobin that results i npolymerization of hemoglobin within the RBC membrane when RBCs are exposed to low oxygen or acidosis; this results in a distorted RBC shape that leads to decreased RBC lifespan due to hemolysis and occlusion of small vessels resulting in distal ischemia, infarction, and organ dysfunction.

hereditary spherocytosis

This normocytic anemia presents with splenomegaly, pallor, weakness, pigmentary gallstones, and aplastic crises. Studies will show an elevated reticulocyte count, hyperbilirubinemia, and *abnormal RBC fragility with osmotic fragility studies*.

HUS

This occurs in a child who has recently recovered from a diarrheal illness and has an AKI, thrombocytopenia, and microangiopathic hemolytic anemia with schistocytes on peripheral smear.

intraventricular hemorrhage

This occurs in infants born before 34 weeks gestation. The immature lining of the ventricles ruptures prior to its physiologic involution at 34 weeks. Screen every premature neonate under 30 weeks with a cranial ultrasound. Manage intracranial pressures; baby may need a craniotomy or VP shunt.

breast feeding jaundice

This occurs in newborns under a week old who don't get a sufficient volume of breast milk. The body resorbs bilirubin and it builds up, resulting in indirect hyperbilirubinemia. Treat by increasing number of feeds.

*Febrile seizure:* a simple febrile seizure doesn't need a workup or imaging beyond identifying the cause of the seizure (usually viral URI, otitis media, etc). If the seizure is focal, >15 minutes duration, or recurrent within the day, investigate for complex febrile seizure with EEG, MRI, LP based on clinical suspicion.

This occurs when fever lowers the seizure threshold in children 6-60 months old, triggering a seizure. If it lasts over 5 minutes, *abort with benzos.* the use of antipyretics may reduce discomfort, but controlling the fever doesn't prevent recurrent seizures. Never give aspirin, as this can trigger Reye syndrome.

chylothorax

This occurs when lymphatic flow through the thoracic duct is disrupted, leading to direct leakage of the lymphatic fluid into the pleural cavity. Can present with tachypnea, hypoxia, and dullness to percussion. Pleural fluid analysis will demonstrated an exudative effusion, with characteristic *milky-white fluid* with lymphocyte predominance and increased triglycerides. Etiologies range from traumatic (eg cardiothoracic surgery) to congenital malformations and malignancy.

sponylolisthesis

This occurs when the body of the vertebra involved in spondylolysis slips anteriorly (subluxates). The subluxed vertebra can impinge on nerve roots. Diagnosis and treatment is the same as for spondylolysis (bone scan, analgesics, rest). Indications for surgery include nerve impingement, persistent pain, or prgression of subluxation.

Wolff Parkinson White

This occurs when there is SVT from anterograde conduction through a bypass tract between the atria and ventricles; it is associated with sudden cardiac death. It can be identified by the presence of a *delta wave* (slurred upslope of QRS complex with short PR interval) on the ECG.

acute iron poisoning

This occurs when young children swallow pre-natal vitamins which contain iron. Free radical production and lipid peroxidation result in abdominal pain and hematemesis, hypovolemic shock, and metabolic acidosis. Bicarb level will be latered, blood pressure will be low, and extremities will be cool. Abdominal x-ray may reveal radiopaque tablets in the gut (the iron containing vitamins). To treat, give IV volume resuscitation and IV *deferoxamine*.

Wilms tumor

This oncologic condition presents with an abdominal mass. Abdominal pain, hematuria, HTN, and nonspecific fever, anorexia, and weight loss can also uccr.

Osgood-Schlatter disease

This orthopedic illness occurs in teenage athletes. It presents as a painful knee with swelling over the tibial tubercle. The athlete can either stop exercising (curative) or play through it (can result in a nodule). Otherwise, it causes no permanent sequelae but does hurt.

slipped capital femoral epiphysis

This orthopedic problem is characterized by displacement of the femoral head on the femoral neck due to disruption of the proximal femoral growth plate. Commonly seen in obese adolescent boys. Patients typically present with hip or referred knee pain of insidious onset that causes limping, though acute presentations can occur. Physical exam shows loss of abduction and internal rotation of the hip as well as external rotation of the thigh while the hip is being flexed. A frog-leg, lateral view x ray of the hip is diagnostic. Patients should be promptly treated with surgical pinning of the slipped epiphysis of the femoral head.

slipped capital femoral epiphysis

This orthopedic problem usually presents in obese children 10-16 years old. Patients classically present with insidious onset of dull hip or referred knee pain and altered gait with no preceding trauma. Minor trauma can sometimes exacerbate the pain and bring it to medical attention. On exam, patients tend to hold the affected hip in passive external rotation and exhibit decreased internal rotation, abduction, and flexion. Diagnosis is made with x-rays of the hip which show displaced femoral head; gold standard treatment is immediate surgical screw fixation to avoid avascular necrosis.

pinworm

This parasite presents with a pruritic anus. Tape test will be positive; treat with albendazole.

lice

This parasite will present with itchy scalp. A comb can be used to go through hair and find nits. Treat definitively with permethrin.

giardiasis

This parasitic infection is acquired by drinking contaminated water; it is characterized by diarhhea which is described as voluminous, watery, and foul smelling. Abdominal pain, cramping, bloating, flatulence, weight loss, and low-grade fever may also occur. Diagnosis is via cysts and trophozoites in the stool or by stool ELISA tests; treatment is with metronidazole.

scabies

This pathogen causes *pruritis.* It is located in the webs of the hands and genitalia; it burrows into the skin, and if the skin is scraped, eggs can be seen on the microscope. Treat with topical permethrin or lindane.

Shigella sonnei

This pathogen causes bloody diarrhea; children may develop seizures secondary to neurotoxin release. Stool WBCs will be present; culture is diagnostic. Treat with third-gen cephs or fluoroquinolones.

EPEC (Enteropathogenic E coli)

This pathogen causes noninvasive watery diarrhea in preschoolers; stool WBCs are absent and diagnosis can be made on stool culture. Oral sulfonamides or quinolones are indicated; hydration is essential.

EHEC 0157:H7 (enterohemorrhagic E coli)

This pathogen is responsible for HUS via endotoxin release. Stool WBCs will be present and culture is diagnostic. Avoid antibiotic therapy, as it may worsen HUS as a result of enhanced endotoxin release.

Yersinia enterocolitica

This pathogen may cause mesenteric adenitis and gastroenteritis which mimics acute appendicitis. Third-gen cephs can treat.

Vibrio cholerae

This pathogen, seen in developing countries, causes diarrhea characterized by massive water loss. Fluid replacement is critical.

absence seizures

This pathology is a sudden onset of inattention which can occur at any time. Length is generally under 20 seconds. There is lack of response to vocal or tactile stimulation. Automatisms are present, such as eyelid fluttering or lip smacking.

Staphylococcal scalded skin syndrome

This pathology is caused by a Staph aureus species that produces an exfoliative toxin. Presentation includes fever, tender skin, and bullae. Large sheets of skin slough several days after the illness begins, and the Nikolsky sign (Extension of bullae with application of pressure to skin) is present.

hemolytic uremic syndrome

This pathology is characterized by the triad of *hemolytic anemia, thrombocytopenia, and acute kidney injury.* Most commonly seen in children, due to Shiga toxin producing diarrheogenic pathogens like E coli O157:H7 or Shigella. It often presents with fatigue and pallor following resolution of bloody diarrheal illness. Fever is uncommon at time of presentation. Management is supportive. Most patients improve within 2-3 weeks.

transient tachypnea of the newborn

This pathology is usually seen after a scheduled cesarean delivery, but can be seen in infants born via cesarean after a trial of labor, as well as in spontaneous vaginal deliveries. The condition is a result of retained fetal lung fluid. These patients have tachypnea, retractions, grunting, and sometimes cyanosis. The chest examination is usually normal; the chest radiograph demonstrates prominent pulmonary vascular markings with fluid in the fissures and hyperexpansion (flat diaphragms). Therapy is supportive, with maintenance of normal oxygen saturation. Resolution usually occurs in the first 3 days of life.

hemopholic arthropathy

This pathology occurs in patients with hemophilia who have recurrent hemarthroses. Hemosiderin deposition within the joint triggers synovial inflammation, which leads to fibrosis and destruction of cartilage and bone. Chronic, worsening joint pain and swelling are accompanied by limited mobility on exam. MRI allows for detection. Early prophylaxis with factor concentrates should be given to hemophilia patients to avoid this.

*Meckel diverticulum:* diagnose with technetium-99m pertechnetate scan, which can identify ectopic gastric mucosa.

This pathology often presents in young toddlers as painless hematochezia without abdominal pain, diarrhea, or vomiting.

*Acute iron poisoning:* iron is radiopaque, and ingested tablets can be seen on x-ray. Chelation therapy with deferoxamine should be provided for moderately to severely ill patients.

This pathology presents as abdominal pain, hematemesis, hypotensive shock, and metabolic acidosis.

transient synovitis

This pathology should be on your differential for septic hip. It is synovial inflammation up to 4 weeks after URI or GI viral illness. Differentiate from septic hip by lack of fever, no leukocytosis, and decreased inflammatory markers (negative Kocher criteria). X-ray will be normal; treat supportively.

*Kasai portoenterostomy:* roux-en-Y intestinal loop attached to the porta hepatis. The treatment of choice to establish bile flow. Success is highest with younger patients and success diminishes with older patients. Look out for cholangitis, a worrisome complication of the procedure that occurs in as many as 50% of patients.

This surgical procedure can be used to treat biliary atresia

Chediak Higashi syndrome

This syndrome is characterized by variable neutropenia and thrombocytopenia and giant lysosomal granules in neutrophils. Neutrophils and monocytes have functional defects, and NK cell function is impaired. S aureus causes the majority of infections. Patients also have partial oculocutaneous albinism.

Waardenburg syndrome

This syndrome is inherited as an autosomal dominant trait with variable penetrance. It includes, in decreasing order of frequency, the following anomalies: lateral displacement of the medial canthi, broad nasal bridge, medial hyperplasia of the eyebrows, partial albinism commonly expressed by a white forelock or heterochromia (or both), and deafness in 20% of cases.

Turner syndrome

This syndrome occurs when only one X chromosome is present. Presents with short stature, webbed neck, shield chest, streak ovaries. It should be considered in any female with pubertal delay.

Parinaud syndrome

This syndrome, due to pineal gland tumors causing mass effect on the dorsal midbrain, presents with limited upward gaze, upper eyelid retraction (Collier sign), and pupils which are non-reactive to light but reactive to accomodation. Signs of obstructive hydrocephalus (papilledema, headache, vomiting, ataxia) may also be present due to mass effect.

Apt-Downey test

This test distinguishes fetal hemoglobin from adult hemoglobin based on the specimen's reaction to alkali (fetal hemoglobin is unchanged, whereas adult hemoglobin changes to hematin). Infants may swallow blood during delivery or from a cracked nipple during breast-feeding and present with bloody stool.

Apt-Downey test

This test, ordered when hematemesis or melena occur in the neonatal period, differentiates fetal from adult hemoglobin in a bloody specimen. Initially described in 1955, the test is based upon the finding that fetal hemoglobin is alkali resistant, while adult hemoglobin will convert to hematin upon exposure to alkali. If the blood in an affected infant's gastric contents or stool is maternal in origin, further workup of the infant is not necessary. There for, order this test before any other further workup in a neonate with hematemesis or melena.

ehrlichiosis (Ehrlichia chaffeensis)

This tick-borne disease occurs in the spring and summer in the southeastern US. It has many of the same symptoms of Rocky Mountain Spotted Fever, but usually no rash, and is sometimes called "spotless RMSF." Symptoms include fever, headache, myalgias, and lymphadenopathy. Treat with doxycycline.

Ewing's sarcoma

This tumor occurs in the flat bones and diaphysis of tubular bones. It presents with pain, swelling, and soft tissue mass. Systemic signs are common and ESR is elevated. On x-ray, the "onion skin" appearance is characteristic.

Gonococcal conjunctivitis (N. gonorrhoeae)

This type of conjunctivitis will present in an infant 2-5 days old with copious exudate and eyelid swelling. It is the most severe form of neonatal conjunctivitis. If left untreated, it can progress to corneal ulceration, scarring, and blindness. Infected infants should get a single IM dose of third-gen cephs (eg cefotaxime, ceftriaxone). Prophylactic erythromycin ointment will not prevent it.

greenstick fracture (incomplete fracture)

This type of fracture occurs if only one side of the cortex is fractured with the other side intact. The intact side of the site of compression injury and may be bent, whereas the fractured side receives the tension and fractures. Because angulation can increase even within a cast, reduction may include fracturing the other side of the cortex.

coxackievirus pharyngitis (hand-foot-mouth disease)

This type of pharyngitis may present with painful vesicles or ulcers on the posterior pharynx and soft palate, called herpangina. Blisters may also be present on the palms and soles.

distal RTA (type I)

This type of renal tubular acidosis is characterized by the inability of the distal renal tubular cells to excrete H+ (acid). It can be an isolated inherited defect, associated with nephrotic syndrome, or associated with amphotericin. Presents with vomiting, growth failure, and acidosis. Nephrocalcinosis and nephrolithiasis occur if left untreated. Treat with small doses of oral alkali.

*Focal seizures:* can secondarily generalize to involve both hemispheres and impair consciousness.

This type of seizure originates in a single hemisphere of the brain and can involve motor (head turning, arm twitching), sensory (paresthesias) and autonomic ((sweating) activity. Automatisms (chewing, sucking) can be seen. Following the seizure, postictal confusion and lethargy are common, and transient postictal paresis or paralysis (Todd paralysis) may also occur.

*Growth hormone deficiency:* sometimes caused by brain tumors such as craniopharyngioma. All patients with GH deficiency must have an MRI of the head to rule out a CNS lesion. In addtiion, look for low IGF-1 and a poor response to growth hormone stimulation testing. Treat with daily subcutaneous injections of recombinant growth hormone.

This uncommon endocrinopathy can cause short stature. It may also present with prolonged neonatal jaundice, hypoglycemia, cherubic facies, central obesity, microphallus, cryptochidism, and midline defects like cleft palate. The growth curve will demonstrate poor growth velocity (less than 2 inches or 5 cm per year)

Candidal vulvovaginitis

This vaginal infection presents with severe itching and a white, curdlike discharge. Fungal hyphae will be seen on wet-mount. Vaginal pH will be normal (< 4.5). Manage with oral fluconazole; partners do not require treatment.

Tb meningitis

This variant of meningitis often demosntrates *basilar enhancement* on brain iamging.

congenital rubella

This vertically transmitted infection presents with cataracts or glaucoma, sensorineural hearing loss, and congenital heart disease (eg PDA). IT can be prevented by maternal immunization with live attenuated vaccine prior to conception.

syphilis

This vertically transmitted infection presents with nonspecific signs of congenital infection (jaundice, hepatosplenomegaly, blueberry muffin spots, growth restriction). More specific findings highyl suggestive of this diagnosis are *sniffles* (copious rhinorrhea) and a maculopapular rash that may *desquamate or become bullous.*

mumps

This viral infection causes bilateral parotid swelling and orchitis in pubertal males. Rarely, sterility occurs. Has a nonspecific prodrome.

mumps

This viral infection presents with fever and parotitis after a nonspecific prodrome. It is most common in school-aged children, who can often have mild disease or may even be asymptomatic. In general, it is self-limiting, but serious complications are possible, with aseptic meningitis and orchitis being most common

diphtheria

Though this disease is rare in the developed world, it will present as pharyngitis with a gray, adherent tonsillar membrane. It is treated with oral erythromycin or parenteral penicillin and a specific antitoxin available from the CDC. Respiratory isolation is very important to prevent spread of infection.

acute apncreatitis

Though uncommon in children, this disease can occur through insults to the pancreas such as ductal obstruction or viral infection, which causes premature activation of pancreatic proenzymes and autodigestion of pancreatic cells. The most common cause is *blunt trauma*, followed by idiopathic (25% of cases) and infections (mumps, enterovirus, EBV, HIV, hep A, hep B). Inflammatory mediators and cytokines damage the pancreas and cause interstitial edema and necrosis. Necrosis of blood vessels may lead to parenchymal hemorrhage.

between 400 mcg and 800 mcg of folic acid daily

To reduce the risk of neural tube defects, it is now recommended that all women capable of becoming pregnant take what medication?

type IV RTA

Transient renal tubular acidosis in infants and children characterized by *hyperkalemia.* Associated with obstructive uropathy and aldosterone deficiency. Patients may be asymptomatic or have FTT. Treat with furosemide to lower serum K+ and oral alkali.

*Vitamin A:* promotes antibody producing cells and regeneration of epithelial cells in the gut, lungs, and retina.

Treatment with what medication reduces morbidity and mortality rates for patients with severe measles (high fever, morbilliform rash, cough, conjunctivitis, coryza requiring hospitalization)?

VSD

Trisomy 18 is associated with what congenital heart defect?

HSV

Vertical transmission of this pathogen often occurs perinatally during delivery by a mother with active genital lesions. Characteristic vesicles on an erythematous base are prominent in skin-eye-mouth disease; however, cutaneous findings may be absent in CNS and disseminated disease. CNS disease will present with encephalitis, seizure, lethargy, poor feeding, and a full fontanelle. There is often temporal lobe hemorrhage and edema. Diagnostic LP reveals a bloody tap. Start empiric acyclovir while awaiting confirmatory PCR testing of the CSF.

recurrent UTIs leading to progressive renal scarring; as such, all children with UTI at age 2-24 months should get a renal ultrasound to evaluate for VUR. Those with recurrent UTIs should also undergo a voiding cystourethrogram to evaluate for VUR.

Vesicoureteral reflux is a risk factor for what renal pathology?

epidemic keratoconjunctivitis

Viral conjunctivitis caused by *adenovirus* and characterized by petechial conjunctival hemorrhage, preauricular lymphadenopathy, and a pseudomembrane along the conjunctiva. The cornea will be inflamed, causing photophobia. There will be no fever or pharyngitis. This is highly contagious. Treatment is supportive.

*Bartonella henselae:* the major etiologic agent for cat-scratch disease (the 5-year-old with a feline scratch). History of a scratch or a bite from a kitten is often positive, and fleas are often a factor in transmission. Diagnosis is usually by history and presenting signs and symptoms, as described in the question. Another form of the disease, Parinaud oculoglandular syndrome, occurs when the primary site of inoculation occurs in or near the conjunctivae, resulting in a moderately severe conjunctivitis and preauricular lymph adenopathy.

Warm, red, tender axillary lymph nodes, and red papules on the hand of a 5-year-old girl who is happily telling you of her recent adventures at her grandmother's farm where she enjoyed playing with Chihuahua puppies, kittens, and ducklings is likely caused by what organism?

cefepime (for MSSA, P. aeruginosa) IV vanc (for MRSA)

What antistaphylococcal antibiotics should be administered immediately in patients with cystic fibrosis who have severe pneumonia?

oral penicillin VK IM benzathine penicillin erythromycin (for penicillin allergies)

What are appropriate antibiotics to treat scarlet fever?

recurrent otitis media hearing loss speech defects

What are common complications of cleft lip and palate?

1) malar rash 2) photosensitivity 3) alopecia 4) Raynaud's phenomenon

What are common skin findings in SLE?

>2 cm firm immobile systemic symptoms

What are considered concerning findings of lymph nodes that should be worked up? Note that in the absence of these findings, palpable lymph nodes can simply be observed for resolution before further workup is performed.

first week of life: 4 per day first year of life: 2 per day by 4 years old: 1 per day adults: from 3/day to 3/week

What are normal defecation frequencies for different age groups?

Delayed meconium passage (>48 hours after birth), onset of constipation during infancy, history of pelvic surgery, encopresis before 3 years of age, inability to toilet train, all suggest organic cause. Development of constipation after toilet training, identification of a sentinel event, and continued otherwise normal growth and development, all suggest FFR.

What are some clues that will help you differentiate organic causes of constipation from functional fecal retention?

hypoxia polycythemia hypoglycemia hypothermia hypocalcemia

What are some common complications which occur for infants born SGA?

failure to thrive delayed sexual development oral aphthous ulcers erythema nodosum arthritis renal stones

What are some extraintestinal complications of Crohn's disease in children?

burns with sharp lines of demarcation, uniform depth, lack of splash marks, and spared flexural creases are all signs that a scald injury may be the result of child abuse where the child was forced into a bathtub with hot water.

What are some marks of suspected child abuse in children with scald injuries?

Prenatal exposure to cocaine and other vasoconstrictive drugs is the major risk factor

What are some risk factors for jejunal atresia, which presents with bilious vomiting and abdominal distention and "triple bubble" sign of abdominal x ray?

Orients to voice = 4 months Orients to name, gestures = 9 months Object permanence = 9 months Says "mama" and "dada" = 10 months

What are some verbal and cognitive milestones in the first year of life?

Tetralogy of Fallot Transposition of the great arteries Tricuspid atresia Truncus arteriosus TAPVR

What are the "five Ts" which cause central cyanosis due to right-to-left shunts caused by congenital heart defects?

insomnia nervousness rebound rhinitis

What are the adverse effects of pseudoephedrine, a decongestant?

*PSGN* *HUS* IgA nephropathy MPGN crescentic glomerulonephritis

What are the causes of nephritic syndrome (with the common pediatric causes in bold)?

*minimal change disease* FSGS membranous nephropathy MPGN

What are the causes of neprhotic syndrome (with bold causes common in the pediatric population)?

Marfanoid body habitus Intellectual disability Downward lens dislocation Hypercoagulability (*CVA*) Fair hair and eyes

What are the characteristic components of homocysteinuria?

*bilateral ptosis* weakness later in the day weakness with sustained activity diplopia preserved DTRs

What are the characteristic findings of juvenile myasthenia gravis?

A murmur that is diastolic or >3/6 is *never* innocent. Innocent heart murmurs are always systolic and low-grade. They don't need workups. If they persist, they are no longer innocent and you should get a CXR, EKG, echo.

What are the characteristics of an *innocent* (that is to say, non-patholigc) heart murmur

*Fever for 5 or more days* plus 4+ of the following: 1) Conjunctivitis: bilateral, nonexudative, spares limbus 2) Oral mucosa: erythema, fissured lips, strawberry tongue 3) Rash 4) Extremities: erythema, edema, desquamation of hands and feet 5) Cervical lymphadenopathy

What are the characteristics that constitute Kawasaki disease?

fever or hypothermia poor feeding lethargy mild jaundice

What are the classic symptoms of neonatal sepsis? Recall that blood, urine, and CSF cultures should be obtained from infants with suspected sepsis before antibiotics are given.

Stain (port wine facial stain) Tram track calcifications Unilateral Retardation Glaucoma Epilepsy

What are the clinical features of Sturge Weber syndrome?

*Acute:* anorexia, apathy, lethargy, anemia, irritability, vomiting; may progress to encephalopathy *Chronic:* most commonly asymptoatmic, but neurologic sequelae like developmental delay, learning problems, and metnal retardation occur.

What are the clinical features of acute and chronic lead poisoning?

1) immunodeficiency (sinopulm infections) 2) progressive cerebellar ataxia 3) telangiectasias 4) malignancies (lymphomas) 5) cafe au lait spots 6) vitiligo 7) prematurely gray hair

What are the clinical features of ataxia telangiectasia?

1) altered LOC 2) nuchal rigidity 3) seizures 4) photophobia 5) emesis 6) headache

What are the clinical features of bacterial meningitis in older children? Recall that in infants and young children, symptoms can be minimal and nonspecific (fever and bulging fontanelle are clues).

1) goiter 2) myxedema ("puffy skin") 3) amenorrhea or oligomenorrhea 4) suboptimal growth velocity (less than 5 cm per year or 2 inches per year) with a delayed bone age.

What are the clinical features of hypothyroidism in children?

cafe-au-lait macules axillary/inguinal freckling Lisch nodules (iris hamartomas) neurofibromas optic pathway glioma (OPG) NF1 patients are at increased risk of neurologic disorders (eg cognitive deficits, learning disabilities, and seizures), as well as intracranial neoplasms.

What are the clinical features typically seen in neurofibromatosis type 1?

fever ear pain decreased hearing pus/fluid drainage (if tympanic membrane perforates)

What are the clinical symptoms of acute otitis media (AOM)?

*<1 month:* Group B Strep E coli (and other gram negatives) Listeria monocytogenes Herpes simplex virus *>1 month:* Strep pneumo N. meningitidis

What are the common causative agents of meningitis in children?

Mumps and other viruses (CMV, EBV, HIV, influenza) usually cause *bilateral* involvement of the parotid glands. Before vaccines were widely available, mumps was the most common cause of parotitis. Bacterial parotitis (Staph aureus, Strep pyogenes, M tuberculosis) usually result in *unilateral* parotid involvement.

What are the common etiologic agents which cause parotitis (inflammation of the parotid salivary glands)?

hypotonia weakness feeding problems

What are the common findings in neonatal myasthenia gravis?

Strep pyogenes (GABHS) coxsackievirus Epstein Barr virus cytomegalovirus Arcanobacterium hemolyticum Corynebacterium diphtheriae

What are the common infectious causes of pharyngitis?

Wilms tumor Aniridia Genitourinary abnormalities mental Retardation

What are the component to WAGR syndrome?

1) *Insulin:* regular insulin 0.1 units/kg/hr 2) *Fluids:* 10-20 ccs/kg bolus NS, followed by 2.5-3.0 L/m2/24 hours 1/2 NS. Calculate body surface area by the formula square root of (body weight (kg) X height (cm) / 3600). 3) *Potassium:* None if hyperkalemic, 40 mEq/L if normokalemic, 60 mEq/L if hypokalemic. Use potassium acetate and potassium phosphate half and half when possible.

What are the components of DKA treatment?

peripheral precocious puberty irregular cafe au lait spots polyostotic fibrous dysplasia (reccurent fractures)

What are the components of McCune Albright syndrome?

The importance and urgency of the lumbar puncture in cases of suspected meningitis outweigh the usual niceties in the performance of procedures. Infants and children require adequate restraints, preferably local anesthesia, and sometimes sedation. Contraindications are few and include increased *intracranial pressure in the patient without an open fontanelle that can result in herniation; severe cardiorespiratory distress; skin infection at the puncture site; and severe thrombocytopenia or other coagulation disorder, suggested by the oozing IV and venipuncture sites.*

What are the contraindications to lumbar puncture in cases of suspected meningitis?

1) anaphylaxis to vaccine ingredients 2) history of intussusception 3) history of SCID 4) history of uncorrected congenital malformation of the GI tract, such as Meckel's diverticulum

What are the contraindications to rotavirus vaccine, which is given at age 2-6 months?

SGA < 5% LGA > 90%

What are the cutoffs for percentile weights which are considered SGA and LGA infants?

*Inpatient:* IV cefoxitin plus oral doxycycline, or IV clindamcyin plus IV gentamicin. *Outpatient:* 14 days ofloxacin and clindamycin, or single dose IM ceftriaxone and 14 days of doxycycline.

What are the differences in inpatient vs outpatient treatment of PID? Recall that the indications for hospitalization include presence of an adnexal mass, uncertainty regarding diagnosis or compliance, pregnancy, or failed outpatient therapy.

1) fasting glucose > 125 2) 2-hour oral glucose tolerance > 200 3) random plasma glucose > 200, + symptoms

What are the different criteria for diagnosing diabetes mellitus?

1) failure to thrive 2) thrombocytopenia 3) recurrent infections 4) lymphadenopathy 5) parotitis 6) recurrent thrush 7) loss of developmental milestones 8) severe varicella or zoster

What are the early symptoms of HIV infection?

Emergency care (life-saving) Sexually transmitted infections Mental heatlh/substance abuse treatment Pregnancy care and contraception Legal emancipation

What are the exceptions to the need to obtain informed consent from parents of minors prior to performing medical care?

1) fever for 5+ days 2) conjunctivitis, bilateral, nonexudative 3) oral mucosa erythemia, fissured lips, "strawberry tongue" 4) polymorphous rash on the trunk 5) erythema, edema, desquamation of hands and feet 6) cervical lymphadenopathy Diagnosis is clinical and requires fever plus 4/5 of the other diagnostic criteria. There will also be elevated CRP and ESR, thrombocytosis, and sterile pyuria.

What are the features of Kawasaki disease, an acute vasculitis of small and medium arteries that occurs in children under 5, usually of Asian ethnicity?

hypertension hematuria/casts oliguria proteinuria casts (HHOP)

What are the features of nephritic syndrome?

edema fatigue hypoalbuminemia elevated urine protein absence of hematuria

What are the features of nephrotic syndrome?

microcephaly low set ears prominent occiput IUGR micrognathia closed fists, overlapping digits rocker bottom feet ventricular septal defects intellectual disability

What are the features of trisomy 18 (Edwards syndrome)?

*C*ardiac anomaly *A*bnormal facies *T*hymic hypoplasia *C*left palate *H*ypocalcemia *22* chromosome defect

What are the features seen in both DiGeorge syndrome and velocardiofacial syndrome, the 22q11.2 deletion syndromes?

Cafe au lait spots Axillary and inguinal freckling Lisch nodules (iris hamartomas) Neurofibromas (nerve sheath tumors) Optic gliomas

What are the features seen in neurofibromatosis type 1, and autosomal dominant disease caused by mutations in the NF1 gene, which codes for neurofibromin?

First, airway secretions should be cleared by suction, and the infant should be dried and kept warm. Soon after, apply silver nitrate solution to the eyes (to prevent neonatal gonococcal ophthalmia) and administer IM vitamin K.

What are the first priorities in newborn care if Apgar scores are good?

mandibular central incisors

What are the first teeth to erupt in most children?

*I*nterventricular septal defect *H*ypertrophy of the right ventricular *O*verriding aorta *P*ulmonary stenosis

What are the four anatomic components of the tetralogy of Fallot, which is the most common cause of central cyanosis presenting beyond the newborn period?

1 min = 60-65% 5 min = 80-85% 10 min = 85-95%

What are the goal O2 sats for an infant at 1, 5, and 10 minutes after birth?

The New England states and parts of the Pacific coast and Midwest

What are the high risk areas for Lyme disease?

*Blood culture* ESR (usually elevated) Rheumatoid factor (positive in 50%) *TEE* (more sensitive than TTE for vegetations)

What are the important lab studies and imaging to obtain in suspected infective endocarditis?

noncardiogenic pulmonary edema (aka acute chest pain), encephalopathy or focal neurologic deficits (stroke), and priapism.

What are the indications for exchange transfusion for sickle cell patients?

1) allergic shiners 2) transverse nasal crease 3) pale, boggy nasal mucosa 4) cobblestoning of posterior oropharynx

What are the key physical exam findings seen in allergic rhinitis?

prolonged aPTT prolonged PT prolonged bleeding time low platelet count petechiae sometimes hemarthroses

What are the lab findings in DIC?

prolonged aPTT normal PT normal bleeding time normal platelet count no petechiae hemarthroses

What are the lab findings in hemophilia (factor VIII/IX deficiency)?

normal aPTT normal PT prolonged bleeding time normal platelet count petechiae no hemarthroses

What are the lab findings in platelet function defects?

normal aPTT normal PT prolonged bleeding time low platelet count petechiae no hemarthroses

What are the lab findings in thrombocytopenia?

prolonged aPTT prolonged PT normal bleeding time normal platelet count petechiae hemarthroses

What are the lab findings in vitamin K deficiency?

prolonged aPTT normal PT prolonged bleeding time normal platelet count no petechiae rare hemarthroses

What are the lab findings in von Willebrand disease?

1) sensorineural hearing loss 2) patent ductus arteriosus 3) congenital cataracts

What are the main components of congenital rubella?

Human milk absorbs better and improves gastric emptying due to high whey content. It also contains lactoferrin, lyoszyme, and secretory IgA that confers immunity. Although calcium and phosphorus content is higher in formula, these minerals are better absorbed from human milk. Breast milk has an inadequate supply of vitamine D and exclusively breast-fed infants should get supplements. Human milk is associated with less reflux and colic than formula.

What are the major differences between human breast milk and formula for infants?

Gradually replete fluids and electrolytes so as not to cause cerebral edema. Replete potassium using potassium acetate and potassium phosphate. Give regular insulin with careful monitoring. The combination of IV fluids and insulin should reverse ketogenesis.

What are the management strategies for DKA?

fever arthralgias leukocytosis increased ESR prolonged PR interval of ECG

What are the minor criteria of rheumatic fever? Recall that the Jones criteria requires evidence of recent strep infection and either two major criteria or one major plus two minor.

Strep pneumo nontypeable H flu

What are the most common causes of acute bacterial rhinosinusitis? Recall that treatment of choice is amoxicillin-clavulanic acid.

In children <5, nonaccidental trauma and near-drowning are the most common causes of coma. In older children, drug OD and accidental head injury are the most common causes of coma.

What are the most common causes of coma based on age?

small palpebral fissures smooth philtrum thin vermilion border microcephaly

What are the pathognomonic facial features of fetal alcohol syndrome?

*21-hydroxylase deficiency* most common; can also be virilizing drug used by mom during pregnancy or virilizing tumor in mother during pregnancy.

What are the possible causes of a genetic 46,XX female with ambiguous genitals?

Error in testosterone synthesis Mixed gonadal dysgenesis (45,XO/46,XY mosaicism) True hermaphroditism Partial androgen insensitivity

What are the possible causes of a genetic 46,XY male with ambiguous genitalia?

*pruritus* weeping and crusting lesions lichenification (thickening) dry scaly skin pigmentary changes secondary bacterial infection personal or family history of atopy

What are the signs and symptoms of atopic dermatitis?

constipation prolonged jaundice sluggishness poor feeding apnea choking macroglossia excessive sleepiness

What are the signs of congenital hypothyroidism?

sudden onset crampy abd pain vomiting and lethargy "currant jelly" stools

What are the symptoms of intussusception?

1) monthly IVIG replacement 2) PCP prophylaxis 3) bone marrow transplant

What are the tenets of SCID management?

oral antibiotics close monitoring of BP fluid restriction

What are the tenets of treatment of PSGN?

1) hyponatremic (Na+ < 130) 2) isonatremic (Na+ 130-150) 3) hypernatremic (Na+ > 150)

What are the three classifications of dehydration by initial serum sodium level?

microangiopathic hemolytic anemia thrombocytopenia acute renal failure

What are the three components of HUS?

prevent premature birth ante-natal steroids perinatal surfactant

What are the three treatment strategies for NRDS?

Epstein pearls

small, white epidermoid-mucoid cysts found on the hard palate of the newborn, which usually disappear within a few weeks.

*Spina bifida occulta:* there is no herniation of tissue through the vertebral cleft, but there is a failure of bone fusion in the posterior midline of the vertebral column. The skin on the back often includes a hairy patch or dimple over the area. *Meningocele:* the meninges herniate through the cleft, most commonly in the lumbosacral area. A fluctuant midline mass is present overlying the spine; the mass is filled with CSF and can be transilluminated. *Myelomeningocele:* both spinal cord tissue and meninges herniate through the bony cleft, most commonly in the lumbosacral region; 20 times more common than meningocele. A fluctuant midline mass is present overlying the spine. Neurologic defects are present.

What are the three types of neural tube defects?

1) allergen avoidance is key 2) intranasal steroids most effective 3) intranasal antihists also 1st line 4) oral antihistamines also used

What are the treatment preferences for allergic rhinitis?

1) Supportive cool mists/fluids 2) Systemic corticosteroids (if stridor at rest) 3) Racemic epinephrine aerosols (if in resp distress) 4) Albuterol when wheezing

What are the treatments for croup?

*Type I:* characterized by presence of antinuclear antibody (ANA) or anti-smooth muscle antibody. More common. *Type II:* characterized by anti-liver kidney microsome antibody or anti-liver cytosol type I antibody.

What are the two types of autoimmune hepatitis?

*AVRT:* atrioventricular re-entrant tachycardia, where retrograde conduction through an accessory pathway leads to SVT. *AVNRT:* AV node re-entrant tachycardia, where the conduction abnormality occurs in different pathways within the AV node itself.

What are the two types of supraventricular tachycardia (SVT) that can occur? Recall that SVT is an abnormally accelerated heart rhythm that originates proximal to the bifurcation of the bundle of His, and it is the most common dysrhythmia in childhood.

Rhinorrhea Abnormal long-bone radiographs Desquamating or bullous rash

What are the typical findings in an infant with congenital syphilis?

persistent fever weight loss night sweats palpable or visible mass

What are the typical presenting features of childhood cancer?

*BAER:* brainstem auditory evoked response. Measures EEG waves generated in response to clicks. The most accurate test, but costly. *EOE:* evoked otoacoustic emission. Measures sounds generated by cochlear hair cells. Less expensive but less accurate. The most effective screening is thought to be use of both the above tests in combination.

What are two types of audiometric tests used to screen for hearing impairments in patients?

*ASCA* is positive in about 55% of those with Crohn disease, but are uncommon in ulcerative colitis; conversely, *p-ANCA* is positive in about 70% of patients with ulcerative colitis, but in less than 20% of patients with Crohn disease.

What autoimmune markers are seen in IBD?

Strep pneumo non-typeable H flu Moraxella catarrhalis

What bacterial pathogens often cause acute otitis media (acute infection of the middle ear space)?

mitral valve prolapse

What cardiac abnormality is seen in Ehlers-Danlos?

bicuspid aortic valve coarctation of the aorta aortic root dilation

What cardiac anomalies are associated with Turner syndrome?

coarctation of the aorta bicuspid aortic valve hypoplastic left heart

What cardiac defects are associated with Turner syndrome?

It is an autoimmune complication of URI with GABHS.

What causes rheumatic fever?

A single amino acid substitution of valine for glutamic acid on number 6 position of the beta globin chain of hemoglobin.

What causes sickle cell disease?

*Group B coxsackievirus:* an enterovirus spread by oral fecal transmission. CSF analysis will show mild pleocytosis with lymphocytic predominance and normal protein and glucose.

What causes the majority of cases of viral meningitis in young children?

ecchymoses/petechiae recurrent spontaneous epistaxis prolonged bleeding recurrent hemarthroses DVT, PE, or stroke

What clinical features would suggest abnormal hemostasis?

factor II facotr VII factor IX factor X protein C protein S

What coagulation factors is vitamin K essential for the synthesis of?

hypercalciuria hyperoxaluria distal RTA hyperuricosuria cystinuria UTIs (Proteus mirabilis) hyperparathyroidism

What conditions are associated with kidney stones in children?

sickle cell cystic fibrosis prolonged TPN therapy

What conditions predispose children to getting cholecystitis (inflammation of the gallbladder, normally associated with gallstones)?

Dietary deficiency is unusual, except during early infancy. Pancreatic insufficiency, biliary obstruction, and prolonged diarrhea may result in diminished ability to absorb vitamin K. Medications (cephalosporins, rifampin, isoniazid, warfarin) may interfere with vitamin K metabolism.

What could cause a deficiency of vitamin K?

Weakness or paralysis of the *gluteus medius* and *gluteus minimus* muscles, which are innervated by the superior gluteal nerve.

What creates a Trendelenburg sign?

A small amount of protein is normally present in the urine, but *proteinuria greater than 100 mg/m2/day* is considered pathologic.

What degree of proteinuria is consdiered pathologic?

*Hirschprung disease:* initial evaluation with barium enema and rectal manometry is appropriate. Identification of an aganglionic segment of bowel by punch or suction biopsy can establish the diagnosis. Histo-chemical tissue examination showing increased amounts of acetylcholinesterase in hypertrophic nerve bundles with an absence of ganglia cells is confirmatory. Rectal manometric studies have shown that in aganglionic megacolon, the usual relaxation of the internal rectal sphincter in response to balloon inflation does not occur. Surgery is indicated as soon as the diagnosis is made.

What diagnosis should be suspected in a child with intractable chronic constipation without fecal soiling or diarrhea?

*Serum sickness reaction:* a type III hypersensitivity reaction that occurs 1-2 weeks after penicillin, amoxicillin, cefaclor, or TMP-SMX therapy. Labs will show hypocomplementemia and elevated ESR/CRP, which is consistent with any type III hypersensitivity. Removal of the offending agent is generally sufficient to relieve symptoms within 48 hours.

What disease causes fever, urticaria, and joint pain weeks after treatment with beta-lactam or sulfa antibiotics?

DKA

What disease should always be on your differential when a previously healthy child presents with vomiting?

Parenchymal disease, such as in pneumonia and pulmonary edema

What do crackles and rales suggest?

Intrathoracic obstruction, as in asthma and bronchiolitis

What does expiratory wheezing suggest?

Extrathoracic obstruction, such as in croup and laryngomalacia

What does inspiratory stridor suggest?

Galactose avoidance by discontinuing breastfeeding or milk based formula feeding and using soy or casein hydrolysate infant formula instead. With appropriate diet, many of the features listed can be avoided or reversed.

What early treatment is essential for neonates with suspected galactosemia?

*Hyperkalemia* may result in cardiac dysrhythmias. *Hyperphosphatemia* may result in *hypocalcemia* with tetany. *Hyperuricemia* may result in renal insufficiency.

What electrolyte abnormalities are observed in ALL patients and what complications might they lead to?

ethosuximide

What epilepsy drug is good for absence seizures?

valproic acid phenobarbital

What epilepsy drugs are good for generalized seizures?

carbamazepine phenytoin

What epilepsy drugs are good for partial epilepsy?

During DKA, serum glucose exceeds the threshold for reabsorption, resulting in osmotic diuresis and glucosuria. This diuresis is accompanied by a net renal loss of potassium with depletion of total body potassium stores. However, serum potassium levels may be normal or elevated, since acidemia and decreased insulin cause a potassium shift to the extracellular fluid compartment.

What happens to the body's potassium in DKA?

*RSV* is most common; less common causes include parainfluenza virus, adenovirus, rhinovirus, infleunza, and Mycoplasma pneumoniae

What infectious agents cause bronchiolitis?

*Parainfluenza viruses* are the most common; RSV, rhinovirus, adenovirus, influenza A and B, and Mycoplasma pneumoniae are also responsible.

What infectious agents most commonly cause viral croup?

femoral/skull fractures subdural hematomas (shaken baby) retinal hemorrhage (shaken baby) burns on feet, ankles, buttocks punctate burns (cigarette) any STD in any child not crying in presence of caregiver running from caregiver different injuries at different healing stages

What injuries in a pediatric population should have a high index of suspicion for abuse?

cerebral edema

What is a common complication of DKA?

95-180 beats/min; varies during feeding, sleep, or crying

What is a normal heart rate for a newborn infant?

The goal of initial management is to secure the airway; in children with resp distress, *endotracheal intubation* should be performed in a controlled setting. If attempts at intubation fail, patients may require a tracheotomy.

What is appropriate initial management of *epiglottitis*, which presents with high fever, sudden onset respiratory distress, dysphagia, and drooling?

salfasalazine (for mild disease) corticosteroids (for exacerbations) immunosuppresives (long-term) metronidazole (perianal Crohns) total parenteral nutrition (during flares)

What is appropriate pharmacotherapy for IBD?

1 mL/kilo/hr

What is considered good urine output for children?

*<40-45:* if a baby is hypoglycemic and symptomatic (jittery, tremorous, lethargic, feeds poorly), give 2 mL/kg D10W and recheck. IF persistent, treat with dextrose infusion.

What is considered hypoglycemia for newborns in the immediate post-partum period?

*>60 breaths/min.*

What is considered tachypnea in a newborn?

For mild cases, acetic acid is sufficient to restore the natural acidity of the external auditory canal. For more severe cases, topical antibiotics, sometimes combined with topical corticosteorid, are prescribed. Perforated AOM complicated by OE is treated with both oral and topical abx.

What is effective management for otitis externa?

SSRIs CBT

What is first-line therapy for OCD?

Since there is a small risk of *intussusception* the vaccine is contraindicated in infants with a history of intussusception.

What is one important contraindication to the rotavirus vaccine?

Over time, inappropriate dilution can cause hyponatremia and water intoxication, leading to seizure activity. Slow correction of the sodium is curative.

What is one possible consequence of formula dilution?

*Involvement of two organ systems:* Skin (hives) Pulm (airway edema) CV (hypotension) GI (diarrhea)

What is required for a clinical diagnosis of anaphylaxis?

One or more phenotypic features, or positive family history, or increased immunoreactive trypsinogen on newborn screen. Lab evidence of abnormal CFTR, including sweat chloride >60 mmol/L or two CF mutations or a characteristic ion transport abnormality across the nasal epithelium.

What is required for diagnosis of cystic fibrosis?

<2 years old 2x more common in males 2 inches in length 2 feet from ileocecal valve 2% of population

What is the "rule of 2s" for Meckel's?

low AFP low unconjugated estriol high beta-hCG (low values of all three markers suggests trisomy 18)

What is the "triple marker" which suggests Down syndrome?

*Methylene blue:* acts as an electron acceptor for NADPH and is reduced to leucomethylene blue, which in turn reduces methemoglobin to hemoglobin. High dose vitamin C can be used when methylene blue is unavailable or contraindicated.

What is the antidote for methemoglobinemia?

In asymptomatic patients, they can be observed for up to 24 hours after ingestion; if the patient is symptoamtic or the time of ingestion is unknown, the coin should be removed promptly by flexible endoscopy.

What is the approach to coins ingested and lodged in the upper esophagus in children?

If you see signs of increased ICP, get an urgent *head CT*

What is the appropriate evaluation for a suspected hydrocephalus?

Arthrocentesis, blood and synovial fluid cultures, and then empiric antibiotic therapy

What is the appropriate initial management of septic arthritis?

Treatment consists of a diet that maintains phenylalanine at levels low enough to prevent brain damage but adequate to support normal physical and mental development. Careful supervision of the low-phenylalanine diet and monitoring of blood levels are necessary. Special formulas are available for the infant; older children have difficulty following the diet. It is not clear when and if the diet can be discontinued.

What is the appropriate treatment for PKU?

Biotinidase is the enzyme responsible for breakdown of biocytin (the lysyl precursor of biotin) to free biotin. Deficiency of the enzyme, which is inherited as an autosomal recessive trait, results in malfunctioning of the biotin-dependent mitochondrial enzymes and in organic acidemia. Clinical problems related to the deficiency appear several months or years after birth and include dermatitis, alopecia, ataxia, hypotonia, seizures, developmental delay, deafness, immunodeficiency, and metabolic acidosis. The treatment is lifelong administration of free biotin.

What is the appropriate treatment for biotinidase deficiency?

Treatment consists of prompt elimination of lactose-containing milk from the diet in infancy and, as a more varied diet is introduced, exclusion of foods that contain casein, dry milk solids, whey, or curds.

What is the appropriate treatment for galactosemia?

A diet which consists of careful regulation of the intake of leucine, isoleucine, and valine.

What is the appropriate treatment for maple syrup urine disease?

Phototherapy with a blue light lamp, which converts to direct bilirubin, which can be excreted in the urine. Note that treatment of direct hyperbilirubinemia with phototherapy would turn the child bronze and not add any therapeutic value.

What is the best treatment for neonatal indirect jaundice?

PCR testing of the nasopharynx

What is the best way to confirm a suspected Bordetella pertussis infection?

The best method is sun avoidance, but since this is impractical, broad-spectrum sunscreens with SPF 15-30 or higher should be applied 15-30 minutes before sun exposure and reapplied at least every 2 hours.

What is the best way to prevent sunburn?

1) increased bilirubin production: at birth, RBC concentration is elevated with shorter life span, with high hemoglobin turnover and bilurubin production. 2) low levels of UGT: hepatic uridine diphosphogluconurate glucuronosyltransferase does not reach adult levels until age 2 weeks. 3) enterohepatic recycling is increased as the sterile newborn gut cannot break down biliurubin for fecal excretion.

What is the cause of physiologic jaundice of the newborn, which often occurs on days 2-4 of life and resolves by age 1-2 weeks?

hyperchloremic metabolic acidosis with a normal serum anion gap.

What is the classic electrolyte presentation of renal tubular acidosis?

low extremity palpable purpura lower extremity arthraliga/arthritis abdominal pain/intussusception renal disease (hematuria, proteinuria)

What is the classic tetrad of clinical findings in Henoch Schonlein purpura?

Electrolyte abnormalities and dehydration should be corrected first, and then surgical correction with a partial pyloromyotomy can be performed.

What is the correct management for pyloric stenosis?

*Total parenteral nutrition* allows patients to maintain adequate nutrition. Early enteral feedings are important to ensure adaptive growth of the remaining small bowel, proper hepatic function, and proper oral motor development. With proper management, patients sometimes are weaned off parenteral nutrition.

What is the correct management for short bowel syndrome?

If it's in the esophagus, immediate endoscopic removal is necessary to prevent mucosal damage and esophageal ulceration. Batteries located distal to the esophagus on CXR pass uneventfully in 90% of cases; such patients are observed to confirm excretion of the battery by stool examination and/or radiographic followup.

What is the correct management of a child who has ingested a battery, depending on the location of the battery shown on CXR?

gluten-free diet for life

What is the correct management of celiac disease?

Bag and mask ventilation should *not* be used because this may distend the bowel and increase compression of the lung. Instead, intubation and mechanical ventilation with 100% oxygen should be initiated immediately. Once stabilized, it will require surgical reduction of the hernia and closure of the diaphragmatic defect.

What is the correct management of congenital diaphragmatic hernia?

Radiography of the neck, which may delay definitive treatment, is often avoided. Rather, preparations are made for *immediate intubation* by skilled personnel, and any attempt to visualize the epiglottis should be avoided until these preparations are complete and the child is in a controlled environment.

What is the correct management of epiglottitis?

excision of extra digit; simply tying these appendages off with string will usually leave a nub

What is the correct management of fifth-finger (postaxial) polydactyly in an otherwise healthy black child?

1) *Elemental iron* (4-6 mg/kg/day) PO given with vitamin C to enhance intestinal iron absorption. 2) Dietary counseling to increase nutritional iron. 3) In severe cases, RBC transfusion may be required. 4) Further eval to rule out causes of IDA (polyps, Meckel's, IBD, PUD, early ingestion of whole cow's milk).

What is the correct management of iron deficiency anemia?

Management depends initially on the presence or absence of perforation; if no evidence of free peritoneal air is found, the infant should be put on bowel rest with nasogastric decompression, and systemic antibiotics are initiated. Electrolytes and vital signs should be monitored closely, and serial abdominal films should be performed to evaluate for perforation. If free air is identified on plain radiographs or if the infant clinically worsens with medical management, surgical consultation is required. An exploratory laparotomy is usually performed, and any necrotic intestinal tissue is removed. Occasionally, removal of necrotic gut will result in an infant without adequate intestinal surface area to absorb nutrition, a condition known as short bowel syndrome.

What is the correct management of necrotizing enterocolitis?

lanugo

thin hair that covers the skin of preterm infants; it is minimally present in term infants.

Neonates may require initial management with *PGE* to improve O2 saturation by keeping the ductus patent, or emergent balloon atrial septostomy (Rashkind procedure), and often life saving procedure that increases the size of an ASD or PFO. Definitive repair, however, is the *arterial switch operation,* in which the great arteries are incised above their respective valves and implanted in the opposite root. The coronary arteries are attached to the original aorta, so the coronaries must also be incised and reimplanted.

What is the correct management of transposition of the great vessels?

*Males:* no testicular enlargement by 14 years of age. *Females:* no breast tissue by 13 years of age, or no menarche by 14 years of age.

What is the definition of delayed puberty in males and females?

>6 RBCs detected on three or more consecutive UAs.

What is the definition of microscopic hematuria?

Defined as an increase in RBCs relative to total blood volume, resulting in *Hct > 60%* or a hemoglobin or hematocrit more than two standard deviations above normal values for age.

What is the definition of polycythemia?

<100,000 platelets

What is the definition of thrombocytopenia?

*flexible endoscopy;* rigid endoscopy has a higher risk of esophageal abrasion and perforation and is usually reserved for impaced sharp objects in the proximal esophagus.

What is the diagnostic and treatment method of choice for foreign body ingestion when objects are lodged in the upper esophagus?

*S*erositis (pleuritis/pericarditis) *O*ral or nasal mucocutaneous ulcers *A*rthritis, nonerosive *P*hotosensitivity *B*loody cytopenias *R*enal disease (hematuria, proteinuria) *A*NA positivity *I*mmune markers (dsDNA, anti-Sm) *N*eurologic symptoms (seizures) *M*alar rash *D*iscoid rash Four out of the eleven *SOAP BRAIN MD* criteria provide a sensitivity and specificity of 96%.

What is the diagnostic criteria for SLE?

ultrasound showing an elongated, narrow pyloric channel ("string sign")

What is the diagnostic method of choice for pyloric stenosis?

upper intestinal contrast imaging (showing abnormal position of ligament of Treitz to right of midline, partial or complete duodenal obstruction, and jejunum to right of midline).

What is the diagnostic test of choice for malrotation of the gut with volvulus?

*Cushing syndrome:* excessive glucocorticoid production from adrenal tumors. *Cushing disease:* excessive glucocorticoid production from excessive ACTH production from a pituitary tumor.

What is the difference between Cushing syndrome and Cushing disease?

simple = no LOC complex = LOC

What is the difference between a simple and complex seizure?

*Central hypotonia* is dysfunction of UMNs (ie cortical pyramidal neurons and their descending corticospinal paths). There is an alter LOC and increased DTRs, often with ankle clonus. *Peripheral hypotonia* is dysfunction of LMNs (ie spinal motor neurons and distally to the muscle fibers). No LOC and muscle bulk and DTRs are decreased.

What is the difference between central and peripheral hypotonia?

*Decerebrate posturing* is extension of arms and legs while comatose; it indicates subcortical injury. *Decorticate posturing* is flexion of arms and extension of legs while comatose; it suggests bilateral cortical injury.

What is the difference between decerebrate and decorticate posturing?

*fulminant acute type AIHA* occurs in infatns and young children and is preceded by a respiratory infection. PResents with pallor, jaundice, hemoglobinuria, and splenomegaly. Complete recovery expected. *Prolonged type AIHA* is characterized by a protracted course and high mortality; underlying disease is frequently present.

What is the difference between fulminant and prolonged AIHA?

*Hypotonia* is decreased resistance of movement during passive stretching of muscles. *Weakness* is decreased normal force generated by active contraction of muscles.

What is the difference between hypotonia and weakness?

While both are caused by abuse or neglect in young children, RAD results in a child who avoids parental figures, while DSED results in overfamiliarity and unhesitant approaches to unfamiliar adults.

What is the difference between reactive attachment disorder and disinhibited social engagement disorder?

*Crigler Najjar type I* is an autosomal recessive disorder in which almost 100% of UDP glucuronyl transferase enzyme activity is absent. Kernicterus occurs almost universally. *Crigler Najjar type II* is an autosomal dominant disorder in which 90% of the enzyme activity is absent, and there is lower likelihood of kernicterus.

What is the difference between type I and type II Crigler Najjar?

Urticarial or erythematous maculopapular lesions which progress to petechiae and palpable purpuric lesions concentrated on the buttocks and lower extremities.

What is the distinct skin lesion seen in the IgA vasculitis Henoch Schonlein purpura?

Topical antifungals such as clotrimazole or terbinafine

What is the first line treatment for tinea corporis, a superficial fungal infection characterized by a scaly, erythematous, pruritic rash with a raised border and central clearing?

*Endotracheal intubation;* do not attempt bag and mask ventilation as it can exacerbate respiratory decline. In addition, a gastric tube should be placed to decompress the stomach and bowel.

What is the first step in management of a newborn with respiratory compromise and suspected congenital diaphragmatic hernia?

Topical antifungals like terbinafine or clotrimazole are first-line

What is the first-line treatment for tinea corporis?

genetic testing showing deletion of the dystrophin gene on Xp21

What is the gold standard for diagnosis of Duchenne muscular dystrophy? REcall that DMD presents with bilateral calf pseudohypertrophy and Gower sign in infants age 2-5. Serum creatine phosphokinase and aldolase levels are elevated in screening tests.

CT brain

What is the initial method of evaluation for a child with increasing head circumference and signs of increased ICP?

*Infection* as a result of decreased splenic function, putting patients at risk for encapsulated bacteria (Hib, Strep penumo, N meningitidis, Salmonella). Osteomyelitis may occur and mimic a painful bone crisis.

What is the leading cause of death in sickle cell disease patients?

Volvulus is a surgical emergency requiring immediate exploration, untwisting of the gut, resection of nonviable segments, and fixation of the gut to prevent recurrence. Fluid resuscitation, NG suctioning, and broad-spectrum parenteral abx should also be given. TPN may be required if large segments of bowel are resected.

What is the management of malrotation of the gut with volvulus?

Allergic rhinitis is an *IgE mediated* inflammatory response in the nasal mucosa to inhaled antigens. Sensitization to airborne allergens induce IgE formation; allergen specific IgE binds to receptors on mast cells and basophils in the nasal mucosa. Subsequent exposure produces an IgE-mediated inflammatory response within minutes; mast cells degranulate and release histamine, leukotrienes, kinins, and prostaglandins. Seasonal rhinitis occurs in specific seasons in response to tree, grass, or weed pollens. Perennial rhinitis occurs all year long in response to indoor allergens like dust mites and animal dander.

What is the mechanism by which allergic rhinitis occurs?

Anaphylaxis is a life-threatening acute *systemic IgE mediated reaction.* Antigen binding to IgE on the surface of mast cells and basophils results in the release of potent mediators that affect vascular tone and bonchial reactivity.

What is the mechanism by which anaphylaxis occurs?

*24 hour urinary protein collection*, with normal < 100 mg/m2/day, is the most accurate method, but is very difficult to obtain in children. Instead, a *random spot urine total protein to creatinine ratio* is usually performed. Normal urine TP:CR ratio is <0.5 for infants 6-24 months and <0.2 for children over 2 years.

What is the most accurate method of detecting proteinuria? What is a more practical method to use for children?

*Voluntary holding:* whether to avoid pain or simply due to embarrassment, kids hold it in, water gets taken out, and it hardens. Disimpaction may be necessary. Make sure to teach child that voluntary holding is dangerous.

What is the most common cause of constipation in pediatric patients?

Inorganic failure to thrive, eg a distubed parent-child bond that results in inadequate caloric intake or retention.

What is the most common cause of failure to thrive?

*Inadequate store of nutrients:* glycogen and fat stores are diminished in premature infants and those who are small for gestational age. Energy stores are inadequate to meet the energy demands after the maternal supply of glucose is interrupted at birth, and hypoglycemia ensues.

What is the most common cause of hypoglycemia in premature SGA infants?

polyarthritis

What is the most common finding in rheumatic fever?

*abdominal ultrasound* note that abdominal CT is useful for diagnosing complications, such as pseudocyst, abscess, or necrosis.

What is the most common method used for diagnosis and monitoring of acute pancreatitis?

Staph aureus

What is the most common pathogen to cause osteomyelitis?

viral URI

What is the most common predisposing factor for acute bacterial sinusitis?

*Type C:* proximal esophagus is blind, distal esophagus has an aberrant connection from the trachea to the stomach.

What is the most common type of tracheoesophageal fistula?

*Allergen skin testing* via prick or intradermal testing; skin tests using purified allergens can diagnose allergic rhintiis. To avoid false negatives, patients should discontinue antihistamine use 4-7 days before skin testing.

What is the most effective way to definitively diagnose allergic rhinitis?

*Pneumatic otoscopy* can identify abnormal movement of the tympanic membrane, and therefore fluid within the middle ear. In contrast, observing erythema and loss of tympanic membrane landmarks are unreliable methods of identifying fluid within the middle ear space.

What is the most reliable method of diagnosing acute otitis media (AOM)?

Peaks at age 8-9 months and resolves by age 2 years.

What is the normal timeframe for stranger anxiety?

The intestines normally return to the abdomen through the umbilical cord during the 10th week of gestation and undergo counterclockwise rotation about the axis of the superior mesenteric artery. The intestines are then fixed to the abdominal wall. Malrotation occurs when normal bowel rotation is interrupted. Lack of fixation of the small bowel results in *Ladd's bands* that can compress the duodenum, causing mechanical obstruction. Also, the narrow pedicle, which suspends the small bowel and base of the superior mesenterics, can easily twist, leading to ischemia and midgut volvulus.

What is the pathogenesis of malrotation of the gut?

Calculous cholecystitis is caused by obstruction of the cystic duct causing increased intraluminal pressure and distention, increased secretion of enzymes and prostaglandins, and progressiv einflammation. Infection, necrosis, and perforations may all occur. Acute acalculous cholecystitis is usually caused by infection (Salmonella, Shigella, E col) but also by abdominal trauma, burns, or vasculitis.

What is the pathophysiology and etiology of cholecystitis?

Bacteria are introduced into the blood, either spontaneously or during an invasive procedure. Bacteria then infect injured cardiac endothelium. Fibrin and platelets adhere to the injury site, creating a vegetation which may induce valve incompetency. Embolic phenomena may occur.

What is the pathophysiology of infective endocarditis?

prematurity

What is the primary risk factor for intraventricular hemorrhage, which presents with anemia, tachycardia, apnea, seizures, and bulging fontanelles?

In these cases, underlying pathology is unlikely and no further evaluation or treatment is required. Patients can be discharged home with education about seizure precautions and supportive care for concurrent infection.

What is the proper management of a simple febrile seizure (eg one that is generalized, lasts under 15 minutes, and does not recur within 24 hours)?

*Topical antibiotics,* such as sulfacetamide, polymyxin B, and trimethoprim sulfate, or gentamicin, tobramycin, and erythramycin.

What is the proper management of bacterial conjunctivitis?

Reassure parents that they will resolve without harm, and counsel them to not undertake potentially harmful resuscitation efforts.

What is the proper management of cyanotic or pallid spells, breath-holding spells precipitated by frustration or fright which occur in young-children?

Oral macrolide therapy (eg erythromycin), as topical therapy has high failure rates.

What is the proper treatment for chlamydial conjunctivitis, which occurs in newborns age 5-14 days with mild eyelid swelling, chemosis, and watery or mucopurulent discharge?

Diagnosis requires evidence of recent strep infection and either two major criteria or one major plus two minor criteria

What is the required by the Jones criteria for diagnosis of rheumatic fever?

*IVIG* is the preferred treatment for children because of its relative safety and ease of use. The mechanism of action of IVIG is unkown. *Plasmapheresis* removes anti-myelin antibodies and is performed over 4-5 day period.

What is the treamtent for Guillain Barre syndrome?

Mild cases (no inspiratory stridor at rest) should be treated with a single dose of steroids (dexamethasone) to decrease airway edema. Patients with moderate to severe croup (respiratory distress, stridor at rest) should be treated with corticosteroids plus nebulized epinephrine, which decreases airway edema and reduces secretions.

What is the treatment for *croup*, a viral illness that causes laryngeal and tracheal inflammation leading to subglottic edema and narrowing which present as a barking cough with stridor?

albendazole pyrantel pamoate (for pregnant patients) The patient and all household contacts should be treated, as the infection is highly contagious.

What is the treatment for Enterobius vermicularis, which causes nocturnal perianal pruritus and can be diagnosed via scotch tape testing?

steroids

What is the treatment for HSP?

High-dose (2 g/kg) *IVIG* in combination with *aspirin* initiated wtihin 10 days of fever decreases prevalence of CAA. In the acute phase, use high-dose aspirin for its anti-inflammatory effect; in the subacute phase, use low-dose aspirin for its antiplatelet effect.

What is the treatment for Kawasaki disease?

1) supplemental O2 2) CPAP 3) mechanical ventilation 4) exogenous surfactant

What is the treatment for RDS?

urgent stem cell transplant

What is the treatment for SCID?

First line treatment is *amoxicillin*; if it recurs, use *amoxicillin-clavulanate*. If it re-recurs (three times in 6 months or four times in 12 months), ear tubes (tympanoplasty) are indicated. If a patient has a penicillin allergy, try a cephalosporin (like cedinir) or azithromycin.

What is the treatment for acute otitis media?

metronidazole

What is the treatment for amebiasis (infection with Entamoeba histolytica)?

Closure by open heart surgery to prevent right-sided heart failure, pulmonary hypertension, atrial dysrhythmias, and paradoxic emboli. Some centers close ASDs using interventional catheterization procedures.

What is the treatment for an ASD?

1) avoidance of triggers 2) low-to-medium potency steroids 3) antihistamines (to decrease itching) 4) baths in tepid water

What is the treatment for atopic dermatitis?

*Corticosteroids* are used for initial control, and immunosuppressives like *azathioprine* or *6-mercaptopurine* are also used. In severe disease, transplant is indicated.

What is the treatment for autoimmune hepatitis?

Increase the frequency and duration of feeds to stimulate milk production, maintain adequate hydration, and promote bilirubin excretion. Neonates should breastfeed approximately every 2-3 hours for >10-20 minutes per breast during the first month of life. IF bilirubin continues to rise, formula supplementation may be necessary.

What is the treatment for breastfeeding jaundice?

A *Pavlik harness* holds the head of the femur against the acetabulum. It's typically used 2-3 months, if the diagnosis is made by 6 weeks of age. If the diagnosis is made later than that, surgery may be required.

What is the treatment for developmental dysplasia of the hip?

*amoxicillin* because doxycycline can slow bone growth in exposed fetuses

What is the treatment for early localized Lyme disease in children under 8?

moisturizers and topical steroids

What is the treatment for eczema, a chronic dermatitis which presents as a pruritic rash in the flexural areas and face in infancy?

IV antimicrobial therapy is directed against the identified organism; treatment for 4-6 weeks is required. Because endocarditis is rarely a medical emergency, therapy may be safely withheld until an adequate number of blood cultures are obtained.

What is the treatment for endocarditis?

*Intubation* and antibiotic therapy, typically with a second- or third-gen ceph IV. IF epiglottitis is secondary to Hib infection, rifampin prophylaxis is also indicated for unimmunized household contacts younger than 4.

What is the treatment for epiglottitis, a medical emergency?

rehydrate with IV fluids and give aggressive cooling with cold water immersion

What is the treatment for heat stroke, which presents with hyperthermia with hot skin, profuse sweating, and mental status changes in young athletes?

treat with vancomycin

What is the treatment for lung empyema, which is characterized on CXR by unilateral white out of the lung field due to pleural or extrapleural fluid collection?

If bacterial, appropriate antibiotics should be given. If viral or postpericardiotomy, give anti-inflammatories like aspirin or steroids. Drainage via placement of a pericardial catheter or surgical window may be indicated.

What is the treatment for pericarditis?

Prolonged vomiting will cause a hypochlormeic, hypokalemic metabolic alkalosis; treatment consists of *IV rehydration* and *normalization of electrolytes* prior to *pyloromyotomy* to decrease risk of postop apnea.

What is the treatment for pyloric stenosis?

First line treatment is topical 5% permethrin or oral ivermectin.

What is the treatment for scabies, which will present as an intensely pruritic rash located in the axillae, periumbilical area, genitalia, and web spaces of the hands and feet?

*Phlebotomy* is used to keep the hematocrit under 60% until the underlying cause is treated or resolves.

What is the treatment for secondary polycythemia?

The deviated eye can be strengthened by patching the normal eye (occlusion therapy) or blurring the vision of the normal eye with cycloplegic drops (penalization therapy). Prescription eyeglasses can be given for correction of significant refractive errors, and use of the deviated eye should be promoted.

What is the treatment for strabismus (ocular misalignment)?

vWD is the most common heritable bleeding disorder, with some studies suggesting prevalence of 1% to 2% in the general population. vWF participates in clot formation by adhering to areas of vascular damage and causing platelets to attach and activate. About 85% of cases of vWD are type I, resulting from decreased production of normal vWF. Several variants of type II vWD are described, with abnormal or dysfunctional vWF the etiology. Patients with type III, the most rare, have undetectable levels of vWF. In type I, *desmopressin* alone can transiently increase the levels of vWF three- to fivefold, so it is frequently used for acute bleeding episodes.

What is the treatment for type I von Willebrand disease?

Remove the offending allergen if possible and administer second-gen H1 antihistamines (*cetirizine, loratidine, fexofenadine*)

What is the treatment for urticaria?

Administration of vitamin K; IM administration of vitamin K after birth prevents hemorrhagic disease of the newborn. In severe disease, FFP may be needed.

What is the treatment for vitamin K deficiency?

*DDVAP* induces vWf release from endothelial cells and is used for mild to moderate bleeding and for prophylaxis before surgery. *Cryoprecipitate,* which contains intact vWf, may be used for serious bleeding, extensive surgeries, or severe disease.

What is the treatment for von Willebrand disease?

Oral antibiotics like amoxicillin + clavulanate

What is the treatment from bacterial sinusitis?

*Physical therapy* and *restriction of vigorous exercise* are effective treamtents; if more than 50% damage to the femoral head, surgery may be indicated.

What is the treatment of Legg Calves Perthes disease?

The majority of patients respond rapidly to *Corticosteroid* therapy; 90% of children have complete remission with the disappearance of proteinuria. Therefore, kidney biopsy is seldom done, only eprformed when there is steorid-resistant or progressive disease, or when older children or adolescents present with MCD symptoms to exclude other causes of nephrotic syndrome.

What is the treatment of choice for minimal change disease?

Most cases occur before age 2, following a viral illness. The resultant hypertrophy of the Peyer patches in the lymphoid rich terminal ileum serves as a nidus for telescoping. A pathological lead point (eg a Meckel diverticulum) should be suspected in older children with recurrent intussusception.

What is the trigger for intussusception?

*Topical mupirocin* can be used, or oral antibiotics like dicloxacillin, cephalexin, or clindamycin.

What is the typical treatment for impetigo, which is most often caused by Staph aureus or Strep pyogenes?

Physical exam of the neonate is notable for facial diplegia (bilateral weakness), hypotonia, areflexia, and mutliple joint contractures. Myotonia develops by 5 years of age but is not present in the newborn. In adulthood, myotonic facies, ptosis, stiff straight smile, and inability to release grip after hand shaking are all characteristic.

What is the usual clinical presentation of congenital myotonic dystrophy?

inhaled epinephrine and steroids

What is the usual therapy for croup?

Ixodes scapularis (deer tick)

What is the vector for Borrelia burgdorferi, the spirochete which causes Lyme disease?

endotracheal intubation

What is your *first priority* in anaphylaxis where there is potential airway compromise due to airway edema?

hemoglobinuria bite cells Heinz bodies

What lab findings are seen in G6PD deficiency?

severe normocytic anemia spherocytosis on smear prominent reticulocytosis leukocytosis direct Coombs test positive

What lab findings are seen in autoimmune hemolytic anemia?

anion gap metabolic acidosis hyperglycemia/glucosuria ketonemia/ketonuria hyperkalemia

What lab findings will be seen in DKA?

CBC: anemia, leukocytosis ESR: elevated Antineutrophil cytoplasmic antibody positive = UC Anti-Saccharomyces cerevisiae antibody positive = Crohn's

What lab values will be seen in IBD?

Splinter hemorrhages under nails Retinal hemorrhages Osler's nodes Janeway lesions Roth's spots in retina Splenomegaly Hematuria

What lesions are indicative of bacterial endocarditis?

GBS HIV Hep B

What maternal labs should be obtained due to potential changes in immediate post-partum management of an infant?

lithium cyclosporin antineoplastic agents cocaine heroin amphetamines ergotamines bromocriptine

What maternal medications are a contradiction to breastfeeding?

haloperidol

What medication do you use to treat Sydenham's chorea?

prostaglandin E1

What medication should you administer if you want to maintain a patent ductus arteriosus in an infant with a severe congenital heart condition?

sulfadiazine pyrimethamine

What medications can be used to treat Toxoplasmosis?

hydroxyurea daily oral penicillin daily folic acid

What medications do sickle cell patients take as prophylais and preventive care?

*Desmopressin (DDAVP)* *Imipramine (a TCA)*

What meds are useful in treating nocturnal enuresis? Recall that medications, if used alone, result in frequent relapses; be efficacy uses a combination of alarms, behavioral modification, and meds

lifts head up prone

What motor milestones should a child have by 1 months?

stands has pincer grasp

What motor milestones should a child have by 10 months?

cruising taking first steps points to objects

What motor milestones should a child have by 12 months?

walking climbs stairs

What motor milestones should a child have by 18 months?

kicks ball

What motor milestones should a child have by 2 years?

feeds self with fork and spoon

What motor milestones should a child have by 20 months?

rides a tricycle

What motor milestones should a child have by 3 years?

copies lines and circles draws stick figures hops on one foot

What motor milestones should a child have by 4 years?

uses buttons and zippers grooms self

What motor milestones should a child have by 5 years?

rolls and sits passes toys hand to hand

What motor milestones should a child have by 6 months?

crawls

What motor milestones should a child have by 8 months?

Fat soluble vitamins (ADEK) due to pancreatic insufficiency

What nutritional deficiencies are present in cystic fibrosis patients?

Nontypeable H flu Strep pneumo Moraxella catarrhalis Staph aureus

What organisms most commonly cause bacterial conjunctivitis?

Staph aureus Strep pyogenes Salmonella (sickle cell) Pseudomonas (nail with sneakers)

What organisms most commonly cause osteomyelitis?

grade I or II decreases with standing early of mid systolic

What parameters usually define a heart murmur as benign, which can be managed with observation and reassurance?

E coli Klebsiella Pseudomonas Staph saprophyticus Serratia Proteus

What pathogens cause UTIs?

Staph aureus Pseudomonas aeruginosa Burkholderia cepacia

What pathogens typically cause pneumonia in cystic fibrosis patients?

*neonatal hypoglycemia* macrosomia birth injury

What perinatal complications are common in infants of diabetic mothers who experience fetal hyperglycemia after the first trimester?

*Increased right ventricular impulse* as a result of right ventricular overload, *systolic ejection murmur* heard best at the upper left sternal borders, and a *fixed-split second heart sound.*

What physical exam findings are associated with an ASD?

Rear facing car seats from 0-2 years; booster seat until 4'9" and 8-12 years old; seat belts for everyone.

What precautions should be taken for car safety for kids?

Zidovudine for 6 weeks TMP/SMX (PCP prophylaxis) No breastfeeding Urine CMV culture to detect coinfection

What precautions should you take in infants of HIV+ mothers until HIV DNA PCR at age 4 months is negative?

IM vitamin K (prevent hemorrhage) hep B vaccine conjunctivitis prophylaxis

What prophylactic medications should be given to newborns in the first hours of life?

<7 mg/dL

What serum IgA level defines IgA deficiency?

bone age evaluation

What should always be a part of the initial evaluation of precocious puberty?

*imperforate anus* *meconium ileus* *Hirschprung's disease* bilious emesis maternal magnesium maternal potassium

What should be on your differential for failure to pass meconium? Recall that there should be concern for failure to pass meconium if a newborn has not passed stool by *24 hours* of life.

macrolides for everyone, regardless of age, immunization status, or symptoms

What should be prophylaxis for all close household contacts of a child that has pertussis?

audiological evaluation

What should be your first step in assessing apparently delayed verbal milestones?

dental procedures

What should children with endocarditis be given prophylactic antibiotics for?

Physicians must respect parents' medical decisions for their children, with the exception of refusal of life-saving treatment. Physicians must counsel parents about the health risks of refusing vaccination and document the discussion in the medical chart. In addition, physicians should be aware of the vaccination exemption laws in their states. However, the physician must respect the parent's decision if they have informed them of the risks, benefits, and potential consequences. Make sure to fully document in the medical record.

What should you do if a child's parent refuses vaccines?

Parents cannot refuse life-saving treatments for their children. In a non-emergency situation, the physician should seek a court order to proceed with the necessary intervention if the parents do not listen to reason. In an emergency situation, emergency treatment is legally authorized regardless of the wishes of the parents.

What should you do if a parent refuses to consent to a life-saving treatment for a child (eg chemotherapy)?

First, assess airway, breathing, circulation. Then, remove contaminated clothing. After that, upper GI endoscopy is the diagnostic study of choice to evaluate extent of injury. Do not attempt to neutralize the alkali with vinegar or lavage, as these interventions may trigger vomiting to cause further mucosal damage.

What should you do to manage caustic substance ingestion?

1) back to sleep 2) no bed sharing 3) hard (not fluffy) cribs 4) flatten occiput 5) smoking cessation

What should you emphasize as good techniques to prevent SIDS to parents?

*Twin-to-twin transfusions* occur in about 15% of monochorionic twins and commonly cause intrauterine death. This disorder should be suspected when the hematocrits of twins differ by more than 15 mg/dL. The donor twin is likely to have oligohydramnios, anemia, and hypovolemia with evidence of shock if the hematocrit is significantly reduced; the recipient twin is likely to have polyhydramnios and plethora and to be larger than the donor twin. A 20% difference in body weight may result. As the central venous hematocrit rises above 65%, infants can develop hyperviscosity, respiratory distress, hyperbilirubinemia, hypocalcemia, renal vein thrombosis, congestive heart failure, and convulsions.

What should you suspect when the hematocrits of twins differ by more than 15 mg/dL?

nuchal rigidity Kernig sign Brudzinski sign

What signs of meningitis are typically not present in a neonate with meningitis?

parallel play (24-36 months) rapprochement (24 months) gender realization (36 months)

What social milestones should a child have from 12-36 months of life?

They should comfortably spend part of the day away from mother by 3 years, and have cooperative play and imaginary friends by 4 years.

What social milestones should a child have from years 3-5 of life?

social smile = 2 months stranger anxiety = 6 months separation anxiety = 9 months

What social milestones should a child have in the first year of life?

CT with contrast

What study is often recommended in cases of bacterial meningitis to evaluate for brain abscesses, especially for patients with focal neurologic findings?

100.4 F (38 C) or above

What temperature is considered a fever in children?

*valproate* phenytoin colchicine vincristine azathioprine methotrexate

What teratogens can cause spina bifida?

Vagal maneuvers (Valsalva, ice pack on face, unilateral carotid massage, placing child upside down, orbital pressure) may all convert SVT into sinus rhythm. IV adenosine is the primary medication used for acute conversion to sinus rhythm. Synchronized cardioversion may be used in hemodynamically unstable patients. Chronic medical management is through digoxin or propanolol (do not use digoxin in WPW). Radiofrequency catheter ablation may be used to destroy accessory pathway in chronic SVT.

What treatments are indicated for SVT?

hydrocephalus intracranial calcifications chorioretinitis

What triad of symptoms is seen in congenital toxoplasmosis?

*Hypogonadotropic hypogonadism:* because of inactivity of the hypothalamus and pituitary gland, these patients have low FSH, low LH, and, in turn, low testosterone and low estradiol, with a flat GnRH stimulation test. *Hypergonadotropic hypogonadism:* because of end-organ dysfunction (ie gonadal failure), these patients have high FSH and high LH levels with low testosterone or low estradiol levels. There is no abnormality in the hypothalamus or pituitary gland.

What two categories of disorders might result in delayed puberty?

LABA

What type of asthma medication should you never use on its own?

normal (0.9%) saline

What type of fluid should be used for initial resuscitation in severe hypovolemic hypernatremia?

*SCD:* 0% HbA; 85-95% HbS, 5-15% HbF *SCT:* 50-60% HbA; 35-45% HbS; <2% HbF Note that SCD patients on hydroxyurea will have higher HbF concentrations, often >15%, as hydroxyurea works by increasing the amount of HbF to dilute the amount of HbS.

What types of hemoglobin are seen in patients with sickle cell disease vs sickle cell trait?

HepB

What vaccine should children receive at birth?

MMR HepA DTaP Hib PCV13 Varicella (MAD HPV)

What vaccines should children receive at 1-1.5 years?

TdaP HPV Meningococcal (Tada! Human Men!)

What vaccines should children receive at 11-12 years?

meningitis booster

What vaccines should children receive at 16-18 years?

HepB (2nd dose) DTaP Rotavirus (RV1) Hib IPV (Inactivated polio) PCV13 (Pneumococcal conjugate) (2 B DR HIP)

What vaccines should children receive at 2 months?

DTaP Rotavirus (RV1) Hib IPV (Inactivated polio) PCV13 (Pneumococcal conjugate) (4 DR HIP)

What vaccines should children receive at 4 months?

Varicella DTaP IPV MMR (Very DIM)

What vaccines should children receive at 4-6 years?

HepB (3rd dose) DTaP Rotavirus (RV1) Hib IPV (Inactivated polio) PCV13 (Pneumococcal conjugate) Influenza (B DR HIP In 6 months)

What vaccines should children receive at 6 months?

There will be intestinal distention with minimal air-fluid levels. Air remains trapped in the meconium; thus, there is no definite air-fluid interface. Fine gas bubbles may be seen mixed with meconium, giving a characteristic *soap bubble* appearance.

What will be seen on abdominal radiographs with meconium ileus?

Air in the stomach and proximal duodenum, the "double bubble" sign

What will you see in abdominal radiography of a patient with duodenal atresia?

1) *Head CT* to rule out acute CNS injury or congenital malformation 2) *Serum electrolytes*, calcium and magnesium, and ammonia, lactate, and pyruvate levels to rule out metabolic disorders. 3) If genetic disorders are suspect (eg Prader Willi), high-resolution chromosome studies and fluorescent in situ hybridization should be ordered.

What workup is necessary when you suspect central hypotonia in a newborn?

1) *serum CK levels* 2) DNA tests for spinal muscular atrophy 3) *EMG* and nerve conduction studies 4) *Muscle biopsy*

What workup should you consider when you suspect peripheral hypotonia in an infant?

Strict vegan diet Juvenile pernicious anemia Crohn's disease

What would cause a B12 deficiency leading to a macrocytic anemia?

Decreased folic acid intake (diet lacking uncooked fresh fruits and vegetables or from exclusive feedings with goat's milk as the sole source of milk protein) or decreased intestinal absorption of folic acid (from diseases affecting the small intestine, like celiac disease, chronic infectious enteritis, Crohn's disease, or meds like anticonvulsants and OCPs.)

What would cause a folic acid deficiency leading to macrocytic anemia in a child?

*Simple:* lasts less than 15 minutes, is generalized. *Complex:* lasts more than 15 minutes, has focal features, recurs within 24 hours.

What's the difference between a simple and a complex febrile seizure?

Recommended at age *9-11* and again at age *17-21*, as lipid levels are relatively stable just prior to and after puberty. Screening outside these periods should occur in patients at high risk for CV disease (eg history of obesity, DM, tobacco exposure, family history of premature coronary disease) and men over 35.

When are pediatric screenings for dyslipidemia recommended (eg when should you get a lipid panel for a kid)?

From 9-15 years of age, with a mean onset of 12.5 years.

When does menarche typically occur

9 months

When does object permanence develop?

*29 months*, but there is a wide range of normal from 16 to 48 months. The average child will achieve bladder control by 32 months, also with a wide range of normal of 18-60 months.

When does the average child achieve bowel control (toilet training)?

The umbilical cord typically separates from a newborn 10 to 14 days after birth, although some will remain for 3-4 weeks. An intact cord after 1 month of age is considered "delayed separation." *Leukocyte adhesion deficiency* type 1 (LAD-1) has been described with delayed cord separation.

When does the umbilical cord normally separate from a newborn? What does delayed separation suggest?

girls age <8 boys age <9

When is development of secondary sexual characteristics considered premature?

6-18 months

When is separation anxiety common?

When *anaphylaxis* develop following DTaP administration, or when *unstable neruologic conditions* like infantile spasms or uncontrolled epilepsy or encephalopathy occur within a week of DTaP administration. Note that short (<5 minutes) self-limited seizures are not a contraindication.

When is the DTaP vaccine contraindicated?

once between 9-15 months again between 4-6 years

When is universal screening of hemoglobin levels recommended?

It is performed routinely at age 4, but can be performed as early as age 3 in cooperative children. Visual acuity worse than 20/40 at age 4 or worse than 20/30 at age 5+ should prompt further investigation.

When should formal visual acuity testing begin? Note that once it begins, vision should be evaluated by Snellen chart at every well-child visit.

These patients are at increased risk for optic pathway gliomas, which can present with headache and decreased visual acuity; NF1 patients with these symptoms should get an MRI to evaluate for an intracranial mass.

When should patients with neurofibromatosis type 1 receive MRI of the brain and orbits?

Surgical closure is indicated immediately if there is heart failure refractory to medical management as a result of the VSD. Large VSDs with pulmonary hypertension are usually closed at 3-6 months of age, while small to moderate VSDs are usually closed between 2 and 6 years of age.

When should you surgically close a VSD?

Enterohemorrhagic Escherichia coli are Shiga-like toxin producing pathogens found in poorly cooked beef, and some have been responsible for outbreaks of bloody diarrhea that were well-publicized in the media. These organisms secrete shiga toxin. Routine stool cultures do not distinguish this particular pathogen from other E. coli; the laboratory must use sorbitol-MacConkey agar to isolate the bacteria. Enzyme assays for shiga toxin are becoming available as well.

When would you use sorbitol-MacConkey agar?

Acidic refluxate induces bronchopulmonary constriction and can also lead to frank aspiration or microaspiration. Common signs and symptoms include chronic laryngitis, hoarseness, wheezing, vocal cord nodules, and subglottic stenosis

Why can upper and lower airway disease occur as a sequelae of GERD?

The knee to chest position in an infant increases venous return the heart and increases SVR, thereby decreasing right-to-left shunt and pumping more blood to the lungs, decreasing hypoxia

Why do children with tet of Fallot squat in response to tet spells?

immaturity of the hypothalamic pituitary ovarian axis; treatment with progestin or OCPs will help regulate menses

Why, in adolescents, can the majority of menstrual cycles during the first few years post menarche be anovulatory and present as painless, irregular, heavy bleeding?

meconium ileus

With this GI disease, associated with CF, echogenic bowel on prenatal ultrasound can be an early hint. Inspissated meconium obstructs the small bowel, usually at the level of the terminal ileum. Radiographs show dilated loops of bowel, and usually reveal a bubbly or granular pattern at the level of obstruction. The enema shows microcolon from disuse.

transposition of the great arteries

With this congenital heart defect, adequate saturation can only be achieved by shunting blood through a patent foramen ovale, ASD, VSD, or PDA. Without these accompanying defects, life is not compatible with this defect.

Administer hepatitis B immunoglobulin and hepatitis B vaccine to the infant.

You are advised by the on-call obstetrician that the mother of a baby she has just delivered has chronic hepatitis B (HBsAg-positive). What is the most appropriate course of action?

*Random orientation of cilia,* since this patient likely has *primary ciliary dyskinesia.*

You are asked by a colleague to evaluate a 5-year-old boy as a second opinion. He has a history of chronic and recurrent upper respiratory tract infections, several admissions to the hospital for pneumonia, and three surgeries for PE tubes for chronic otitis media. Of note is a right-sided heart on repeated radiographs. Convinced you know the diagnosis based on history alone, you confirm your diagnosis with a biopsy of the nasal mucosa. What do you expect to find?

This child likely has *idiopathic pulmonary hemosiderosis (IPH)*. While fever, respiratory distress, and localized chest radiograph findings should point initially toward an acute pneumonia, the history of recurrence, the rapid clearing of radiographic findings, and the hemoptysis suggest pulmonary hemorrhage. The examination finding of digital clubbing suggests a chronic process. Other typical findings would include microcytic and hypochromic anemia, low serum iron levels, and occult blood in the stool (from swallowed pulmonary secretions). *Bronchoalveolar lavage* will reveal hemosiderin-laden macrophages and would be most likely to make the diagnosis. A distinct subset of patients with pulmonary hemosiderosis has hypersensitivity to cow's milk (the association is called Heiner syndrome) and may improve with a diet free of cow's milk products.

You are asked to evaluate a 4-year-old boy admitted to your local children's hospital with a diagnosis of pneumonia. The parents state that the child has had multiple, intermittent episodes of fever and respiratory difficulty over the past 2 years, including cyanosis, wheezing, and dyspnea; each episode lasts for about 3 days. During each event he has a small amount of hemoptysis, is diagnosed with left lower lobe pneumonia, and improves upon treatment. Repeat radiographs done several days after each event are reportedly normal. Rheumatologic and immunologic evaluations were normal during the prior admission. His examination on the current admission is significant for findings similar to those described above, as well as digital clubbing. What is the diagnosis and appropriate intervention?

Infants born to mothers with gestational diabetes are at risk for being large for their gestational age and thus at increased risk for peripheral nerve injuries such as *Erb-Duchenne and phrenic nerve paralysis.* An *ultrasound or fluoroscopy of the chest* would reveal asymmetric diaphragmatic motion in a seesaw manner.

You are asked to evaluate an infant born vaginally 3 hours previously to a mother whose only pregnancy complication was poorly controlled insulin dependent gestational diabetes. The nursing staff noticed that the infant was breathing abnormally. On examination, you find that the infant is cyanotic, has irregular, labored breathing, and has decreased breath sounds on the right side. You also note decreased tone in the right arm. You provide oxygen and order a stat portable chest radiograph, which is normal. What is the likely diagnosis and how would you confirm it?

*Neonatal lupus* is a rare manifestation of transferred maternal IgG autoantibodies. Infants can have thrombocytopenia, neutropenia, rash, liver dysfunction, and a congenital heart block. Most manifestations are self-resolved; however, the congenital heart block is permanent and frequently requires pacing.

You are called to the nursery to evaluate a newborn. The mother has a history of systemic lupus erythematosus and gestational diabetes. The nurses are concerned because the baby has developed petechiae and bruising after his bath. Vital signs have been stable, with a heart rate of 62 beats per minute and a respiratory rate of 40 breaths per minute. You note a large liver, scattered petechiae, and an erythematous rash on the cheeks and on the bridge of the nose. What is the diagnosis?

The child has a subgaleal or subaponeurotic hemorrhage, which can be life threatening; infants may lose a third or more of their blood volume into this potential space, leading to hypovolemic shock. A subgaleal hemorrhage will feel like a cephalohematoma that crosses the midline, but rapidly expands and can have cardiovascular complications. Dependent pooling of blood in the occipital area of the scalp may be observed when the infant is lying on his back. Careful monitoring is essential. Although some infants require fluid resuscitation, observation alone in this case may be appropriate, but should be accomplished in an ICU setting.

You are examining a newborn infant in the well-baby nursery. The infant was the product of a benign pregnancy and vaginal delivery; he appears to be in no distress. Interestingly, your measurement of fronto-occipital head circumference is about 2 cm larger than the initial measurement done several hours before. Your examination otherwise is significant for tachycardia and a "squishy" feel to the entire scalp. You can elicit a fluid wave over the scalp. What is the diagnosis and management?

*Admit the child to the hospital.* Fever, cough, and tachypnea in a patient with sickle-cell anemia can be manifestations of pneumonia, pulmonary vaso-occlusive crisis, or sepsis. Aside from being relatively common in patients with sickle-cell anemia, these diseases can be rapidly progressive and quickly fatal. It is therefore important for the patient to be evaluated and treated on an emergency basis. Children with sickle cell and fever are typically managed in the hospital with empiric antibiotics; without a functional spleen, they are at increased risk for serious bacterial infection.

You receive a telephone call from the mother of a 4-year-old child with sickle-cell anemia. She tells you that the child is breathing fast, coughing, and has a temperature of 40°C (104°F). What is your course of action?

acrocyanosis

cyanosis of the hands and feet which is a common findings during the first 48-72 hours of life; in some infants, it can last throughout the first month of life, particularly when the infant is cold. Along with cutis marmorata (mottling of the skin with venous prominence), this is a frequent intermittent sign of vasomotor instability characteristic of some infants.

Mongolian spots

dark blue hyperpigmented macules over the lumbrosacral area and buttocks of no pathologic significance. Most frequently seen in Hispanic, Asian, and African infants.

*prematurity* male sex perinatal asphyxia maternal diabetes cesarean section birth

list the risk factors for NRDS

strictures fistulas abscesses adhesions

list three complications of Crohn's disease

COL1A1 codes for type I collagen

osteogenesis imperfecta is due to a mutation in what gene?

infant botulism

The incidence of this disease is highest in California, Pennsylvania, and UTah, which have the greatest concentration of pathogen spores in their soil. Ingestion of spores from environmental dust leads to colonic colonization, followed by production of a toxin which blocks ACh release at presynaptic neuromuscular junctions. The toxins can also be found in honey and other canned foods. Suspect this disease in infants with bilateral bulbar palsies (ptosis, sluggish pupillary response to light, poor suck) followed by symmetric descending flaccid paralysis. Constipation and drooling also occur. Treat with immune globulin.

seborrheic dermatitis

The incidence of this rash peaks in the first year of life and again in adulthood. It is due to colonization by Malassezia species and primarily affects areas with numerous sebaceous glands. This includes the scalp, eyelids, nasolabial folds, postauricular area, and umbilicus. It presents as erythematous patches and plaques with yellow, oily scales. Mild pruritis may be present. Spontaneous resolution is common, and gentle emollients and non-medicated shampoo can be used to treat. If severe, low-potency glucocorticoids creams or topical ketoconazole can be used.

hypocalcemia

The infant of a diabetic mother has diaphoretic and irritable, and seems to have some twitching and tremors of the extremities. This is after glucose has been stabilized, between 48 and 96 hours of life. What metabolic derangement would you suspect?

prematurity

The leading risk factor for cerebral palsy

G6PD deficiency

The most common RBC enzymatic defect. Cause acute hemolytic disease induced by infection or medications, but can also occur as chronic hemolytic disease.

immune mediated thrombocytopenia (ITP)

The most common acquired platelet abnormality in childhood. May be viral, drug induced, or idiopathic (most common). Illness typically occurs 1-4 weeks after a virla infection, beginning abruptly with petechiae and bruising or epistaxis and gum bleeding. Labs show thrombocytopenia and smear shows few large "sticky" platelets. Treat with IVIG or corticosteroids. Anti-D immunoglobulin is a second-line agent that may also be effective.

iron deficiency anemia

The most common blood disease during infancy and childhood; the majority of cases are caused by inadequate iron intake. This nutritional deficiency presents in two age groups. In infants 9-24 months, there are insufficient iron stores. In adolescent girls, menstrual bleeding can cause iron deficiency when coupled with poor diet and rapid growth.

acute lymphoblastic leukemia

The most common cancer in children, with peak incidence age 2-5. Presents nonspecifically with fever, fatigue, weight loss, etc. Physical exam typically reveals signs of bone marrow failure - pallor, petechiae, etc from anemia and thrombocytopenia. The patient will have recurrent bacterial and fungal infections due to severe neutropenia (neutrophil count <500). Bone pain, lymphadenopathy, and hepatosplenomegaly are all characteristic. Diagnose with bone marrow biopsy showing >25% lymphoblasts.

endocarditis

The most common cardiac finding in rheumatic fever; typically causes insufficiency of the left-sided valves (mitral and aortic); it rarely affects the pulmonic or tricuspid valves.

E coli

The most common causative organism of UTIs

sickle cell disease

The most common cause of *pediatric stroke*; diagnosis can be confirmed via hemoglobin electrophoresis

duodenal atresia

The most common cause of GI obstruction in the neonatal period. Due to failure of the lumen to recanalize at 8-10 weeks gestation. Male predominant and associated with Down syndrome.

KAwasaki disease

The most common cause of acquired heart disease in children in the US; common in males and children of Asian ethnicity. Mean age of presentation 18-24 months.

migraine headaches

The most common cause of acute and recurrent headaches in the pediatric population. They typically present with unilateral or bilateral frontal pain, photophobia, phonophobia, nausea, vomiting, and sometimes an aura. First-line treatment in children includes acetaminophen, NSAIDs, and supportive management. Triptans may be tried if these measures are not effective.

anemia of prematurity

The most common cause of anemia in preterm infants. It is due to diminished EPO levels, shorted RBC life span, and blood loss. Lab studies will show decreased hemoglobin and hematocrit and a relatively low retic count.

*back strain*, defined as muscle soreness from overuse or bad body mechanics

The most common cause of back pain in children. Diffuse muscular pain is present without neurologic deficits and physical exam is normal. Treat with rest and analgesics.

Campylobacter jejuni

The most common cause of bacterial bloody diarrhea in the US. Often self-limited. Spread by comtaminated poultry and other food. Oral erythromycin is treatment, though it often resolves without intervention.

fibroadenoma

The most common cause of breast mass in an adolescent. A solitary, rubbery, mobile, well-circumscribed breast mass, often located in the outer quadrant of the breast. Premenstrual tenderness is common. IF identified, reexamine over at least one menstrual cycle; if the mass decreases in size or tenderness after the menstrual period, the patient can be reassured that the probability of benign disease is very high.

abusive head trauma

The most common cause of death from child abuse; repetitive acceleration-deceleration forces cause shearing of the subdural bridging veins and vitreoretinal traction, resulting in subdural and retinal hemorrhages.

anal fissure

The most common cause of hematochezia in infants, typically associated with constipation. If it occurs in older children or teenagers, consider underlying IBD.

internal tibial torsion

The most common cause of in-toeing in children under 2. Characterized by a foot that points medially when knee is flexed to 90 degrees. the patella faces forward. Usually bilateral. Manage with observation only; prognosis is excellent with improvement by 3 years.

transient proteinuria

The most common cause of isolated proteinuria in children; should be reevaluated with a repeat urine dipstick testing on two separate occasions to rule out persistent proteinuria, which requires further evaluation for udnerlying renal disease

Klinefelter syndrome

The most common cause of male hypogonadism and infertility. Due to a XXY genotype. Presents with tall stature and long extremities, delayed puberty due to lack of testosterone, gynecomastia, variable intelligence, and behavioral findings including antisocial behavior and excessive shyness or aggression.

minimal change disease

The most common cause of nephrotic syndrome in children, with a median age of 2-3. Caused by T-cell mediated injury to podocytes causing increased molecular permeability to albumin. Majority of cases, however, are idiopathic. Clinical features include edema, fatigue, and lack of hematuria. Diagnosed via proteinuria, hypoalbuminemia, and renal biopsy without microscopic changes. However, renal biopsy is invasive and unlikely to change management in suspected disease, so give corticosteroids for diagnostic treatment.

Hirschprung's

The most common cause of organic constipation in an otherwise healthy child.

*Viral infection:* coxsackievirus, echovirus, adenovirus, influenza virus, parainlfuenza virus, EBV.

The most common cause of pericarditis in children

viruses

The most common cause of pneumonia in all age groups

constitutional growth delay

The most common cause of short stature and pubertal delay in adolescents; affected individuals have normal birth weight and height, but growth velocity slows and they drop on their growth curve to the 5th to 10th percentile. Around 3 years, they regain a normal growth velocity which follows that curve. Puberty is delayed but eventually occurs, and the child will have a normal growth spurt and reach a normal adult height. Bone age radiographs will show a bone age that is delayed compared to chronological age.

juvenile polyps

The most common cause of significant lower GI bleeding beyond infancy. Bleeding is painless, intermittent, and often streaky. Colonoscopy with polypectomy is the definitive treatment.

functional fecal retention

The most common form of constipation during childhood; results from inappropriate constriction of the external anal sphincter. Infants and toddlers may inappropriately retain stool as a result of traumatic events (hard stool, painful diarrhea, diaper rash, abuse), leading to a self-fulfilling cycle of icnreased fecal mass, fecal hardness, and painful defecation. Behavioral in pathophysiology.

achondroplasia

The most common genetic form of skeletal dysplasia. Affected persons bear a striking resemblance to one another and are identified by their extremely short extremities; prominent foreheads; short, stubby fingers; and marked lumbar lordosis. Although they go through normal puberty, affected females must have children by cesarean section because of the pelvic deformity. Other complications include hydrocephalus secondary to bony overgrowth at the foramen magnum. It is inherited in an autosomal dominant manner, but most cases represent spontaneous mutations in unaffected parents.

enterobius vermicularis (pinworm)

The most common helminth infection in the US. Transmitted by fecal-oral ingestion of eggs. Features include anal or vulvular pruritus, insomnia, anorexia, enuresis, and nighttime teeth grinding. Treat with mebandazole, albendazole, or pyrantel pamoate, and treat all close contacts.

von Willebrand disease

The most common hereditary bleeding disorder, of autosomal dominant inheritance. Most patients have mild to moderate bleeding, usually involving mucocutaneous surfaces. Common signs include epistaxis, menorrhagia, bruising and bleeding after dental extraction or tonsillectomy, and excessive bleeding after trauma. Hemarthroses are unusual. Bleeding time and aPTT will be prolonged.

Friedreich ataxia

The most common hereditary form of ataxia. An autosomal recessive condition that classically presents with progressive ataxia in adolescence. Caused by GAA repeats in the frataxin gene. Findings include dysarthria, limb weakness, loss of DTRs, progressive gait and limb ataxia, and loss of position and vibratory senses due to deterioration of the dorsal spinal column. Kyphoscoliosis and pes cavus are common. HCM occurs in most patients, with an increased risk of arrhythmia and heart failure, usually leading to death by age 40. Diagnosis is confirmed by genetic testing.

IgA deficiency

The most common immune deficiency. Usually presents with respiratory infections, GI manifestations (Giardiasis, chronic diarrhea), autoimmune and rheumatic disease, and atopic disease. Diagnosis is by quantitative measurement of serum Ig showing Iga <7 with normal levels of other Ig.

rotavirus

The most common infectious agent causing gastroenteritis; usually seen in the winter months, spread by fecal-oral route. Infected patients present with vomiting, diarrhea, and dehydration. Diarrhea is usually self-limited and lasts 4-7 days. Treatment is supportive, with attention to fluid management.

hereditary spherocytosis

The most common inherited abnormality of the RBC; occurs predominantly in persons of Northern European ancestry. Caused by abnormality of RBC membrane protein *spectrin*, causing RBC to assume a spherical shape. *Autosomal dominant*.

osteogenic sarcoma

The most common malignant bone tumor; incidence peaks during rapid growth spurt of adolescence. About 50% occur near the knee, in the metaphysis of the long bones. It presents with pain, swelling, and a soft tissue mass; systemic signs are uncommon. On x-ray, the periosteal reaction with "sunburst" appearance is characteristic. Metastases occur in 15% at presentation, most commonly in the lung..

congenital hypothyroidism

The most common metabolic disorder; due to thyroid dysgenesis or hypoplasia, but can also be due to PTU use during pregnancy or maternal autoimmune thyroid disease. Most newborns are asymptomatic at birth, but present later with prolonged jaundice and poor feeding, lethargy and constipation,and large fontanelles, protruding tongue, umbilical hernia, myxedema, mottled skin, hypothermia, delayed neurodevelopment, and poor growth. Immediately begin L-thyroxine treatment,

*Iron deficiency:* they may not have the telltale signs like pica, fatigue or pallor, so universal screening is recommended by age 1 year. Empiric oral iron therapy is the most cost-effective approach to treatment.

The most common nutritional deficiency in children, especially in those who drink cow's milk excessively. Remember that children should consume <24 ounces per day of cow's milk to avoid this.

Waardenburg syndrome

The most common of several syndromes that are characterized by both deafness and pigmentary changes. Features of this syndrome, which is inherited as an autosomal dominant disorder, include a distinctive white forelock, heterochromia irides, unilateral or bilateral congenital deafness, and lateral displacement of the inner canthi.

PCP

The most common opportunistic infection in HIV+ children. Risk of infection correlates with CD4 count. Presents with fever, hypoxia, and itnerstitial pulmonary infiltrates. Prophylaxis is with oral Bactrim (TMP/SMX); treatment is with Bactrim, pentamidine, or atovaquone.

Pseudomonas aeruginosa

The most common pathogen to cause otitis externa

*Appendicitis:* occurs when the lumen of the appendix is obstructed by eithre a fecalith or by lymphoid tissue causing appendiceal distention and ischemia. This distention produces visceral pain referred to the T-10 dermatome or periumbilical region and the release of inflammatory mediators. Without surgical intervention, this process usually leads to perforation within 36-48 hours.

The most common pediatric emergency operation, with peak incidence 10-12 years of age

painless lymphadenopathy

The most common presenting feature of lymphoma

osteosarcoma

The most common primary bone tumor affecting children and young adults. Boys between 13 and 16 are at higher risk. In children, it occurs most frequently at the metaphyses of long bones like the distal femur, proximal tibia, and proximal humerus. On physical exam, there will be a *tender soft-tissue mass.* Characteristic x-ray findings include a spiculated *sunburst pattern* and periosteal elevation known as the *Codman triangle.* Alk phos and LDH are both elevated due to turnover of damaged osteocytes, and high levels may correlate with adverse prognosis. Treatment includes excision and chemotherapy.

Selective IgA Deficiency

The most common primary immunodeficiency and typically benign. Recurrent respiratory or GI infections can be seen. Big red flag is patients who get anaphylactic reaction after blood transfusions from expsosure to IgA.

Wilms tumor

The most common primary renal neoplasm of childhood, usually diagnosed at age 2-5 years and affecting a single kidney. It usually presents as an asymptomatic firm, smooth, unilateral abdominal mass found incidentally. There can also be abdominal pain, HTN, hematuria, and fever. Get an abdominal US to identify the lesion, followed by CT contrast abdomen and CT chest to look for pulmonary metastases.

achondroplasia

The most common skeletal dysplasia, characterized by rhizomelia (short humerus and femur). Can be autosomal dominant, but most cases are sporadic. Caused by a mutation in the FGF3 gene. Features include megalencephaly (large brain), foramen magnum stenosis, frontal bossing. lumbar kyphosis infancy which evolves to lumbar lordosis in later childhood, and trident hands. There is also recurrent otitis media with conductive hearing loss.

rhabdomyosarcoma

The most common soft tissue sarcoma in childhood; a malignant tumor of the same embryonic mesenchyme that gives rise to skeletal muscle. Often occurs in children under 10. The initial presentation is usually a painless soft tissue mass. The head and neck, including the orbit, are the most common sites of involvement. The second most common site is the genitourinary tract. The third most common is the extremities.

bat bites

The most common source of rabies transmission in the US; bites can be painless and may even go unnoticed.

Shiga toxin associated HUS

The most common subtype of HUS seen in childhood. Occurs as a result of intestinal infection with a toxin producing bacteria, most commonly E coli 0157:H7. The toxin causes vascular endothelial injury by binding to endothelial cells, especially in the renal vasculature, causing thrombi formation and renal ischemia. Presents as diarrheal prodrome followed by hemolytic anemia, thrombocytopenia, and acute renal failure. Treatment is supportive and antibiotics are not indicated.

Down syndrome

The most common trisomy; risk increases with maternal age. Incidence estimated to be 1:660 live births.

low-grade astrocytoma

The most common type of brain tumor in children

IgA nephropathy (Berger's disease)

The most common type of chronic glomerulonephritis worldwide; more prevalent in Asia and Australia and in Native Americans. Cause is poorly understood. Clinical features include recurrent bouts of gross hematuria associated with respiratory infections. Diagnosis is by renal biopsy, which shows mesangial proliferation and increased mesangial matrix on light microscopy. Immunofluorescent microscopy reveals mesangial deposition of IgA. 50% of patients have elevated serum IgA. Treatment is supportive.

tonic-clonic

The most common type of generalized seizure. Characterized by increased thoracic and abdominal muscle tone, followed by clonic movements of the arms and legs, eyes rolling up, incontinence, decreased consciousness, and a postictal state.

hepatoblastoma

The most common type of liver tumor in childhood. It almost always occurs in children under 3, and is associated with Beckwith-Wiedemann syndrome.

ornithine transcarbamylase deficiency

The most common urea cycle defect; X linked recessive, so it affects males mostly. Presents as vomiting and lethargy leading to coma. Diagnosed based on elevated urine orotic acid, decreased serum citrulline, and increased ornithine, as well as by liver biopsy. Manage with low protein diet; liver transplant may be necessary.

nevus simplex "salmon patch" telangiectatic nevus

The most common vascular lesion of infancy, occurring in 30-40% of newborns and appearing as a pink macular lesion on the nape of the neck *("stork bite")*, upper eyelids, glabella, or nasolabial. It is often transient.

pericarditis

The most frequent cardiac manifestation of SLE

emergency surgical drainage and IV abx

The most helpful intervention for preventing permanent joint destruction in septic arthritis.

hypertrophic pyloric stenosis

Thickening of the circular pylorus muscle resulting in gastric outlet obstruction with projectile vomiting. More common in *first-born male children.* Etiology unknown. Associated with duodenal atresia, TEF, trisomy 18, and Cornelia de Lange syndrome.

*Malrotation with volvulus:* malrotation results when incomplete rotation of the intestines occurs during embryologic development. The most common type of malrotation is failure of the cecum to move to its correct location in the right lower quadrant. Most patients present in the first weeks of life with bilious vomiting indicative of bowel obstruction and/or intermittent abdominal pain. Acute presentation, similar to that in the question, is caused by a volvulus of the intestines. The diagnosis is confirmed by radiographs; barium contrast studies (upper GI and/or enema) demonstrate malposition of the cecum in the vast majority of cases. Treatment is surgical.

This GI illness presents with bilious vomiting. Plain KUB will show dilated stomach and proximal bowel loops. A cross-table upper GI will demonstrate a "curly Q" twist of barium as it passes through the affected area.

Crohn's disease

This IBD may involve any segment of the GI tract, from mouth to anus. It is characterized by eccentric segmental inflammation with "skip lesions." Inflammation is trnasmural and may lead to sinus tracts, fistulas, and crypt abscesses. Most children have disease involving the terminal ileum. Small bowel disease often leads to malabsorption with resultant iron, zinc, folate, vitamin B12 deficiencies.

Henoch Schonlein purpura

This IgA-mediated leukocytoclastic vasculitis is characterized by palpable purpura, arthritis/arthralgias, abdominal pain (and sometimes intussusception), and renal disease similar to IgA nephropathy. Labs will show normal platelets and coag studies, normal to increased creatinine, hematuria, RBCs casts, and proteinuria. Renal biopsy will show IgA deposition in the mesangium. Treatment is supporitve (hydration and NSAIDs and steroids if severe).

HSV-1/HSV-2

This STD features painful shallow ulcers, constitutional synptoms, and inguinal adenopathy. Diagnosed by typical lesions and positive Tzanck smear, positive culture from lesion, or elevated antibodies. Treat with oral acyclovir until resolution; severe infection with disseminated disease requires IV acyclovir.

primary syphilis

This STD presents with a painless, single ulcer with well-demarcated borders and a clean base. There may also be painless inguinal adenopathy. Manage with IM penicillin or oral doxycycline if there is a penicillin allergy.

*chancroid* (Haemophilus ducreyi)

This STD presents with painful ulcers with red, irregular borders and purulent bases. There will also be painful inguinal adenopathy; nodes may be fluctuant. Treat with oral azithromycin, erythromycin, or IM ceftriaxone.

renal tubular acidosis

This abnormality often presents as a normal anion gap acidosis and failure to thrive. It can either be caused by a defect in hydrogen excretion or bicarb resorption in the kidney. Treatment consists of oral bicarb replacement.

hypocalcemia

This abnormality presents with tetany (carpopedal spasm, laaryngospasm, paresthesias) and seizures. Prolonged QT interval is often seen.

vascular ring

This abnormality results from abnormal development of the aortic arch, which causes tracheal, bronchial, and/or esophageal compression. This can either be complete (eg circumferential around the trachea or esophageal), such as a double aortic arch, or incomplete. They usually present in patients under 1 year old with respiratory symptoms like stridor, wheezing, or coughing, and esophageal symptoms like dysphagia, vomiting, difficulty feeding. Stridor is often biphasic (both inspiratory and expiratory). The stridor will also improve with neck extension, which decreases tracheal compression.

*ceftriaxone* use cefotaxime instead

This antibiotic should not be used in children under 30 days as it causes hyperbilirubinemia

SVT

This arrhythmia is characterized by rapid heart rate (in this case about 250 beats per minute), little rate variability, and a consistent P wave for each QRS complex. If prolonged, it can lead to heart failure with hepatomegaly and respiratory compromise. The first-line treatment is to stimulate the vagus nerve using techniques such as carotid massage, immersion of the face in cold water, or voluntary straining. Rapid infusion of IV adenosine can affect resolution if the maneuvers are not successful.

cromolyn sodium nedocromil sodium

This asthma drugs are anti-inflammatory and used for prophylaxis. They work by inducing inhibition of activation and release of inflammatory mediators. They have no effect on acute symptoms, but may help prevent exacerbations.

Type 1 Diabetes Mellitus

This autoimmune disease is characterized by lymphocytic infiltration of the pancreas; islet cell antibodies are present in the majority of cases. The classic presentation includes several weeks of polyuria, polydipsia, nocturia, and sometimes enuresis. As symptoms progress, weight loss, vomiting, and dehydration occur.

physiologic jaundice

This benign type of neonatal jaundice becomes apparent on the second or third day of life, peaks to levels no higher than about 12 mg/dL on the fourth or fifth day, and disappears by the end of the first week of life. The rate of rise is less than 5 mg/dL per 24 hours and levels of conjugated bilirubin do not exceed about 1 mg/dL

Chiari II malformation

This brain lesion is characterized by a downward displacement of the cerebellum and medulla through the foramen magnum, blocking CSF flow. This malformation is often associated with a lumbosacral myelomeningocele

coarctation of the aorta

This cardiac defect turns up in babies with HTN, claudication, or obvious temp differences between arms and legs. If you suspect it, get BPs on arms and legs; there will be a large difference. Do an echo to diagnose. Surgically correct. X ray will show rib notching as collaterals erode the ribs.

Tetralogy of Fallot

This cardiac pathology presents with hypercyanotic spells due to obstruction of the right ventricular outflow tract during exertion. There is also a harsh crescendo-decrescendo systolic murmur over the left upper sternal border, reflecting turbulence at the stenotic pulmonary artery.

thalassemia

This cause of anemia will present with low MCV, normal RDW, and elevated reticulocyte count. The *Mentzer index* (MCV/RBC) will be less than 13.

pyloric stenosis

This cause of nonbilious projectile vomiting is most common in first-born boys age 3-5 who are fed formula. Vomiting can result in dehydration and weight loss. Diagnosis can be confirmed with abdominal ultrasound.

cholestasis

This characterized by retention of bile within the liver, with prolonged elevation of conjugated (direct) bilirubin. Can be caused by infection, metabolic derangements, mechanical obstruction, alpha 1 antitrypsin deficiency, and other idiopathic causes. Usually presents with jaundice, acholic or light stools, dark urine, hepatomegaly, bleeding, and failure to thrive.

pseudocyst

This complication of acute pancreatitis is a collection of fluid rich in pancreatic enzymes that arises from pancreatic tissue. If small, it may resolve on its own; if large, it may require surgical drainage.

infantile botulism

This condition is characterized by bulbar weakness and paralysis that develops in infants during the first year of life secondary to ingestion of Clostridium botulinum spores and absorption of toxins. Source of toxin is infected foods like honey. Spores are unearthed from soil. The toxin prevents the presynpatic release of acetylcholine. Symptom onset occurs 12-48 hours after ingestion of spores, with constipation followed by weak cry and suck, loss of motor milestones, ophthalmoplegia, and hyporeflexia. Paralysis is symmetric and descending.

hereditary angioedema

This condition is characterized by recurrent edema without associated pruritus or urticaria. It is autosomal dominant and caused by a deficiency of C1 inhibitor, which leads to elevated bradykinin. It usually presents in adolescence with acute onset of swelling after a dental procedure, stress, or trauma. The face, limbs, and genitalia are most commonly affected. Bowel wall edema can present as abdominal pain, vomiting, and diarrhea. The most life-threatening risk is laryngeal edema, which can cause laryngospasm and airway obstruction. Diagnosis is via depressed C4 levels; treatment is by giving C1 inhibitor concentrate.

febrile seizure

This condition is defined as any seizure that is accompanied by a fever owing to non-CNS cause in patients from 6 months to 6 years. The mechanism is unknown. First-time or occasional seizures are not treated with anticonvulsants; only frequent, recurrent seizures require daily anticonvulsant prophylaxis and abortive rectal diazepam.

primary craniosysnostosis

This condition is usually sporadic (occurring in 1 in 2000 births) and more commonly affects the sagittal suture, which results in scaphocephaly (a long and narrow skull). In general, premature fusion of a single suture does not cause increased intracranial pressure or hydrocephalus; these features are more common with premature fusion of two or more sutures. The therapy for this condition is controversial, but usually involves surgery; consultation with a neurosurgeon would be indicated.

transient tyrosinemia of the newborn

This condition occurs in premature infants who get high protein diets. Clinical features include poor feeding or lethargy; elevated serum tyrosine and phenylalanine will be seen. Management includes decreasing protein intake during acute episode and using vitamin C to eliminate tyrosine. Prognosis is good and the self-limited disease should resolve within a month.

secondary adrenal insufficiency

This condition results from any process that interferes with release of cortisol-releasing hormone from the hypothalamus or ACTH from the pituitary; it will present with signs of cortisol deficiency (anorexia, weakness, hyponatremia, hypotension) but there will be no aldosterone deficiency due to the intact RAAS. Thus, serum potassium will be normal.

*Primary adrenal insufficiency* Addison's disease congenital adrenal hyperplasia adrenoleukodystrophy

This condition results from destruction of the adrenal cortex or from an enzyme deficiency. Patients will present with signs of both cortisol deficiency (anorexia, weakness, hyponatremia, hypotension, increased pigmentation) and aldosterone deficiency (failure to thrive, salt craving, hyponatremia, *hyperkalemia*).

vitamin A deficiency

This condition usually manifests in the second or third year of life as impaired adaptation to darkness (which may progress to night blindness), photophobia, dry scaly skin, dry conjunctiva (xerosis conjunctiva), dry cornea (xerosis cornea) and a wrinkled, cloudy cornea (keratomalacia). Bitot spots (dry, silver-gray plaques on the bulbar ocnjunctiva) and follicular hyperkeratosis of the shoulders, buttocks, and extensor surfaces can also be seen, though they are less common findings.

hydrocephalus

This condition usually presents with increasing head circumference. In infants, there will be open cranial sutures which are split and large anterior and posterior fontanelles. There will also be a sunset sign, or tonic downward deviation of both eyes caused by pressure from an enlarged third ventricle on the upward gaze center in the midbrain. In older children, there will be signs of increased ICP, since sutures are closed; these include headache, nausea, vomiting, unilateral sixth nerve palsy, papilledema, and brisk DTRs with a downward plantar response.

milk protein allergy

This condition, exclusive to infants, is a non-IgE mediated immunologic response to proteins in formula or breast milk causing rectal and colonic inflammation. The clinical presentation includes eczema, regurgitation or vomiting, and painless bloody stools. Rectal bleeding should stop within 2 weeks of eliminating maternal dairy and soy products or switching to hydrolyzed formula.

cholecystitis

This condition, uncommon in children, presents with RUQ abdominal pain, fever, anorexia, and vomiting. Jaundice may be a late findings. Physical exam may reveal Murphy's sign, wherein palpation of the RUQ during inspiration elicits intense pain and causes patient to stop inspiratory effort. Guarding and peritoneal signs may also be present.

tricuspid valve atresia

This congenital cardiac anomaly should be suspected in a cyanotic infant with left axis deviation and small or absent R waves in the precordial leads. This is due to a hypoplastic right ventricle and diminished right ventricular forces on ECG. There will also be decreased pulmonary markings on CXR due to hypoplasia of the right ventricle and pulmonary outflow tract.

aortic stenosis

This congenital cardiac defect results in reduced left ventricular output which may lead to myocardial ischemia. In severe cases in the neonate, this be result in hypoplasia of the left ventricle as a result of impaired fetal left ventricular development.

VSD

This congenital cardiac lesion procues a holosystolic murmur that is loudest at the left lower sternal border, and an apical diastolic rumble due to increased blood flow across the mitral vavle. Patients may present with growth failure, easy fatigability, or heart failure.

congenital cystic adenomatoid malforamtion (CCAM)

This congenital disorder is thought to arise from an embryonic disruption before the 35th day that causes improper development of bronchioles. The cystic mass is usually identified on prenatal ultrasound around the 20th week. Large lesions (as that noted on the radiograph on the patient's right side) may compress the affected lung and cause pulmonary hypoplasia, and cause midline shift away from the lesion (note the heart is shifted toward the patient's left on the radiograph). Treatment is typically surgical excision of the affected lobe. Some patients may be at risk for primary pulmonary malignancy.

coarctation of the aorta

This congenital heart defect can present in older children with hypertension noted in the right arm and BP reduced in the lower extremities. The femoral pulse is dampened and delayed until after the radial pulse. A bruit of turbulence through the defect may be audible at the left upper back near the scapula.

VSD

This congenital heart defect causes a *left to right shunt* due to the lower resistance within the pulmonary circulation compared with resistance within the systemic circulation. Over time, this leads to hypertrophy of the pulmonary vessels and increased PVR (pulmonary HTN). If PVR remains elevated, pulmonary HTN can become irreversible, even if the defect is surgically closed. A holosystolic murmur is often heard, with intensity that increases with the size of the defect.

VSD

This congenital heart defect has a characteristic holosystolic murmur heard at the left lower sternal border due to turbulence. An apical diastolic rumble may also be heard from increased flow across the mitral valve. Results in left-to-right shunt which in turn leads to pulmonary HTN, growth failure, diaphoresis, easy fatiguability, and CHF.

PDA

This congenital heart defect leads to *left to right shunt.* This increases pulmonary blood flow. Small defects produce no symptoms, but moderate or large defects generally result in CHF symptoms due to increased pulmonary blood flow. In additon, a "machine-like" murmur is heard continuously at the upper left sternal border. If the shunt is large, there may also be diastolic rumble and a widened pulse pressure (>30 mmHg) with brisk pulses.

total anomalous pulmonary venous return (TAPVR)

This congenital heart defect occurs when the pulmonary veins drain into the systemic venous side rather than into the left atrium. May be supracardiac (into the right SVC), cardiac (into the right atrium or coronary sinus), or infracardiac (into the portal system. Systemic and pulmonary venous blood enter the right atria and mix together. Blood must pass into the left atrium by PFO or ASD. Cyanosis is present, as is a pulmonary flow murmur at the mid-left sternal border caused by increased pulmonary blood flow.

truncus arteriosus

This congenital heart defect will present with a systolic ejection murmur from increased flow across the truncal valve and a single S2 caused by the presence of only one atrioventricular valve.

viral croup

This disease begins with a URI prodrome for 2-3 days, followed by stridor. Inspiratory stridor, barky cough, and hoarse voice are typical and last 3-7 days. Stridor and cough worsen at night and with agitation. AP CXR will show "Steeple sign" of subglottic narrowing.

HSP

This disease can be diagnosed by history and physical exam; labs will include *increased serum IgA* and *normal platelet count* in spite of the presence of petechiae and purpura.

orbital cellulitis

This disease can be differentiated from preseptal cellulitis (A more mild condition), though they both present with eyelid erythema, swelling, and chemosis, by the presence of pain with extraocular movements, visual changes, proptosis, or ophthalmoplegia. It is a severe infection that can lead to permanent visual impairment or intracranial complications like infection or thrombosis.

aplastic anemia

This disease can either be congenital or acquired. Signs include bruising, petechiae, pallor, or serious infections. Labs show pancytopenia, low reticulocyte count, and hypocellular bone marrow.

SLE

This disease can present with lab findings of elevated ESR and CRP, anemia, leukopenia, thrombocytopenia, decreased C3 and C4, and UA with proteinuria. ANA is almost universally elevated but is nonspecific for this disease.

asthma

This disease causes breathing difficulty through smooth muscle bronchoconstriction, airway mucosal edema, increased secretions with mucous plugging, eventual airway wall remodeling, and production of inflammatory mediators. These responses are prompted by triggers such as exercise, cold air, emotions, allergens, GERD, and pollutants.

Kartagener syndrome (primary ciliary dyskinesia)

This disease causes recurrent sinopulmonary infections due to dysmotile cilia that are the result of aberrant production of ciliary dynein arms. These defects result in impaired cilial function, poor mucociliary clearance of secretions, and chronic infections. The classic presentation is *situs invertus, recurretn sinusitis, and bronchiectasis.*

GERD

This disease commonly presents in infants with emesis, feeding refusal, irritability, or constant hunger. In older children, symptoms include midepigastric pain temporarily relieved with food, nausea (espeically on awakening), hoarseness, halitosis, and wheezing.

pericarditis

This disease features chest pain most intense while supine and relieved when sitting upright; physical exam findings include friction rub, distant heart sounds, hepatomegaly, and pulsus paradoxus (>10 mmHg reduction in systolic BP on deep inspiration). ECG may show ST-segment changes or low-voltage QRS. CXR shows enlarged heart shadow.

pyloric stenosis

This disease features vomiting of nonbilious milky fluid starting during the second or third week of life. Vomiting is often described as projectile and typically occurs immediately after feeding. Affected infants may be irritable but hungry and 5% are jaundiced. Dehydration may occur. Physical exam will show a palpable mass just above and to the right of the umbilicus.

neonatal polycythemia (hematocrit >65%)

This disease in newborns presents with a ruddy or plethoric appearance, lethargy, irritability, resp distress, tachypnea, poor feeding, and cyanosis. It can be caused by excessive transfusion (as in delayed umbilical cord clamping, twin to twin transfusion syndrome), intrauterine hypoxia (maternal diabetes or hypertension, smoking) or genetic conditions (trisomy 21).

cat scratch disease

This disease is caused by Bartonella henselae, a fastidious gram-negative bacilli. It is often transmitted by a cat scratch or bite. It manifests as a papule at the scratch/bite site with associated regional adenopathy. There can also be fever. Diagnosis is usually clinical and the disease is generally self-limiting, though azithromycin can be given and is typically recommended.

rocky mountain spotted fever

This disease is caused by Rickettsia rickettsii, a Gram-negative intracellular coccobacillus that is transmitted by tick bite. It is endemic to the southeastern US. Features include fever, petechial rash that begins on the extremities and moves in a caudal and centripetal direction, myalgias, hepatosplenomegaly, jaundice, headache, and hypotension. Labs will show thrombocytopenia, elevated LFTs, and hyponatremia. CSF may show an aseptic meningitis picture. Diagnosis is by serologic tests for Rickettsia. Treat with oral or IV doxycycline.

von Gierke disease

This disease is caused by deficient glucose-6-phosphatase in the liver, kidneys, and intestinal mucosa, resulting in impaired conversion of glycogen to glucose. This leads to glycogen accumulation in the affected organs. Patients typically present at age 3-4 months with hypoglycemia (often with seizures) and lactic acidosis. Other labs include hyperuricemia and hyperlipidemia. Physical exam shows a doll-like face with rounded cheeks, thin extremities, short stature, and hepatomegaly. Spleen and heart are not involved.

CGD

This disease is caused by impaired intracellular killing by phagocytes. Pneumonia, cutaneous abscesses, and suppurative adenitis are common. Diagnosis is made by neutrophil function testing (eg dihydrorhodamine 123 or nitroblue tetrazolium testing).

pellagra (niacin deficiency)

This disease is characterized by *diarrhea, dermatitis, dementia*, and, if severe, death. It is mainly seen in resource-limited countries where the main diet is grain or corn, but can also be seen in Crohn disease, anorexia, and alcoholism.

SCID

This disease is characterized by profoundly defective T-cell and B-cell function. Patients with this disease have increased susceptibility to infection within the first few months of life with common pathogens and opportunistic ones like Candida and Pneumocystis carinii. Chronic diarrhea and failure to thrive are common.

Langerhans Cell Histiocytosis

This disease is characterized by skeletal involvemtn, most commonly the skull, with single or multiple bony lesion pesent which may be painful, palpable, and swollen. Chronic draining ears may be present with involvement of the mastiod bone. Also involved is the skin, and seborrheic dermatitis of the diaper area and scalp are often present. Finally, pituitary and hypothalamic involvement may lead to growth retardation, diabetes insipidus, hypogonadism, and panhypopituitarism.

XLA

This disease is due to mutations in the BTK gene on the X chromosome; the BTK gene is critical to normal B cell function, and mutations lead to a block in development from pre-B cell to mature B cells.

beta thal minor

This disease is often misdiagnosed as iron deficiency anemia; however, iron will be normal or elevated. Hypochromia, microcytosis, target cells, and anisocytosis will be noted on smear.

*XLA:* also known as Bruton agammaglobulinemia. Caused by a defect in tyrosine kinase that prevents development of mature B cells. Diagnosed by decreased Ig and B cells with normal T cell concentration and no response to vaccines. Treat with immunoglobulin replacement and prophylactic antibiotics.

This disease manifests as recurrent sinopulmonary and GI infections after age 6 months. On exam, lymphoid tissue like tonsils and lymph nodes will be absent.

hemolytic uremic syndrome

This disease occurs after infection with E coli O157:H7 which produces Shiga toxin. The toxin destroys the colonic epithalium, producing bloody diarrhea and abdominal pain. The classic triad of findings is *microangiopathic hemolytic anemia, thrombocytopenia, and acute kidney injury.*

amniotic band syndrome

This disease occurs as a result of rupture of the amniotic sac; fluid leak leads to intrauterine constraint, and small strands from the amnion may wrap around the fetus, causing limb scarring and amputation.

DKA

This disease occurs with hyperglycemia greater than 300 mg/dL along with ketonuria and serum bicarb <15 or serum pH < 7.3. It is a sequelae of diabetes, usually type 1. It often presents with vomiting, polyuria, dolydipsia, and dehydration; if severe, there can be abdominal pain, Kussmaul respirations, and coma. Fruity breath is characteristic due to ketones.

juvenile idiopathic arthritis (JIA)

This disease of children asymmetrically involves large joints, especially the knee, and often has no other symptoms. A major morbidity is chronic uveitis, resulting in blindness. Many patients have anterior uveitis as their only significant systemic manifestation.

retinopathy of prematurity

This disease of neonates occurs when normal development of retinal vasculature is interrupted by premature birth. While the capillary growth (vascularization) stops, capillary growth in general doesn't, leading to abnormal capillary growth (neovascularization). This leads to bleeding and scarring, causing distortion and detachment of the retina. Worsened by prematurity, high oxygen delivery. Primary treatment is laser photocoagulation.

*Neonatal tetanus:* due to Clostridium tetani, which is ubiquitous in soil. C tetani produces a toxin, tetanospasmin, that prevents inhibitory neurotransmitter release, causing spasms and hypertonicity. Treatments include antibiotics and tetanus immune globulin.

This disease of the newborn presents with difficult feeding due to trismus (lockjaw) as well as spasms and hypertonicity (clenched hands, dorsiflexed feet, opisthotonus). It often presents in the setting of an unimmunized mother and poor umbilical cord hygiene.

congenital hypothyroidism

This disease of the newborn presents with gradually developing apathy, weakness, hypotonia, large tongue, sluggish movement, abdominal bloating, and umbilical hernia.

viral pneumonia

This disease often begins with URI complaints like nasal congestion and rhinorrhea; fever, cough, and dyspnea typically follow. Physical exam may demonstrate tachypnea, wheezing, rales, or resp distress. Diagnosis is suggested by interstitial infiltrates on CXR and a WBC < 20,000 with a lymphocytic predominance. Management is supportive.

rubella

This disease present with low-grade fever, maculopapular rash with rapid cephalocaudal spread, and posterior auricular and suboccipital lymphadenopathy. Adolescents, especially females, may also present with arthralgias or arthritis that can persist after the rash resolves.

bronchiolitis

This disease presents with gradual onset URI symptoms which progress to tachypnea, fine rales, wheezing, and respiratory distress. The spleen and liver may appear enlarged as a result of lung hyperinflation. CXR reveals hyperinflation with air trapping, patchy infiltrates, and atelectasis. Improvement is often noted within 2 weeks, though more than 50% have recurrent wheezing.

rubella

This disease presents with low-grade fever, maculopapular rash with rapid cephalocaudal spread, and posterior auricular and suboccipital lymphadenopathy.

acute pancreatitis

This disease presents with periumbilical or epigastric pain, which may radiate to the back. Fever, anorexia, nausea, and vomiting are common. Physical exam will reveal epigastric tenderness, abdominal distention, and decreased bowel sounds. In acute hemorrhagic cases, hypotension, tachycardia, hypoxia, and capillary leak may all be seen.

DMD

This disease presents with slow progressive weakness affecting the legs first. Pseudohypertrophy of the calves is present, as is Gowers' sign. Cardiac involvement (cardiomegaly, tachycardia, cardiac failure) occurs in 50% of patients. CK levels are very high.

acute asthma exacerbation

This disease presents with tachypnea, dyspnea, nasal flaring, retractions, and multiphonic wheezing with a prolonged expiratory phase. CXR will reveal hyperinflation, peribronchial thickening, and patchy atelectasis. PFTs will reveal decreased expiratory flow rates.

retinoblastoma

This disease presents with, instead of a red light reflex, a *pure white retina* seen in the back of the eye. Don't confuse it with a cataract in the front of the eye. It is a tumor which needs to be resected; avoid radiation, as it increases the risk of secondary knockout in the good eye. Observe the patient for future *osteosarcoma,* especially in the distal femur.

retropharyngeal abscess

This disease resembles epiglottitis, with drooling and a fever, but the child will keep their neck fixed in sniffing position with refusal to move. they'll have a muffled voice and unilateral cervical lymphadenopathy. Get a CT scan and start abx, and make sure to consult surgery as soon as possible, usually ENT. Get intraoperative cultures to guide abx therapy.

infective endocarditis

This disease usually occurs in children with structural abnormalities of the heart or soon after cardiac surgery. It is an infection of the endocardium, most commonly by *Strep viridans* (alpha hemolytic strep) and *Staph* species, with gram-negative and fungal organisms more rarely causing disease.

parotitis

This disease usually presents as swelling centered above the angle of the jaw and fever. Physical exam of the oropharynx may reveal pus expressed from Stensen's duct.

cystic fibrosis

This disease usually presents with chronic progressive pulmonary insufficiency, pancreatic insufficiency, and high sweat electrolytes. Meconium ileus is often present at birth. Typical presenting features include recurrent or chronic respiratory symptoms, steatorrhea, and failure to thrive. PFTs show decreased resp flow rates and decreased lung volumes.

acute appendicitis

This disease usually presents with initial periumbilical pain followed by vomiting. Within hours, the pain usually localizes to the right lower quadrant. Fever and anorexia are usually present. Physical exam classically demonstrates TTP at an area two third of the way from the umbilicus to the anterior superior iliac spine called McBurney's point. Voluntary guarding may progress to involuntary guarding and rebound tenderness as peritoneal irritation increases. Labs will show leukocytosis with polymorphonuclear predominance

congenital cataracts

This disease will present with a *milky white appearance* in the front of the eye. Often caused by vertically transmitted infections or a galactosemia acquired early in life. Should be corrected surgically before amblyopia sets in.

immune thrombocytopenia

This disease, characterized by platelet count below 100,000, is most common in children age 2-5. It is due to autoantibodies against platelet membrane antigens, leading to destruction of antibody-platelet complexes in the spleen. It is usually preceded by a viral infection, and presents with petechiae and purpora. For children with cutaneous symptoms only, observation alone is reocmmended, as the course is usually self limitied with spontaneous recovery within 3 months. For children with mucosal bleeding, glucocorticoids or IVIG are first line treatment

acquired aplastic anemia

This disorder can occur with insult to the bone marrow from radiation, drugs, or infection. It results in pancytopenia. Patients present with pallor and fatigue (anemia), easy bruising and bleeding (thrombocytopenia) and fever (neutropenia). Diagnosis is by bone marrow biopsy showing profound hypocellularity and fatty infiltration of the marrow.

*Langerhans cell histiocytosis:* a rare disorder in which macrophages within tissue (histiocytes) proliferate and infiltrate organ systems.

This disorder features lytic bone lesions, skin lesions (purplish papules and eczematous rash), lymphadenopathy, hepatosplenomegaly, pulmonary cysts and nodules, and central diabetes insipidus.

CVID

This disorder is characteized by hypogammaglobulinemia. Most patients have normal numbers of B cells and T cells but variable degrees of T cell dysfunction. Usually featues respiratory infections by Hib, Moraxella, and Strep penumono, GI infectiosn by G lamblia and Campylobacter, and autoimmune disorders. There is increased risk of malignancy. Diagnosed by quantitative Ig measurement showing decreased serum Ig concentrations.

autism spectrum disorder

This disorder is characterized by abnormal social development and extreme behavioral rigidity, with onset in early development. Higher functioning individuals with normal language and intellect may be diagnosed later, when deficits become more apparent with increased social demands.

JIA

This disorder is characterized by chronic joint inflammation in children. Mean age of onset is 1-3 years. It is more common in females.

protein C deficiency

This disorder is due to absence of a vitmain K dependent factor that is a potent anticoagulant. Presents with purpura folminans, a nonthrombocytopenic purpura characterized by fever, shock ,and rapidly spreading skin bleeding and intravascular thrombosis. DVTs or CNS thrombosis are also common.

tuberous sclerosis

This disorder is predominated by *intellectual disability* and *seizures*. Presence of *ash leaf spot* and *angiofibromas* are characteristic, and neuroimaging will reveal cortical tubers. Treatment is largely supportive and kids will progress to severe intellectual disability.

fetal alcohol syndrome

This disorder may occur if a woman chronically drinks alcohol or binges during pregnancy. Features include SGA birth, FTT, microcephaly, long smooth philtrum with thin smooth upper lip, mental retardation, ADHD, and cardiac defects like VSD.

neonatal abstinence syndrome

This disorder of the newborn is caused by infant withdrawal to opiates and usually presents in the first few days of life. It is characterized by irritability, a high-pitched cry, poor sleeping, tremors, seizures, sweating, sneezing, tachypnea, poor feeding, vomiting, and diarrhea.

Angelman syndrome "happy puppet" syndrome

This disorder, a result of imprinting, has characteristic jerky gait and happy demeanor with frequent laughter and smiling of affected individuals. It is caused by deletion of a region on the maternally derived chromosome 15. Mental retardation is severe with significant speech delay.

DIC

This disorder, which can be triggered by endotoxin shock, results ultimately in the initiation of the intrinsic clotting mechanism and the generation of thrombin (prolonged PT and PTT, decreased fibrinogen concentration, and an increase in fibrin split products). Fibrin deposited in the microcirculatory system can lead to tissue ischemia and necrosis, further capillary damage, release of thromboplastic substances, and increased thrombin generation. Simultaneous activation of the fibrinolytic system produces increased amounts of fibrin split products, which inhibit thrombin activity. Peripheral smear will show fragmented cells and few platelets. Of utmost importance in the treatment of children who have this disorder is the management of the condition that precipitated the disorder.

hydroxyurea

This drug is indicated for sickle cell patients with frequent pain crises, a history of acute chest syndrome, or severe symptomatic anemia. It increases fetal hemoglobin, thus proportionally decreasing sickled hemoglobin. The primary dose-limiting side effect is myelosuppression (neutropenia, anemia, thrombocytopenia), but it is otherwise relatively safe.

propanolol

This drug may cause growth retardation when given to the mother *throughout pregnancy*, not only during the first trimester like many teratogenic drugs. It diminishes the ability of an asphyxiated infant to increase heart rate and cardiac output. It has also been associated with hpyoglycemia and apnea.

cholesteatoma

This ear abnormality will present with new-onset hearing loss or chronic ear drainage despite antibiotic therapy. Granulation tissue and skin debris may be seen within retraction pockets of the tympanic membrane on otoscopy.

*Glucocorticoid excess:* Cushing syndrome Iatrogenic (exogenous steroids)

This endocrine abnormality presents with poor growth with delayed bone age, central obesity, moon facies, nuchal fat pad, easy bruisability, purple striae, hypertension, and glucose intolerance. There will be elevated free cortisol in a 24-hour urine collection, and absence of the expected cortisol suppression seen in an overnight dexamethasone suppression test.

G6PD

This enzyme is critical for protecting RBCs from oxidative stress. Deficiency results in RBC damage when RBC is exposed to oxidants from infection, drugs (sulfa, salicylates, antimalarials), or fava beans. Symptoms occur 24-48 hours after oxidant exposure.

neonatal sepsis

This factor is associated with increased risk of neurologic damage in a jaundiced newborn, as it interrupts the BBB, allowing diffusion of bilirubin into the brain, leading to kernicterus

HBeAg

This factor rises very early in active hep B infection and is useful in diagnosing acute infection

umbilical hernia

This finding in the newborn is caused by incomplete closure of the umbilical ring. It is noticed as a soft swelling beneath the skin around the umbilicus that often protrudes during crying or straining. They occur more frequently in African American children. Most close spontaneously and usually no treatment is required. Those that persist beyond 4-5 years of age and those that cause symptoms may require surgical treatment.

jaundice

This finding is *always abnormal* within the first 24 hours of life, but is subsequently seen during the first few days following birth, when it is usually not associated with serious disease.

Pierre Robin syndrome

This genetic disorder features micrognathia, cleft lip and palate, and large protruding tongue. Feeding is often difficult because of the cleft palate. Complications include recurrent otitis media and upper airway obstruction that often requires tracheostomy.

Cri du chat syndrome

This genetic disorder is caused by a partial deletion of the short arm of chromosome 5, most often sporadically. Features include slow growth, microcephaly, mental retardation, hypertelorism, downslanting palpebral fissures, and a characteristic catlike cry.

phenylketonuria

This genetic disorder is caused by an autosomal recessive mutation in phenylalanine hydroxylase leading to a failure to convert phenylalanine into tyrosine, resulting in high levels of phenylalanine which cause neurologic injury. Clinical features include intellectual disability, seizures, musty body odor, and hypopigmentation of the skin, hair, and eyes. Diagnosis is done either via newborn screening (tandem mass spectrometry) or quantitative amino acid analysis showing increased levels of phenylalanine. Treat with dietary restriction of phenylalanine.

Russell Silver syndrome

This genetic disorder is known for features of short stature, skeletal asymmetry, and normal head circumference. Patients will be born SGA with a small triangular face, cafe au lait spots, and excessive sweating.

milia

very small cysts formed around the pilosebaceous follicles, which appear as tiny, whitish papules that are seen over the nose, cheeks, forehead, and chin. They usually disappear within a few weeks and do not require treatment.

colonoscopy with biopsies of the colon and terminal ileum

How do you confirm a diagnosis of IBD?

corneal abrasion

Damage to the corneal epithelium. Characterized by severe pain, tearing, photophobia, and foreign body sensation. Use fluorescein staining on the cornea to identify the lesion. Healing usually occurs within 24-48 hours.

Daily daytime Sx Nighttime Sx >1 time/week FEV1 60-80% of predicted Using SABA daily SABA for symptom relief Medium-dose inhaled steroids OR low-dose inhaled steroids + SABA

Define moderate persistent asthma and its treatment

Continuous Sx Frequent nighttime Sx Limited physical activity FEV1 < 60% predicted SABA for symptom relief High-dose inhaled steroid + LABA Systemic steroids if needed

Define severe persistent asthma and its treatment

IBD (Crohns + UC)

A cause of bloody diarrhea; diarrhea commonly lasts for >6 weeks and is associated with weight loss. Usually there is a family history of similar illness. Usually seen in 10-20 year old patients. A double contrast enema and colonoscopy are useful in diagnosing

11 beta hydroxylase deficiency

A cause of congenital adrenal hyperplasia that presents with characteristic hypertension and hypokalemia. Patients will have increased levels of 11-deoxycortisol.

3 beta hydroxysteroid dehydrogenase deficiency

A rare cause of congenital adrenal hyperplasia which presents with salt-wasting crisis, glucocorticoid deficiency, and ambiguous genitalia as a result of an early block in all three adrenal cortex steroid pathways. Patients will have increased levels of DHEA and 17-hydroxypregnenolone.

biliary atresia

Babies with this congenital condition will have persistent or worsening jaundice after 2 weeks of age. Labs will show direct hyperbilirubinemia. US can demonstrate pathology, but the best test is a HIDA scan which can, after 5-7 days of phenobarbital stimulation, show lack of bile reaching the duodenum. Treat with Kasai procedure.

erythema toxicum neonatorum

A *benign, transient* rash of newborns characterized by blanching erythematous papules which progress to pustules. Usually occurs in the first two weeks of life. It typically spares the palms and soles but can occur on any other part of the body. The rash will resolve spontaneously and parents can be reassured.

scoliosis

A developmental disorder of the spine mainly seen in female adolescents. Thorax tips to the side, causing a cosmetic deformity. More severe disease can cause respiratory issues. Perform an *Adam's test* (patient bends forward, asymmetric shoulders are diagnostic) and confirm with x-rays. Treat by bracing, with the goal of slowing progression, not curing. Surgery with rod placement is reserved for severe cases.

Chiari type II malformations with hydrocephalus

90% of lumbosacral myelomeningoceles are associated with what other neurologic abnormality?

strabismus

A "lazy eye," which should be corrected to prevent amblyopia. Confirmed on physical exam when reflection of light comes from separate locations on each eye. Should be corrected around 6 months. Later onset can often be treated with patching of dominant eye, glasses, or surgery.

*Tension pneumothorax*, a recognized complication of staphylococcal (and other) pneumonia, is caused by toxin production by the bacteria leading to rupture of the alveoli into the pleural space. Tension pneumothorax can be quickly lethal if not recognized and treated; this makes a high index of suspicion and prompt diagnosis mandatory. Immediate action to relieve the pressure of a tension pneumothorax is essential, usually accomplished by inserting a needle or catheter into the second or third intercostal spaces in the midclavicular line while the patient is supine.

A 1-year-old boy presenting with cough, fever, and mild hypoxia was admitted to the hospital last night. At that time, he had evidence of a right upper lobe consolidation on his chest radiograph. A blood culture has become positive in less than 24 hours for Staphylococcus aureus. Approximately 20 hours into his hospitalization, the nurse calls you because the child has acutely worsened over the previous few minutes, with markedly increased work in breathing, increasing oxygen requirement, and hypotension. What do you suspect is occurring?

Patients born normal who then have progressive developmental delay and hepatosplenomegaly with coarse facial features are likely to have a storage disease. *Hurler syndrome,* mucopolysaccharidosis type I, is an autosomal condition caused by a deficiency of α-l-iduronidase, which causes a deposition of dermatan sulfate and heparan sulfate in the body, and excessive excretion in the urine. Other features of this condition include umbilical hernia, kyphoscoliosis, deafness, cloudy corneas, and claw hand deformity. Death is common in childhood, a result of respiratory or cardiac compromise.

A 1-year-old boy presents with the complaint from his parents of "not developing normally." He was the product of an uneventful term pregnancy and delivery, and reportedly was normal at birth. His previous healthcare provider noted his developmental delay, and also noted that the child seemed to have an enlarged spleen and liver. On your examination, you confirm the developmental delay and the hepatosplenomegaly, and also notice that the child has short stature, macrocephaly, hirsutism, a coarse facies, and decreased joint mobility. What is the likely diagnosis?

The case is that of a child with *classic central nephrogenic diabetes insipidus*, a hereditary disorder in which the urine is hypotonic and produced in large volumes because the distal tubule and collecting duct of the kidneys fail to respond to antidiuretic hormone (vasopressin). Males are primarily affected through an X-linked recessive inheritance; autosomal dominant and recessive forms are also known. Although the condition is present at birth, the diagnosis is often not made until several months later, when excessive thirst, frequent voiding of large volumes of dilute urine, dehydration, and failure to thrive become obvious. Repeated measurement of serum sodium and urine specific gravity levels during a controlled fluid restriction challenge can make the diagnosis. Maintenance of adequate fluid intake and diet and use of saluretic medications (promoting the renal excretion of sodium) are the bases of therapy for this incurable disease. Acquired nephrogenic diabetes insipidus can be a result of lithium, methicillin, rifampin, and amphoteri-cin. It may also be seen with hypokalemia, hypercalcemia, polycystic kidney disease, Sjögren syndrome, and sickle-cell disease.

A 1-year-old child has been admitted to the hospital for failure to thrive. The family reports that the child is always crying for something to drink but voids almost constantly despite their limiting his liquid intake. They report 3 previous hospital admissions in the first 6 months of life for dehydration that was not associated with vomiting or diarrhea. Over the years, other family members reportedly have had similar histories. What is the likely diagnosis?

*Reye syndrome* is an acquired mitochondrial hepatopathy that results from the interaction of an influenza (or varicella) infection and aspirin use. While prevalence has decreased over the last few decades and it is now a rare disease, mortality remains the same at more than 40% of cases. Liver enzymes and ammonia are elevated, but total bilirubin is not. Patients initially present toward the end of a viral infection with sleepiness, emesis, and abnormal liver functions. As the disease progresses, the patient may develop seizures, coma, hyperventilation, and decorticate posturing. Ultimately they may develop respiratory arrest, loss of deep tendon reflexes (DTRs), and fixed and dilated pupils. Death is usually from cerebral edema and subsequent herniation. While aspirin is no longer routinely used in children as an antipyretic or pain reliever, the increase in the use of aspirin in adults with heart disease requires specific counseling for parents of children with influenza and varicella to avoid aspirin use. In addition, both of these infections are preventable with proper immunization.

A 10-month-old infant on long-term aspirin therapy for Kawasaki disease develops sudden onset of high fever, chills, diarrhea, and irritability. A rapid swab in your office identifies influenza A, adding her to the long list of influenza patients you have seen this December. Over the next few days, she slowly improves and becomes afebrile. However, 5 days after your last encounter you hear from the hospital that she has presented to the emergency center obtunded and posturing with evidence of liver dysfunction. What is the likely diagnosis?

*PUD:* Although the majority of children with periumbilical or epigastric pain have a functional gastrointestinal disorder, the presence of nocturnal abdominal pain and GI bleeding in a patient with a positive family history supports a diagnosis of peptic ulcer disease (PUD). A dull or aching pain is the most common symptom; the classic complaint of epigastric pain relieved by eating is not typical in the pediatric population. Symptoms often persist for several years before diagnosis. The increased incidence of PUD in families (25%-50%) and concordance in monozygotic twins suggest a genetic basis for the disease. About half of the patients with PUD will have hematemesis or melena. Diagnosis may be made conclusively with endoscopy; stains and cultures obtained during endoscopy can diagnose the subset of PUD caused by Helicobacter pylori.

A 10-year-old boy has been having "bellyaches" for about 2 years. They occur at night as well as during the day. Occasionally, he vomits after the onset of pain. Occult blood has been found in his stool. His father also gets frequent, nonspecific stomachaches. What si the likely diagnosis?

*PUD* can result in hematemesis and melena, along with the typically epigastric abdominal pain. Children can have both chronic and acute blood loss associated with ulceration. Fiberoptic endoscopy is the diagnostic method of choice. An upper GI series can sometimes reveal an ulcer as well. While H pylori serum assays are available, they have limited usefulness in children.

A 10-year-old boy has complained for 1 month of intermittent epigastric pain that awakens him from sleep. He notes that eating food sometimes helps. He reports black stools during the prior week, and also admits that he has occasionally vomited frank blood. What is the diagnosis and workup?

*Leptospirosis* is the most common zoonotic infection worldwide, and is often a mild or subacute illness, frequently escaping detection. Usually a history of exposure to dogs, cats, livestock, rats, or other wild animals is obtained. Two distinct courses of infection are described: "anicteric" leptospirosis and "icteric" leptospirosis (also called Weil syndrome). Both courses start with similar symptoms, as described in the case vignette, termed the "septicemic" phase. The majority of cases are anicteric, and after a few days of symptom resolution, patients go on to the "immune" phase in which meningitic symptoms return and can last up to a month. Between 50% and 90% of cases have meningeal involvement. Less than 10% of leptospirosis cases are "icteric," but these patients go on to have more severe symptoms involving liver and kidney dysfunction. Penicillin and tetracycline (in children 10 years and older) are appropriate treatments, and seem to shorten the duration of the illness if started in the first week of symptoms. Some recommend prophylaxis with weekly doxycycline in endemic areas of the world.

A 10-year-old boy was healthy until about 10 days ago when he developed 7 days of fever, chills, severe muscle pain, pharyngitis, headache, sclerai injection, photophobia, and cervical adenopathy. After 7 days of symptoms, he seemed to get better, but yesterday he developed fever, nausea, emesis, headache, and mild nuchal rigidity. Cerebrospinal fluid (CSF) shows 200 white blood cells (WBCs) per microliter (all monocytes) and an elevated protein. What is the likely diagnosis?

*Cushing syndrome,* most likely from bilateral adrenal hyperplasia.

A 10-year-old child weighs 55 kg (> 99 percentile; 50 percentile for a 14 year old), has central fat distribution, is 125-cm tall (5 percentile), has a blood pressure of 120/80 mm Hg, a hematocrit of 17%, and her bone films are read as "osteopenic." What is the likely diagnosis?

maxillary sinusitis

A 10-year-old girl has had a "cold" for 14 days. In the 2 days prior to the visit to your office, she has developed a fever of 39°C (102.2°F), purulent nasal discharge, facial pain, and a daytime cough. Examination of the nose after topical decongestants shows pus in the middle meatus. What is the most likely diagnosis?

*GERD:* medications to treat GERD include acid blockade with H2 blockers or proton pump inhibitors, resulting in decreased esophagitis. Prokinetic agents are frequently used in conjunction with acid blockade for this illness, but have not been consistently shown to decrease symptoms.

A 12-month-old girl has been spitting up her meals since 1 month of age. Her growth is at the 95th percentile, and she is otherwise asymptomatic and without findings on physical examination. What is the likely diagnosis?

*Bartter syndrome* (also known as juxtaglomerular hyperplasia) is an autosomal recessive condition that causes hypokalemia, hypercalciuria, alkalosis, hyperaldosteronism, and hyperreninemia; blood pressure is usually normal. Clinical presentations occurring frequently between 6 and 12 months of age include failure to thrive with constipation, weakness, vomiting, polyuria, and polydipsia. Treatment is aimed at preventing dehydration, providing nutritional support, and returning the potassium level to normal.

A 12-month-old girl whose height and weight are less than the fifth percentile; she has had several bouts of constipation and two previous admissions for dehydration. She is again admitted for dehydration and is noted to have serum potassium of 2.7 mEq/L. What is the likely diagnosis?

The majority of all cases of acute scrotal pain and swelling in boys 12 years of age and older are caused by testicular torsion. If surgical exploration occurs within 4 to 6 hours, the testes can be saved 90% of the time. Too often, delay caused by scheduling imaging and laboratory tests, such as those outlined in the question, results in an unsalvageable gonad. You should call a surgical consult immediately.

A 12-year-old boy comes to the emergency department at midnight with a complaint of severe scrotal pain since 7 pm. There is no history of trauma. What should you do?

*Tension headaches* are common in the older child and adolescent. They will worsen during the day, and may worsen with stressful situations like tests. They are typically described as squeezing, but are not usually pulsatile. Nausea and vomiting are not typical

A 12-year-old boy has chronic headache that worsens during the school day. These headaches are not associated with nausea or emesis, and he does not have any symptoms prior to the headache. What is the likely diagnosis?

Check serum thyroid function tests.

A 12-year-old girl has a solitary thyroid nodule found on routine examination; she has no symptoms. What is the most appropriate next step for this patient?

The earlier the permanent teeth are reimplanted, the greater the rate of success, decreasing from 90% in the first 30 minutes to 5% after 2 hours. The rate of success is a function of the integrity of the periodontal ligament. The teeth can be rinsed in cold water, but not brushed (to avoid damaging the root and periodontal ligament). Milk and saline are good transport media if the child is uncooperative or for some other reason the teeth cannot be reinserted at the scene. Teeth may also be transported in the mouth of the older, cooperative patient or the parent. The immediate application of acrylic splints is needed to keep the teeth in place, so urgent dental attention is required.

A 12-year-old girl was hit in the face by an errant softball pitch while batting and has had her mandibular incisors knocked out. What is the best plan of action?

*Gaucher disease* is characterized by β-glucocerebrosidase deficiency, which causes an abnormal accumulation of glucocerebroside in the reticuloendothelial system. Bone marrow aspirate shows the typical Gaucher cells engorged with glucocerebroside. Replacement of marrow with these cells leads to anemia, leukopenia, and thrombocytopenia. The liver and spleen can also be involved. Serum acid phosphatase is elevated. X-ray evaluation demonstrates an Erlenmeyer-flask appearance of the long bones. The diagnosis of Gaucher disease is confirmed by the absence of glucocerebrosidase activity in leukocytes, in cultured skin fibroblasts, and in liver cells. Prenatal diagnosis by enzyme analysis is now possible.

A 12-year-old healthy girl has some dizziness while at synagogue. At the outside ER where she is seen testing, shows her to have a hemoglobin of 8 mg/dL, a white blood cell (WBC) count of 4000/μL, and a platelet count of 98,000/μL. Physical examination reveals an enlarged spleen. Her urine pregnancy test is negative, as are her chest radiographs and EKG. As she was no longer dizzy, she was discharged home to follow up with you. She arrives 3 days later having sustained an injury to her thigh. You obtain a radiograph of her femur described as "appearing to be an Erlenmeyer flask". What is the diagnosis?

The child in the case likely has urolithiasis. While *renal stones* in children are relatively uncommon in the United States and are usually related to metabolic abnormalities (where they are twice as common in boys), they are endemic in southeast Asia and related to diet. Symptoms of a stone in the renal pelvis, calyx, or ureter cause obstruction and the symptoms presented in the case. A stone in the distal ureter results in symptoms of dysuria, urgency, and frequency. Once in the bladder the stone is asymptomatic unless it moves into the urethra, then dysuria and voiding difficulties may arise. While some stones may be visible via plain abdominal radiograph, a *non-contrast spiral CT* scan will accurately diagnose the number and location of the stone as well as assist in confirming if the affected kidney is hydronephrotic. Once the child's condition has stabilized (the stone passed or retrieved), a metabolic evaluation is undertaken in an attempt to identify the cause of the stone.

A 14-year-old Asian boy arrives via ambulance to your emergency room from the local international airport. He developed severe, intermittent abdominal pain that radiates into his scrotum about half-way through his 12-hour flight. He reports no fever, some dysuria, and he thinks he may have blood in his urine. What should you do?

infectious mononucleosis

A 14-year-old boy is seen in the ER because of a 3-week history of fever between 38.3°C and 38.9°C (101°F and 102°F), lethargy, and a 2.7kg (6-lb) weight loss. Physical examination reveals marked cervical and inguinal adenopathy, enlarged tonsils with exudate, small hemorrhages on the soft palate, a WBC differential that has 50% lymphocytes (10% atypical), and a palpable spleen 2 cm below the left costal margin. What is the diagnosis?

*Hypothermia* can develop in any cold weather exposure. As the core temperature drops, the individual becomes lethargic, tired, uncoordinated, apathetic, mentally confused, irritable, and bradycardic.

A 14-year-old boy on a mountain-climbing expedition in December becomes tired, clumsy, and begins to hallucinate. His heart rate is 45 beats per minute. What should you be concerned about?

*Crohn disease:* the presentation of Crohn disease (granulomatous colitis) depends on the location and extent of lesions. Onset of the GI or extraintestinal symptoms can be insidious. The "textbook" presentation is as described, although only 25% of patients have the "triad" of diarrhea, weight loss, and abdominal pain. Crohn disease characteristically is associated with transmural, granulomatous intestinal lesions that are discontinuous and can appear in both the small and large intestines. Although Crohn disease can first appear as a rectal fissure or fistula, the rectum is often spared. Arthritis/arthralgia occurs in a minority of affected children. Other extraintestinal symptoms include erythema nodosum or pyoderma gangrenosum, liver disease, renal calculi, uveitis, anemia, specific nutrient deficiency, and growth failure. In relation to the general population, the risk of colonic carcinoma in affected persons is increased, but not to the degree associated with ulcerative colitis.

A 14-year-old girl has a 9-month history of diarrhea, periumbilical and postprandial abdominal pain, fever, and weight loss. Her menses are now irregular. She has had several episodes of blood in her stools. What is the likely diagnosis?

pseudoseizure

A 14-year-old girl with a history of seizures is admitted to the hospital with the diagnosis of status epilepticus. Her valproic acid level is in the therapeutic range. You arrange a 24-hour video electroencephalogram (EEG). During the EEG, she has several episodes of tonic and clonic movements with moaning and crying, with no loss of bowel or bladder control. The neurologist tells you that during the events the EEG had excessive muscle artifact but no epileptiform discharges. What is the most likely diagnosis?

The clinical presentation is that of a *tricyclic antidepressant ingestion*. In smaller children, the central nervous system (CNS) symptoms of drowsiness, lethargy, coma, and seizures are more commonly seen than the cardiac effects of tachycardia, initial hypertension followed by hypotension, widening of the QRS complex and ventricular dysrhythmias, which are often seen in adolescents. While therapy is mostly supportive, patients with a QRS complex wider than 100 ms, intractable hypotension, or ventricular dysrhythmias are candidates for alkalinization with an initial 1-2 mEq/kg of *sodium bicarbonate* followed by a continuous infusion, with the goal serum pH of 7.45 to 7.55.

A 14-year-old girl, angry at her mother for taking away her MP3 player, takes an unknown quantity of a friend's pills. Within the first hour she is sleepy, but in the emergency center she develops hypotension and a widened QRS complex of 130 ms on her ECG. What is the diagnosis and treatment?

*Prolonged QT syndrome* occurs in 1:10,000 to 1:15,000 children, usually first causing syncopal episodes in late childhood or adolescence. During the syncope episode, arrhythmias may be noted, including ventricular fibrillation. These episodes may result in death. QT intervals are elongated on ECG. An autosomal recessive form associated with deafness (Jervell-Lange-Nielsen syndrome) and an autosomal dominant form (Romano-Ward syndrome) have been described.

A 15-year-old adolescent female comes to be evaluated for syncopal episodes. Her only other medical problem is congenital deafness. She notes through a signing interpreter that syncopal episodes happen during stressful or emotional situations and that they started only within the past year or so. Her mother and father are from the same small farming town and are second cousins. What is the diagnosis?

HCM

A 15-year-old adolescent male presents to the office for a sports physical. In his screening questionnaire, he notes that he occasionally gets short of breath and dizzy during exercise, with occasional chest pain. He lost consciousness once last summer during football practice, but attributed it to the heat. His grandfather died suddenly at the age of 35 of unknown etiology, but otherwise the family is healthy. What is the likely diagnosis?

*Choanal atresia:* most neonates are obligate nose breathers and cannot breathe adequately through their mouths. Infants with choanal atresia have increased breathing difficulty during feeding and sleeping and improved respirations when crying. A variety of temporizing measures to maintain an open airway have been used, including oropharyngeal airways, positioning, tongue fixation, and endotracheal intubation, but surgical correction with placement of nasal tubes is most effective. The diagnosis can be made by failure to pass a catheter through the nose to the pharynx or by checking for fog developing on a cold metal instrument placed under each naris.

A 2-hour-old full-term newborn infant is noted by the nursing staff to be having episodes of cyanosis and apnea. Per nursery protocol they place an oxygen saturation monitor on him. When they attempted to feed him, his oxygen saturation drops into the 60s. When he is stimulated and cries, his oxygen levels increase into the 90s. What is the likely diagnosis and what intervention will help?

Collect urine for measurement of the calcium to creatinine ratio. Hypercalcemia can develop in children who are immobilized following the fracture of a weight-bearing bone. Serious complications of immobilization hypercalcemia, and the hypercalciuria that occurs as a result, include nephropathy, nephrocalcinosis, hypertensive encephalopathy, and convulsions. The early symptoms of hypercalcemia—namely, constipation, anorexia, occasional vomiting, polyuria, and lethargy—are nonspecific and may be ascribed to the effects of the injury and hospitalization. Therefore, careful monitoring of these patients with serial measurements of the serum-ionized calcium and the urinary calcium to creatinine ratio is critical during their immobilization. A ratio of greater than 0.2 establishes a diagnosis of hypercalciuria. Although complete mobilization is curative, additional measures, such as vigorous intravenous hydration with a balanced salt solution, dietary restrictions of dairy products, and administration of diuretics, can be instituted. For patients who are at risk for symptomatic hypercalcemia, short term therapy with calcitonin is highly effective in reducing the concentration of serum calcium by inhibiting bone resorption.

A 15-year-old boy has been immobilized in a double hip spica cast for 6 weeks after having fractured his femur in a skiing accident. He has become depressed and listless during the past few days and has complained of nausea and constipation. He is found to have microscopic hematuria and a blood pressure of 150/100 mm Hg. What should you do?

*Vascular headaches (migraines)* can occur in all ages, and patients usually have a family history of migraine. While the typical scotomata discussed in adult migraine is not normally associated in children with migraine, pediatric migraines may have a nonspecific prodrome consisting of a change in mood, temperament, or appetite.

A 15-year-old girl has an acute, recurrent, pulsatile headache localized behind the eyes that tends to occur more frequently around menses. She has no symptoms that occur prior to the headache; her neurologic examination is normal. What is the likely diagnosis?

This patient has glomerular disease, fever, malaise, and weight loss as well as the arthritis involving mainly small joints; these are common findings of *systemic lupus erythematosus (SLE).* Raynaud phenomenon resulting in digital ulceration and gangrene in a few patients may also be seen. Not described in this patient is the oft-seen malar rash in a butterfly distribution across the bridge of the nose and the cheeks.

A 15-year-old girl has had intermittent fever, malaise, and weight loss over the previous several months. Recently she has developed swollen hands, wrists, and ankles, the pain of which seems out of proportion to the clinical findings. She also complains of cold extremities and has some ulceration of her distal digits. Her urine specimen has RBC casts. What is the diagnosis?

Listeria monocytogenes

A 16-day-old infant presents with fever, irritability, poor feeding, and a bulging fontanel. Spinal fluid demonstrates Gram-positive rods. What is the most likely causative organism?

*Toxic shock syndrome (TSS)* is usually caused by S aureus, but a similar syndrome (sometimes called *toxic shock-like syndrome [TSLS]*) may be caused by Streptococcus sp. The strains of S aureus responsible secrete toxic shock syndrome toxin 1 (TSST-1), and can cause "menstrual" TSS (associated with intravaginal devices like tampons, diaphragms, and contraceptive sponges) or "nonmenstrual" TSS associated with pneumonia, skin infection (as in this patient), bacteremia, or osteomyelitis. The diagnosis is made clinically, and the case description is typical. Treatment includes blood cultures followed by aggressive fluid resuscitation and antibiotics targeting S aureus.

A 16-year-old boy presents to the emergency center with a 2-day history of an abscess with spreading cellulitis. While in the emergency center, he develops a high fever, hypotension, and vomiting with diarrhea. On examination, you note the erythematous rash demonstrated in the picture along with injected conjunctiva and oral mucosa, and a strawberry tongue. He is not as alert as when he first arrived. What is the likely diagnosis?

*Vitamin C* deficiency impairs wound healing. In its severe form, also termed scurvy, children can have diffuse tenderness, which is worse in the legs; evidence of hemorrhage; irritability; low-grade fever; swelling; tachypnea; and poor appetite. Diagnosis is based on clinical picture and radiographic findings; if laboratory studies are desired, a plasma ascorbate level < 0.2 mg/dL suggests deficiency.

A 17-month-old toddler has been irritable over the past month. She now refuses to walk and seems to have tenderness in both of her legs. She has had a low-grade fever, and she has petechiae on her skin and mucous membranes. She has a small cut that has not healed well. Radiographs of the legs reveal generalized bony atrophy with epiphyseal separation. What vitamin supplementation is necessary?

Get a *RUQ ultrasound.* Cholecystitis and cholelithiasis are unusual diseases in children and are almost always associated with predisposing disorders such as hemolytic anemia, pregnancy, CF, Crohn disease, obesity, rapid weight loss, or prior ileal resection. Pain of the right upper quadrant, nausea, vomiting, fever, and jaundice are symptoms of *acute cholecystitis.* The diagnosis is confirmed with an ultrasound of the gallbladder.

A 17-year-old adolescent female is 6 weeks postpartum. She presents to the emergency room with the complaints of increased jaundice, abdominal pain, nausea, vomiting, and fever. Her examination is remarkable for jaundice, pain of the right upper quadrant with guarding, and a clear chest. Chest radiographs appear normal. What is the next best step?

The patient in the question has a classic description of *Goodpasture disease*, a rare disease in children. The pulmonary hemorrhage can be life threatening and the renal impairment progressive. Diagnosis is suggested by finding hemosiderin-laden macrophages on sputum or pulmonary secretions and the finding of antibodies to the glomerular basement membrane; a kidney biopsy may be required as well.

A 17-year-old boy is brought to the emergency department by his parents with the complaint of coughing up blood. He is stabilized, and his hemoglobin and hematocrit levels are 11 mg/dL and 33%, respectively. During his hospitalization, he is noted to have systolic blood pressure persistently greater than 130 mm Hg and diastolic blood pressure greater than 90 mm Hg. His urinalysis is remarkable for hematuria and proteinuria. What is the likely diagnosis?

*Tuberculous meningitis* would be the most likely cause of meningitis to linger for 3 weeks; other infectious causes of meningitis would lead to rapid deterioration.

A 2-year-old boy has been vomiting intermittently for 3 weeks and has been irritable, listless, and anorectic. He "feels warm" to his parents. His use of language has regressed to speaking single words. What should be on your differential?

*Kawasaki:* many conditions can be associated with prolonged fever, a limp caused by arthralgia, exanthem, adenopathy, and pharyngitis. Conjunctivitis, however, is suggestive of Kawasaki disease. Kawasaki disease typically presents with prolonged fever, a polymorphous rash, extremity changes (including swelling in the early period of the illness with peeling from the fingertips coming later), nonpurulent conjunctivitis, lymphadenopathy, and oral mucosal changes including fissured lips and oropharyngeal mucosal erythema. Although these are considered the diagnostic criteria, the diagnosis is still possible in the absence of one or two of these physical findings. Other clinical findings may include vomiting and diarrhea with abdominal pain, hydrops of the gall bladder, arthritis or arthralgias, irritability and aseptic meningitis. Coronary artery aneurysms can develop, as can aneurysms in other areas. Initial treatment is typically intravenous immunoglobulin (IVIG) and highdose aspirin. The child will usually defervesce shortly after the infusion. Aspirin is typically kept at a higher dose until the platelet count begins to decrease, and then is continued at a lower dose for several weeks.

A 2-year-old boy is brought into the emergency room with a complaint of fever for 6 days and the development of a limp. On examination, he is found to have an erythematous macular exanthem over his body, conjunctivitis, dry and cracked lips, a red throat, and cervical lymphadenopathy. There is a grade 2/6 vibratory systolic ejection murmur at the lower left sternal border. A white blood cell (WBC) count and differential show predominant neutrophils with increased platelets on smear. What is the likely diagnosis?

Meckel diverticulum

A 2-year-old boy presents to the emergency center with several days of rectal bleeding. The mother first noticed reddish-colored stools 2 days prior to arrival and has since changed several diapers with just blood. The child is afebrile, alert, and playful, and is eating well without emesis. He is slightly tachycardic, and his abdominal examination is normal. What is the likely diagnosis?

1) Kawasaki disease 2) scarlet fever

A 2-year-old child is admitted to your hospital team. The child's primary care doctor has been following the child for several days and has noted her to have had high fever, peeling skin, abdominal pain, and a bright red throat. What *two* conditions are on your differential here?

These patients likely have *scabies* and should be treated with *permethrin*

A 2-year-old child presents with a 4-day history of a rash limited to the feet and ankles. The papular rash is both pruritic and erythematous. The 3-month-old sibling of this patient has similar lesions also involving the head and neck. What is the appropriate treatment?

*Septic arthritis* (in this case, of the hip) requires urgent intervention to preserve joint mobility. Joint aspiration is diagnostic and can be helpful in treatment. Opening the joint space may be required in a septic hip to assist in draining purulent material. These children need treatment for 4 to 6 weeks.

A 2-year-old child refuses to walk, has fever, has significant pain with external rotation of the right leg, and has an elevated WBC count. What is one diagnosis on your differential?

*Ebstein anomaly:* a quadruple rhythm associated with the murmur of tricuspid regurgitation and a middiastolic murmur at the lower left sternum suggests the diagnosis of Ebstein anomaly (downward displacement of the tricuspid valve). The presence of right atrial hypertrophy and right ventricular conduction defects confirms the diagnosis.

A 2-year-old child with minimal cyanosis has an S3 and S4 (a quadruple rhythm), a systolic murmur in the pulmonic area, and a middiastolic murmur along the lower left sternal border. An ECG shows right atrial hypertrophy and a ventricular block pattern in the right chest leads. What is the most likely diagnosis?

Hydrocarbons with low viscosity and high volatility are the most likely agents to cause respiratory symptoms. Gasoline, kerosene, and furniture polish are common causes of *hydrocarbon aspiration.* Symptoms are usually seen within 6 hours of aspiration, and include dyspnea, cyanosis, and respiratory failure. Treatment is symptomatic, sometimes requiring intubation and mechanical ventilation, and your initial step of management should be to *perform pulse oximetry and obtain arterial blood gases.* Induction of emesis is contraindicated, as this may cause further aspiration.

A 2-year-old girl is playing in the garage with her Chihuahua, only partially supervised by her father, who is weed-whacking around the garden gnomes in the front yard. He finds her in the garage, gagging and vomiting. She smells of gasoline. In a few minutes she stops vomiting, but later that day she develops cough, tachypnea, and subcostal retractions. She is brought to your emergency center. What is the diagnosis and management?

The most common type of hereditary nephritis is *Alport syndrome.* Clinically, patients present with asymptomatic microscopic hematuria, but gross hematuria is also possible, especially after an upper respiratory infection. Hearing loss, eventually leading to deafness, is associated with Alport syndrome in up to 75% of cases. End-stage renal disease, as hinted in the question (the father was at a dialysis center), is common by the second or third decade of life. This syndrome is mostly commonly an X-linked dominant disorder, which explains the more severe course in males. Other findings include ocular abnormalities (30% to 40%) and, rarely, leiomyomatosis of the esophagus or respiratory tree.

A 2-year-old patient arrives late to your office with her father and a sign-language translator. They are very apologetic, but the father communicates that they had car trouble at his dialysis center and thus was late picking up the child from day care. The father is concerned about his child's having intermittent red, bloody-looking urine, especially after an upper respiratory infection. A gross inspection of the child's urine in your office looks normal, but the dipstick demonstrates 3+ blood. What is the diagnosis?

congenital rubella

A 21-year-old woman has just delivered a term infant. She has had only one visit to her obstetrician, and that was at about 6 weeks of pregnancy. She provides her laboratory results from that visit. The delivered infant is microcephalic, and has cataracts, a heart murmur, and hepatosplenomegaly. Your further evaluation of the child demonstrates thrombocytopenia, mild hemolytic anemia, and, on the echocardiogram, patent ductus arteriosus and peripheral pulmonary artery stenosis. What is the likely diagnosis?

acute lymphoblastic leukemia

A cancer commonly seen in boys age 2-5. It presents with anemia, thrombocytopenia, and normal or elevated WBC count. The presence of blasts on peripheral smear and bone marrow biopsy is characteristic.

Hodgkin's disease

A cancer of the antigen processing cells found within the lymph nodes or spleen. Associated with *EBV infection.* It often presents with *painless lymphadenopathy*, most commonly in the supraclavicular or cervical regions.

The presentation described is characteristic of *visceral larva migrans* from infestation with a common parasite of dogs, *Toxocara canis.* Dirt-eating children ingest the infectious ova. The larvae penetrate the intestine and migrate to visceral sites, such as the liver, lung, and brain, but do not return to the intestine, so the stools do not contain the ova or parasites. The diagnosis can be made by a specific ELISA for Toxocara. Patients with minimal symptoms may be managed expectantly, while patients with more significant symptoms may be treated with albendazole. Some experts recommend concomitant therapy with steroids to decrease the inflammatory response from dying parasites.

A 4-year-old boy was admitted to the hospital last night with the complaint of "difficulty breathing." He has no past history of lung infection, no recent travel, and no day-care exposure; he does, however, have an annoying tendency to eat dirt. In the emergency center, he was noted to be wheezing and to have hepatomegaly. He is able to talk, relaying his concern about his 6-month-old Chihuahua being left alone at home. Laboratory studies revealed leukocytosis with marked eosinophilia (60% eosinophils). What is the likely diagnosis?

The patient with the features listed likely has pseudohypoparathyroidism (*Albright hereditary osteodystrophy*). Such patients have chemical findings of hypoparathyroidism (low calcium, high phosphorus), but parathyroid hormone levels are high, indicating resistance to the action of this hormone. Parathyroid hormone infusion does not produce a phosphaturic response. Phenotypically, these patients demonstrate shortness of stature with delayed bone age, mental retardation, increased bone density throughout the body (especially evident in the skull), brachydactyly (especially of the fourth and fifth digits), obesity with round facies and short neck, subcapsular cataracts, cutaneous and subcutaneous calcifications, and perivascular calcifications of the basal ganglia.

A 4-year-old child has mental retardation, shortness of stature, brachydactyly (especially of the fourth and fifth digits), and obesity with round facies and short neck. The child is followed by an ophthalmologist for subcapsular cataracts, and has previously been noted to have cutaneous and subcutaneous calcifications, as well as perivascular calcifications of the basal ganglia. What is the likely diagnosis?

myasthenia gravis

A 4-year-old child is observed to hold his eyelids open with his fingers and to close one eye periodically, especially in the evening. He has some trouble swallowing his food. He usually appears sad, although he laughs often enough. He can throw a ball, and he runs well. What is the most likely diagnosis?

mumps

A 4-year-old child presents in the clinic with an illness notable for swelling in front of and in back of the ear on the affected side, as well as altered taste sensation. What is the diagnosis?

G6PD deficiency

A 4-year-old previously well African American boy is brought to the office by his aunt. She reports that he developed pallor, dark urine, and jaundice over the past few days. He stays with her, has not traveled, and has not been exposed to a jaundiced person, but he is taking trimethoprim sulfamethoxazole for otitis media. The CBC in the office shows a low hemoglobin and hematocrit, while his "stat" serum electrolytes, blood urea nitrogen (BUN), and chemistries are remarkable only for an elevation of his bilirubin levels. His aunt seems to recall his 8-year-old brother having had an "allergic reaction" to aspirin, which also caused a short-lived period of anemia and jaundice. What is the likely diagnosis?

pseudohypoparathyroidism

A 5-day-old LGA girl with brachydactyly, round facies, and short neck might have what diagnosis?

The patient in the question has *acute lobar nephronia*, which is in the middle of the spectrum between pyelonephritis and renal abscess. Patients have prolonged fever curves despite appropriate antibiotics. A CT scan is most useful in diagnosing nephronia, but a renal ultrasound can identify the process as well. Treatment is prolonged IV and then PO antibiotics.

A 5-year-old girl without past history of UTI is in the hospital on antibiotics for E coli pyelonephritis. She is still febrile after 4 days of appropriate antibiotics. A renal ultrasound revealed no abscess, but a focal enlargement of one of the lobes of the right kidney. CT scan of the abdomen reveals a wedge-shaped area in the right kidney distinct from the normal tissue with minimal contrast enhancement. What is the diagnosis and treatment?

*Laurence-Moon-Biedl* (Bardet-Biedl) syndrome is transmitted as an autosomal recessive trait. Obesity, mental retardation, hypogonadism, polydactyly, and retinitis pigmentosa with night blindness are the principal findings in affected children. There is no known effective treatment.

A 5-year-old obese boy with mental retardation and polydactyly likely has what disease?

*Prader-Willi syndrome* is a disorder consisting of hypotonia, hypogonadism, hyperphagia after the newborn period, hypomentia, and obesity. A deletion of a portion of chromosome 15 has been found in approximately 70% of patients. Children affected by this syndrome exhibit little movement in utero and are hypotonic during the neonatal period. Feeding difficulties and failure to thrive can be the presenting complaints in the first year; later, obesity becomes the most common presenting complaint. The enormous food intake of affected children is thought to be caused by a defect in the satiety center in the hypothalamus. Stringent caloric restriction is the only known treatment.

A 6-day-old LGA infant with severe hypotonia and poor feeding since birth might have what diagnosis?

The patient in the question likely has *hereditary fructose intolerance*, manifest only when fructose in fruit juice is provided in the diet.

A 6-month-old infant has been exclusively fed a commercially available infant formula. Upon introduction of fruit juices, however, the child develops jaundice, hepatomegaly, vomiting, lethargy, irritability, and seizures. Tests for urine-reducing substances are positive. What is the diagnosis?

atrial septal defect

A cardiac defect that can be found at any age. Characterized by fixed wide split S2. Closure achieved via catheter directed device closure.

necrotizing enterocolitis

A cause of GI bleed in premature babies. It will present with *bloody diarrhea in premature infants.* An x-ray or babygram will show *pneumatosis intestinalis* (air in the bowel wall), which confirms the diagnosis. Baby should be NPO immediately and get started on TPN and IV abx. Hold off from surgery unless there's no improvement or conditions worsen.

congenital CMV

A developmentally delayed 6-month-old child had intrauterine growth retardation (including microcephaly), hepatosplenomegaly, prolonged neonatal jaundice, and purpura at birth. There are periventricular calcific densities seen on skull x-ray. What is the likely diagnosis?

The nonspecific findings of anorexia, polydipsia and polyuria, vomiting, and unexplained fevers, along with the more specific laboratory abnormalities of glucosuria but normal blood sugar, abnormally high urine pH in the face of mild or moderate serum hyperchloremic metabolic acidosis, and mild albuminuria in the presence of normal serum protein and albumin, suggest *Fanconi syndrome.* Fanconi syndrome can be hereditary or acquired; hereditary forms are usually secondary to a genetic abnormality such as cystinosis, galactosemia, Wilson disease, and some mitochondrial abnormalities. A number of agents can cause Fanconi syndrome, including gentamicin (or other aminoglyco-sides), outdated tetracycline, cephalothin, cidofovir, valproic acid, strepto-zocin, 6-mercaptopurine, azathioprine, cisplatin, ifosfamide, heavy metals (eg, lead, mercury, cadmium, uranium, platinum), paraquat, maleic acid, and toluene (from sniffing glue). The mechanism of action of these agents is through acute tubular necrosis, alteration of renal blood flow, intratubu-lar obstruction, or allergic reactions within the kidney itself. Many of these toxic effects are reduced or eliminated with removal of the offending agent.

A 6-month-old infant has poor weight gain, vomiting, episodic fevers, and chronic constipation. Laboratory studies reveal a urinalysis with a pH of 8.0, specific gravity of 1.010, 1+ glucose, and 1+ protein. Urine anion gap is normal. Serum chemistries show a normal glucose and a normal albumin with a hyperchloremic metabolic acidosis. Serum phosphorus and calcium are low. What is the diagnosis?

Chlamydiae organisms, sexually transmitted among adults, are spread to infants during birth from genitally infected mothers. The sites of infection in infants are the conjunctivae and the lungs, where chlamydiae cause inclusion conjunctivitis and afebrile pneumonia, respectively, in infants between 2 and 12 weeks of age. Diagnosis is confirmed by culture of secretions and by antibody titers.

A 6-week-old child arrives with a complaint of "breathing fast" and a cough. On examination you note the child to have no temperature elevation, a respiratory rate of 65 breaths per minute, and her oxygen saturation to be 94%. Physical examination also is significant for rales and rhonchi. The past medical history for the child is positive for an eye discharge at 3 weeks of age, which was treated with a topical antibiotic. What is the diagnosis?

*Legg-Calvé-Perthes* disease is avascular necrosis or idiopathic osteonecrosis of the femoral head; the cause of this disorder is unknown. Boys between the ages of 2 and 12 years are most frequently affected (incidence in boys is four- to fivefold greater than in girls), with a mean of 6 to 7 years old. Presenting symptoms include a limp and pain in the anterior thigh, groin, or knee, although classic symptoms include a painless limp.

A 6-year-old boy has developed a limp and has limited mobility of the hip, but denies pain and fever. What might you suspect?

The findings of poor growth, diabetes insipidus, and papilledema could be explained by a *craniopharyngioma.* This tumor is one of the most common supratentorial tumors in children, often causing growth failure through disruption of pituitary excretions such as growth hormone. Upward growth of a craniopharyngioma results in compression of the optic chiasm. Particularly affected are the fibers derived from the nasal portions of both retinas (ie, from those parts of the eyes receiving stimulation from the temporal visual field). Early in the growth of a craniopharyngioma, a unilateral superior quadrantanopsic defect can develop, and an irregularly growing tumor can impinge upon the optic chiasm and cause homonymous hemianopia. Treatment is surgical excision and may include radiation therapy for large lesions. However, significant morbidity is associated with these tumors and their removal, including vision loss, growth failure, and panhypopituitarism.

A 6-year-old boy is seen in the office for evaluation of polyuria. Further questioning reveals several months of headache with occasional emesis. Your physical examination reveals a child who is less than 5% for weight. He has mild papilledema. His glucose is normal, and his first urine void specific gravity after a night without liquids is 1.005 g/mL. Bilateral hemianopsia is noted. What is the likely diagnosis?

*Chilblains* are small, ulcerated lesions on exposed areas such as the ears and fingers. Lesions may last 1 to 2 weeks.

A 6-year-old boy returns from playing all day in the snow with several erythematous, ulcerative lesions on his fingertips; he complains the lesions are painful and itchy. What is the likely diagnosis?

Sydenham chorea

A 6-year-old boy with emotional lability, poor school performance, and milkmaid's grip (irregular contractions of the hand muscles when squeezing the examiner's fingers). What is the diagnosis?

In addition to the findings listed in the question, patients experiencing *malignant hyperthermia* (MH) also have clinical findings of tachycardia, arrhythmia, tachypnea, and cyanosis, as well as laboratory findings of myoglobinuria, elevated serum creatine kinase levels, and evidence of acute renal failure. This myopathy is usually inherited as an autosomal dominant trait; the gene is on chromosome 19 and codes for the ryanodine receptor, a calcium release channel. A family history of similarly affected relatives would suggest the need to evaluate all family members for this condition; prevention (or treatment) is with *dantrolene* sodium. The test of choice to identify a patient at risk for this condition is the caffeine contracture test, in which a muscle biopsy tissue specimen is attached to a strain gauge and then exposed to caffeine. Patients at risk for MH have a diagnostic muscle spasm. Ryanodine receptor (RYR1) gene sequencing is also available.

A 6-year-old child has had repeated episodes of otitis media. She undergoes an uneventful surgical placement of pressure-equalization (PE) tubes. In the recovery room, she develops a fever of 40°C (104°F), rigidity of her muscles, dark colored urine, and metabolic and respiratory acidosis. What is the likely diagnosis?

With *acute glomerulonephritis*, oliguria (often presenting with dark, cola-colored urine) frequently occurs as a direct consequence of the disease process itself; on occasion, it can be profound, with virtual anuria for several days. During this period of time, it is vital to monitor and restrict fluid intake lest massive edema, hypervolemia, and even pulmonary edema and death occur

A 6-year-old girl with a complaint of "dark urine"; she has a blood pressure of 120/80 mm Hg. What is the likely diagnosis?

*Intussusception:* unlike adults, kids don't have a "lead point" as a cause. US is sensitive for intussusception and can be used to track resolution. If diagnosed, jump straight to air-contrast barium enema, which usually cures.

A cause of GI bleed that results from part of the bowel telescoping into another, compromising the blood supply. Causes an abrupt onset of colicky abdominal pain in an otherwise healthy baby. Usually occurs in kids 3 months to 3 years. Kids will usually assume the knee-chest position for pain relief, and a sausage-shaped mass can be felt in the abdomen. Currant jelly diarrhea can be seen.

Salt-losing congenital adrenal hyperplasia (adrenogenital syndrome, 21-hydroxylase deficiency) usually manifests during the first 5 to 15 days of life as anorexia, vomiting, diarrhea, and dehydration. Hypoglycemia can also occur. Affected infants can have increased pigmentation, and female infants show evidence of virilization, that is, ambiguous external genitalia. Hyponatremia, hyperkalemia, and urinary sodium wasting are the usual laboratory findings. Death can occur if the diagnosis is missed and appropriate treatment is not instituted. In classic 21-hydroxylase deficiency, serum levels of 17-hydroxyprogesterone are markedly elevated beyond 3 days of life (in the first 3 days of life they can normally be high). Blood cortisol levels are usually low in salt-losing forms of the disease.

A 7-day-old boy is admitted to a hospital for evaluation of vomiting and dehydration. This is the third child for this family, and the prenatal history was uncomplicated. Physical examination on the child is normal except for some hyperpigmentation of the nipples. Serum sodium and potassium concentrations are 120 mEq/L and 9 mEq/L (without hemolysis), respectively; serum glucose is 35 mg/dL. What is the diagnosis?

Forceful emesis can result in small tears in the esophagus, termed Mallory-Weiss syndrome. This is usually a benign condition, only occasionally resulting in significant blood loss. In a patient who is otherwise stable, diagnostic procedures are not indicated.

A 7-year-old has been vomiting for 2 days and has had diarrhea for 1 day. He now notes that he has small streaks of blood in his emesis. The rest of his family has had similar symptoms. What should you do?

Cow's milk contains an insufficient quantity of *iron* to sustain normal RBC production. Therefore, children whose primary caloric source is cow's milk are likely to develop iron-deficiency anemia, characterized by microcytosis and hypochromia on the peripheral smear

A 9-month-old infant, who has been fed cow's milk exclusively for 4 months, is tachycardic and pale. What vitamin do you need to supplement?

*Brain tumors* are the most common solid tumor in childhood, and account for 25% to 30% of all pediatric malignancies. While supratentorial tumors predominate in the first year of life (including choroid plexus tumors and teratomas), brain tumors in children 1 to 10 years old are more frequently *infratentorial (posterior fossa)* and include cerebellar and brainstem tumors such as *medulloblastoma or cerebellar astrocytoma.* After 10 years of age, supratentorial tumors (eg, diffuse astrocytoma) are again more common.

A 9-year-old child has developed headaches that are more frequent in the morning and are followed by vomiting. Over the previous few months, his family has noted a change in his behavior (generally more irritable than usual) and his school performance has begun to drop. Imaging will reveal what abnormality?

Cold panniculitis is destruction of fat cells caused by exposure to cold weather or a cold object; in this case, the child had "Popsicle panniculitis," which is usually a benign condition that self-resolves.

A 9-year-old girl presents during summer break with an area of erythematous, firm, and slightly swollen skin at the corner of her mouth and extending to her cheek. The area is not tender. What is the likely diagnosis?

*Left to right shunts:* ASD VSD PDA

A CV defect caused by connection between high and low pressures, allowing blood to flow from the left ventricle (which is oxygenated) back into the pulmonary circulation. This causes increased vascular markings on CXR. The response to high pressure in the pulmonary circulation is right ventricular hypertrophy with resultant pulmonary HTN. If it persists, there will eventually be a flow reversal (Eisenmenger's) turning noncyanotic lesions into cyanotic ones.

*Pyloric stenosis:* give IV fluids for rehydration, and fix the metabolic derangements before treating definitively with pyloromyotomy.

A baby boy 2-8 weeks of age presents with projectile non-bilious vomiting after feeds. The baby has not had any prior problems. Exam reveals an olive-shaped mass and visible peristaltic waves. A CMP shows hypochloremic, hypokalemic, metabolic alkalosis. What is the diagnosis?

superficial infantile hemangioma (strawberry hemangioma)

A benign capillary tumor formed from abnormally proliferating endothelial cells. Appears during the first few weeks of life as a bright red, sharply demarcated, blanching nodule or plaque. Typically undergoes rapid growth during the first year of life following by spontaneous regression during childhood. Beta blockers are recommended for lesions that are complicated, but most patients require no intervention.

congenital melanocytic nevus

A benign proliferation of melanocyte cells in newborns. Present within the first few months of life as solitary, hyperpigmented lesions with an increased density of overlying dark coarse hairs. Large lesions hold a risk of transformation to melanoma and are often removed surgically; small lesions may be removed for cosmetic reasons given their low risk of melanoma transformation.

erythema toxicum neonatorum

A benign rash seen most frequently in the first 72 hours after birth, characterized by erythematous macules, papules, and pustules (resembling flea bites) on the trunk and extremities but not the palms and soles. Occurs in 50% of full term infants and is found much less frequently in preterm infants. Lesions are filled with eosinophils. No treatment is required.

osteoid osteoma

A benign, bone-forming tumor that occurs in adolescent boys. The femur, tibia, and spine are most commonly affected. Patients develop increasing pain worse at night and unrelated to activity. Pain is *relieved by NSAIDs*, a classic feature strongly suggestive of the diagnosis. X-ray will show a characteristic *small round lucency* with sclerotic mergins. NSAIDs can be given for symptomatic relief, and serial imaging is performed to monitor the lesion.

Staphylococcal scalded skin syndrome

A blistering skin condition most commonly found in infants. Involves infection with Staph aureus strains that produce exfoliative toxins which disrupt keratinocyte adhesion in the superficial epidermis. Presents with fever, generalized erythema followed by flaccid blisters and bullae in the axillae and groin. Nikolsky sing and epidermal shedding will be positive. Pustules may be seen. Treat with nafcillin or vancomycin.

Ewing's sarcoma

A bone tumor found mid-shaft, caused by t(11:22) translocation.

osteogenic sarcoma

A bone tumor which presents with sunburst onion skin pattern, typically at the distal femur. Associated with retinoblastoma

coarctation of the aorta

A congenital heart defect which results in thickening of the tunica media of the aortic arch near the ductus arteriosus. Clinically in infants, it will present with HTN in the upper extremities, decreased perfusion to the lower extremities (decreased femoral pulses, lower extremity claudication), heart failure (irritability, poor feeding, diaphoresis) or cardiogenic shock. Treatment is surgical repair.

choanal atresia

A congenital nasal malformation caused by failure of the posterior nasal passage to canalize completely, leaving either a bony or a membranous obstruction. It presents with cyclic cyanosis that worsens during feeding and is relieved by crying. CT scan will reveal narrowing at the level of the pterygoid plate in the posterior nasal cavity.

nevus flammeus "port wine stain"

A congenital vascular malformation composed of dilated capillary-like vessels (a form of capillary hemangioma) that may be located over the face or trunk and may become darker with increasing postnatal age. Those located in the area of the ophthalmic branch of the trigeminal nerve may be associated with intracranial or spinal vascular malformations, seizures, and intracranial calcifications (*Sturge Weber syndrome*). Usually seen in the newborn.

patent ductus arteriosus

A connection between the aorta and pulmonary artery. Exam will reveal a continuous machine-like murmur. The murmur may not be apparent on day one, but may be noticed on exit exam. In term infants, these are usually no big deal, and most self-resolve within 7 days. In preterm infants, they often need indomethacin or surgery to close or they will cause hemodynamic instability. Use prostaglandins to keep open if needed for a critical heart lesion.

measles (rubeola)

A contagious disease spread by respiratory droplets. There is a characteristic prodrome of fever, cough, coryza, and conjunctivitis. Koplik spots, which are small white lesions found on the buccal mucosa opposite the molars, are pathognomonic. An erythematous blanching maculopapular exanthem characteristically develops on the face and spreads in a cephalocaudal and centrifugal pattern to the rest of the body.

patent ductus arteriosus

A continuous flow murmur is associated with what cardiac defect?

*Depo-Provera* (depomedroxyprogesterone acetate)

A contraceptive injection which involves the slow release of progestin. Allows for contraceptive protection for 3 months after each injection.

amblyopia

A cortical blindness and defect of development. Can occur due to obstruction of vision (eg cataracts) or two competing inputs (strabismus). The brain will "turn off" input from a the abnormal eye. One eye will b enormal while the other is blind. Attempt to treat it by patching the dominant eye, but optimally it should be prevented from happening in the first place.

total proctocolectomy

A curative procedure for UC which is reserved for intractable colitis

Patients with *tricuspid atresia* typically have a hypoplastic right ventricle, and therefore the ECG shows left-axis deviation and LVH; this translates to a left ventricular impulse on physical examination

A cyanotic newborn is suspected of having congenital heart disease. He has an increased left ventricular impulse and a holosystolic murmur along the left sternal border. The ECG shows left-axis deviation and left ventricular hypertrophy (LVH). What is the likely diagnosis?

Reye syndrome

A dangerous complication in children who receive aspirin for virus-induced fever. Hyperammonemia, transaminitis, coagulopathy, nausea, vomiting, hepatomegaly, and mental status changes are typical menifestations, as it causes fulminant hepatic failure and encephalopathy. The presence of *microvesicular steatosis* on liver biopsy is consistent with diagnosis.

congenital adrenal hyperplasia

A deficiency of *21 hydroxylase* which results in inhibited conversion of progesterone to 11-deoxycorticosterone, a precursor to aldosterone. This can cause dehydration (sunken fontanelles, dry mucosa) and salt-wasting (hypotension, hyponatremia, hyperkalemia). Newborn screening allows early detection, and significantly *elevated 17-hydroxyprogesteorne* level confirms diagnosis. Treatment includes glucocorticoids and mineralocorticoid replacement.

Guillain Barre syndrome

A demyelinating polyneuritis characterized by ascending weakness, areflexia, and normal sensation. It is most commonly associated with Campylobacter jejuni, which causes a prodromal gastroenteritis. The principal sites of demyelination are the ventral spinal roots and peripheral myelinated nerves; injury is triggered by a cell mediated immune response to the infectious agent. Ascending symmetric paralysis may progress to resp arrest and cranial nerve involvement.

autoimmune hepatitis

A destructive and progressive liver disease characterized by elevated LFTs, hypergammaglobulinemia, and circulating autoantibodies. Occurs predominantly in females with presentation before puberty. 20-40% of patients have other nonhepatic autoimmune diseases (UC, vasculitis, vitiligo).

Get a *bone marrow biopsy*, as this clinical picture is concerning for leukemia. All of the symptoms in the vignette are typically found with leukemia: clinical and laboratory evidence of marrow failure with anemia and thrombocytopenia. The WBC count can be normal, high, or low. Automated systems initially may report blast forms as atypical lymphocytes.

A father brings his 3-year-old daughter to the emergency center after noting her to be pale and tired and with a subjective fever for several days. Her past history is significant for an upper respiratory infection 4 weeks prior, but she had been otherwise healthy. The father denies emesis or diarrhea, but does report his daughter has had leg pain over the previous week, waking her from sleep. He also reports that she has been bleeding from her gums after brushing her teeth. Examination reveals a listless pale child. She has diffuse lymphadenopathy with splenomegaly but no hepatomegaly. She has a few petechiae scattered across her face and abdomen and is mildly tender over her shins, but does not have associated erythema or joint swelling. A CBC reveals a leukocyte count of 8,000/μL with a hemoglobin of 4 g/dL and a platelet count of 7,000/μL. The automated differential reports an elevated number of atypical lymphocytes. What should be your next step?

generalized <15 minutes in duration single episode in 24 hours no underlying neurologic problems

A febrile seizure must meet all of what criteria in order to be considered simple febrile (no investigation needed) vs complex (investigation needed)

early-onset pauciarticular JIA

A female-predominant subtype of JIA where onset is 1-5 years of age, there are less than 4 joints involved, and patients have a positive ANA and are at high risk for developing chronic uveitis. Uveitis must be monitored by regular slit-lamp evaluation.

fever of unknown origin (FUO)

A fever that lasts longer than 8 days to 3 weeks when prior history, physical exam, and labs have failed to give a diagnosis. It is usually due to a common infection with an unusual presentation, rather than a rare illness, but the differential is extensive. Hospitalization is usually recommended.

Sydenham's chorea

A finding in rheumatic fever. It often begins subtly, months after GABHS pharyngitits. It reflects involvement of the basal ganglia and caudate nuclei. It may start as hand clumsiness and progress to choreoathetoid movements with emoitional lability.

This patient likely has infective endocarditis. Admit directly to the hospital, get two more blood cultures, and start vancomycin. *Infective endocarditis* may present as fulminant disease with shock and overwhelming sepsis, but the subacute form presents with more subtle findings, including low grade fever, weight loss, lethargy, sleep disturbances, arthralgias, and myalgias. The much discussed Osler nodes (small tender nodules in the tips of fingers and toes), Janeway lesions (nontender hemorrhagic lesions on the hands and feet), and splinter hemorrhages (dark lines under nails) are uncommon findings. Children at the greatest risk of endocarditis are those with unrepaired cyanotic heart disease, those with prosthetic material from a repair, or those with a prior history of infectious endocarditis.

A five-month-old child with unrepaired tetralogy of Fallot presents to the clinic with 2 weeks of intermittent low-grade fever, malaise, weight loss, and irritability with movement. His parents have been watching for "tet spells" but have noted no cyanosis. A workup performed at a freestanding clinic the prior day revealed a normal white count, but a blood culture grew Kingella kingae. What is the likely diagnosis?

hydrocele

A fluid collection within the tunica vaginalis (a sac surrounding the testis). Common in neonates; it presents with painless unilateral or bilateral scrotal swelling which may fluctuate with crying or straining due to increased intra-abdominal pressure if it is communicating. Transillumination will show the swelling is cystic and transmits light. This should spontaneously resolve by age 1 year and can be safely observed during that period.

milk protein allergy

A food allergy seen in children around 6 months old. Symptoms such as feeding intolerance, vomiting, failure to thrive, and bloody stool will be the tip-off. Due to cross-reactivity with soy. Treat by avoiding cow's milk protein until 2-3 years of age; use hydrolyzed formula in the interim.

Staph aureus

A foodborne pathogen that causes illness due to ingestion of preformed toxins. Vomiting is the preeminent symptom and usually begins 1-6 hours after exposure. Diarrhea may or may not be present. Dairy items, meats, eggs, produce, and potato salad are common offenders.

talipes equinovarus (clubfoot)

A foot fixed in inversion with no flexibility, usually bilateral. The ankle is held in plantarflexion and inversion, and the forefoot is curved medially. The deformity is rigid with very little range of motion in the ankle. Treat by casting and, if there is no improvement after 3 months of casting, surgical correction.

membranous nephropathy

A form of glomerulonephritis seen in young children. Presents with heavy proteinuria and often progresses to renal insufficiency.

Alport's syndrome

A form of progressive hereditary nephritis secondary to defects in side chains of type IV collagen. X-linked dominant. Renal manifestations include HTN and hematuria; ESRD may occur. Hearing loss begins in childhood and progresses, and coular abnormalities involving the lens and retina also occur. Therapy includes treatment of HTN, use of ACE inhibitors, and eventually renal transplant.

Infants with *upper brachial plexus injury* (cervical nerves 3, 4, 5) can also have ipsilateral *phrenic nerve paralysis*. These infants can present with labored, irregular breathings and cyanosis; the injury is usually unilateral. Confirmation of the diagnosis is made with ultrasound or fluoroscopy, which confirms "seesaw" movements of the diaphragm during respiration.

A large-for-gestation-age term infant delivered via scheduled cesarean section develops, at 15 minutes of age, tachypnea, grunting, flaring, and retractions. The child does not move his left arm well, but you find no clavicular fracture. A chest radiograph shows the left diaphragm to be markedly higher than the right. What is the likely diagnosis?

*Bacterial tracheitis* is an uncommon but severe and life-threatening sequela of viral laryngotracheobronchitis. The typical story is that presented in the case, with several days of viral upper respiratory symptoms, followed by an acute elevation of temperature and an increase in respiratory distress. Inspiratory stridor is typical in croup; the biphasic stridor and high fever in this patient should be a clue to consider alternative diagnoses. Children may also present acutely and without the initial viral symptoms. The differential must include epiglottitis; the lack of drooling and dysphagia (and the rarity of epiglottitis since the introduction of the Haemophilus influenza B vaccine) help make this a case of tracheitis. Management for tracheitis includes *establishing an airway with endotracheal intubation and IV antibiotics.* Special attention is focused on preservation of the airway, as even intubated children with tracheitis can have secretions thick and copious enough to occlude the airway.

A fully immunized 2-year-old presents to the emergency room with several days of low-grade fever, barking cough, and noisy breathing. Over the past few hours he has developed a fever of 40°C (104°F) and looks toxic. He has inspiratory and expiratory stridor. The family has not noticed drooling, and he seems to be drinking without pain. Direct laryngoscopy reveals a normal epiglottis. What is the likely diagnosis?

absence epilepsy of childhood

A generlaized 3 Hz spike and wave discharge on EEG suggests what diagnosis?

Ehlers-Danlos syndrome

A genetic disorder characterized by production of defective type V collagen, resulting in hyperextensible joints, fragile vessels, and loose skin. Autosomal dominant. Complications include aortic dissection and GI bleeding as a result of blood vessel fragility.

*Mullerian agenesis:* failure of Mullerian duct development. Normally, this duct develops into the uterus, cervix, and upper third of the vagina. It does not affect ovarian development, and thus patients have normal estrogen levels and normal secondary sexual characteristics. They will have a normal external genitalia and a vagina which ends in a blind pouch.

A girl with primary amenorrhea, normal secondary sexual characteristics, and absent uterus likely has what abnormality?

Neisseria gonorrhoeae

A gram-negative intracellular diplococcus that infects the cervical columnar epithelium. Infection presents with mucopurulent endocervical discharge, sometimes with vaginal bleeding. There will also be dysuria, urinary frequency, and dyspareunia. Diagnose with culture on Thayer-Martin media (gold standard), Gram stain, urine PCR, NAAT. Manage with IM ceftriaxone, and also treat presumptive co-infection with C. trachomatis. Partners should also be treated.

*Right to left shunts:* Transposition of the great arteries Tetralogy of Fallot Truncus arteriosus Tricuspid atresia TAPVR

A group of congenital cardiac defects where deoxygenated blood is released into the systemic circulation rather than the lungs. Results in cyanosis (blue baby) and decreased vascular markings on CXR. Present with acute cyanosis or chronic effects like clubbing.

cerebral palsy

A group of static (i.e. nonprogressive) encephalopathies caused by injury to the developing brain in which motor function is primarily affected. Intelligence may be normal, but injuries to the brain often lead to other neurologic effects, like seizures, cognitive deficits, mental retardation, learning disabilities, sensory loss, and visual/auditory deficits.

Some common causes of eosinophilia in the peripheral blood smear include asthma, recurrent urticaria, infantile eczema, drug reactions, angioneurotic edema, helminth infections, collagen vascular disease, and some neoplasms. Note that while allergic rhinitis can cause eosinophilia in nasal secretions, but usually does not cause dramatic peripheral eosinophilia.

A healthy 1-year-old child comes to your office for a routine checkup and for immunizations. His parents have no complaints or concerns. The next day, the CBC you performed as customary screening for anemia returns with the percentage of eosinophils on the differential to be 30%. What are the most likely explanations?

C1 esterase deficiency

A hereditary angioedema caused by a defect in the C1 inhibitor. Doesn't contribute to increased susceptibility to infection. Since angioedema is not IgE mediated, there is no urticaria and response to antihistamines is poor. Treat with FFP.

hydrogen cyanide

A highly toxic colorless liquid that becomes a gas just above room temperature. Interestingly, it has a bitter almond-like odor that only some people can detect due to a particular genetic trait. It has been used as a rodenticide, and is produced as part of the manufacturing process for synthetic fibers and plastics. Fire victims are also commonly exposed. Poisoning presents with acute headache, shortly followed by agitation, confusion, and LOC, with dysrhythmia on the cardiac monitor and severe metabolic acidosis. Hydroxocobalamin is an approved antidote.

Common Variable Immunodeficiency (CVID)

A humoral immunodeficiency seen in both males and females, adults and children. There will be deficiencies in at least 2 out of 3 IG types (IgA, IgG, IgM). Treat with scheduled IVIG.

They have not been taking their insulin

A known diabetic presents with DKA. What is the most likely cause?

dermatitis herpetiformis

A pruritic papular or vesicular rash associated with celiac disease. Often located on the knees, elbows, forearms, and buttocks.

*Reassurance:* per CDC recommendations, varicella-zoster immunoglobulin (VZIG) should be administered to the infant immediately after delivery if the mother had the onset of varicella within 5 days prior to delivery, and immediately upon diagnosis if her chicken pox started within 2 days after delivery. If untreated, about half of these infants will develop serious varicella as early as 1 day of age. However, if a normal full-term newborn is exposed to chicken pox 2 or more days postnatally, VZIG and isolation are not necessary because these babies appear to be at no greater risk for complications than older children. Acyclovir may be used in infants at risk for severe varicella, such as those infants exposed perinatally.

A mother calls you frantic because she has just been diagnosed with varicella (chicken pox). She delivered a term infant 7 days ago that appears to be eating, stooling, and urinating without difficulty. The child has been afebrile and seems to be doing well. What is the most appropriate step in management?

Admit the child to the hospital immediately for surgical debridement and antibiotic treatment. Human bites can pose a significant problem. They can become infected with oropharyngeal bacteria, including S aureus, Streptococcus viridans, Eikenella corrodens, and anaerobes. A patient with an infected human bite of the hand requires hospitalization for appropriate drainage procedures, Gram stain and culture of the exudate, vigorous cleaning, debridement, and appropriate antibiotics.

A mother calls you on the telephone and says that her 4-year-old son bit the hand of her 2-year-old son 2 days ago. The area around the injury has become red, indurated, and tender, and he has a temperature of 39.4°C (103°F). What is the most appropriate response?

cystic fibrosis

A multisystem disorder that results in altered content of *exocrine gland secretions.* An *autosomal recessive* disease resulting from a mutation on *chromosome 7* that produces an abnormal ion-channel regulator protein (*CFTR*) that causes sodium and chloride transport dysfunction. This leads to *abnormal mucus* produced in airways, creating obstruction, inflammation, and infection.

breast milk jaundice

A neonatal jaundice caused by breast milk inhibiting glucuronyl transferase, which converts indirect bilirubin to direct bilirubin. Supplement with formula to prevent.

metachromatic leukodystrophy

A neurodegenerative disorder caused by arylsulfatase A deficiency; presents with ataxia, seizures, and mental retardation.

Rett syndrome

A neurodevelopmental disorder that occurs mainly in girls due to mutations in the X-linked MECP2 gene. Characterized by normal development for subsequent regression of speech, loss of purposeful hand movements (inability to feed or dress oneself), gait abnormalities, and stereotypical hand movements at age 6-18 months. Deceleration in head growth may be an early sign. Seizures occur in the majority of cases. Mechanism is unclear.

*Congenital rubella* infection affects all organ systems. Infants will be small, with intrauterine growth retardation. They may also manifest cataracts, microphthalmia, myocarditis, and a red or purple macular rash ("blueberry muffin" rash). Structural heart defects (such as a patent ductus arteriosis, pulmonary artery stenosis, and septal defects) are typical of congenital rubella, but not in the other TORCH infections. Laboratory anomalies may include a hemolytic anemia with thrombocytopenia, elevated liver functions, and pleocytosis in the spinal fluid. Affected children do not have a good prognosis. Congenital rubella is not commonly seen in developed countries with high immunization rates.

A newborn has bilateral cataracts and microphthalmia, intrauterine growth retardation, hemorrhagic skin lesions scattered throughout the body, and a harsh systolic murmur heard at the left sternal border and radiating to the lung fields. What is the likely diagnosis?

*Toxoplasmosis* can cause symptoms similar to other congenital infections, but the combination of hydrocephalus, chorioretinitis, and intracranial calcifications is considered the "classic triad" of toxoplasma infection in a neonate. Infection usually occurs during primary infection of the mother or as a reactivation of infection in an immune-compromised host. These infants may also display symptoms similar to other congenital infections, such as anemia, a petechial rash, organomegaly, jaundice, and seizures.

A newborn has hydrocephalus, chorioretinitis, intracranial calcifications, and anemia. What may have caused this?

acute airway obstruction

A rare but potentially serious complication of infectious mononucleosis. Symptoms include throat tightness and difficulty swallowing. Patients exhibit severe tonsillar enlargement, experience difficulty swallowing, and have labored breathing. Treatment includes corticosteroid administration.

The infant described has *spinal muscular atrophy (SMA) type I*, also referred to as Werdnig-Hoffman disease, or infantile progressive SMA. The defect is found in the survivor motor neuron (SMN) gene that stops apoptosis of motor neuroblasts. During development, an excess of motor neuroblasts is noted, and through apoptosis only about half survive in the normal newborn; the SMN gene regulates this natural destruction. A defect in the SMN gene results in a continuation of apoptosis, resulting in progressive destruction of motor neurons in the brain stem and spinal cord. The only currently available treatment is supportive care, and infants with SMA I usually die of respiratory complications by the second or third year of life.

A newborn infant has respiratory distress and trouble feeding in the nursery. The mother has no significant medical history, but the pregnancy was complicated by decreased fetal movement. On physical examination, you note that aside from shallow respirations and some twitching of the fingers and toes, the infant is not moving, and is very hypotonic. In the mouth, there is pooled saliva and you note tongue fasciculations. Deep tendon reflexes are absent. Spinal fluid is normal. What is the diagnosis?

congenital diaphragmatic hernia

A newborn presents with a *scaphoid abdomen* (with abdominal contents in the thorax) and severe respiratory insufficiency from pulmonary hypoplasia. There is severe hypoxemia and acidosis. Breath sounds are decreased and bowel sounds may be heard in the chest. What is the cause of this?

*Displaced clavicular fracture:* most often due to difficult delivery. Risk factors include birth weight >4 kg, shoulder dystocia, and vacuum delivery. X-rays should be performed to confirm a fracture. Because these fractures heal rapidly (7-10 days) with no long term sequelae, management includes parental reassurance and guidance on gentle handling.

A newborn presents with crepitus over the clavicle, asymmetric Moro reflex, and pain with passive motion of the upper extremity. What is the likely diagnosis?

*Diabetic embryopathy:* infants born to diabetic mothers are frequently macrosomic and may become hypoglycemic. However, they can have many other problems as well, including cardiac septal hypertrophy, congenital heart disease, caudal regression, vertebral defects, and a single umbilical artery.

A newborn with hypoglycemia, hypocalcemia, and hypoplastic lower extremities likely has what diagnosis?

Edwards syndrome (trisomy 18)

A newborn with microcephaly, micrognathia, overlapping fingers, absent palmar creases, and rocker bottom feet likely suffers from what?

necrotizing enterocolitis

A newborn with rectal bleeding, feeding intolerance, or abdominal distention should have you considering what diagnosis?

erythema marginatum

A nonpruritic rash seen in rheumatic fever patients. IT starts as pink to red macules which may coalesce and spread centripetally with central clearing over the trunk and proximal limbs.

Clostridium difficile

A normal component of the gut flora that may cause colitis if it overgrows the rest of the gut flora, as is seen after antibiotic use. Endoscopy may demonstrate pseudomembranes. Oral or IV metronidazole is effective treatment; oral vancomycin is reserved for resistant cases

microangiopathic hemolytic anemia

A normocytic anemia that results from mechanical damage to RBCs caused by passage through an injured vascular endothelium. Causes include severe HTN, HUS, artifical heart valves, a giant hemangioma, and DIC. Anemia and thrombocytopenia are characteristic; labs show burr cells and target cells on smear. Management is supportive and treatment of underlying conditions.

buccal cellulitis

A now uncommon form of cellulitis that presents as unilateral bluish discoloration on the cheek of a young unimmunized child. The causative agent is Hib. Treat with antibiotics like cefuroxime or cefotaxime.

*Hemophilia:* an X-linked recessive bleeding disorder caused by deficiency of factor VIII (hemophilia A) or FActor IX (hemophilia B). Often, patients will present with *recurrent hemarthroses* which are characterized as joint jain and swelling with little or no trauma, due to spontaneous bleeding into a joint.

A patient has a history of bleeding after a dental procedure. What diagnosis should you consider?

*Turner syndrome:* these patients are at increased risk of *osteoporotic fracture* due to estrogen deficiency from ovarian dysgenesis.

A patient has absent menarche at age 14 and different blood pressures in the upper and lower extremities. What diagnosis do you suspect and what is one complication of that diagnosis?

Get an *FSH level*, which will help distinguish between central and peripheral causes of amenorrhea in the HPO axis. Central causes have low to normal FSH, while peripheral causes have high FSH.

A patient has primary amenorrhea (age >13 with no menarche or secondary sexual characteristics) and apelvic ultrasound determines that a uterus is present. What is the next step in diagnosis?

uncal herniation

A patient is comatose with a unilateral dilated nonreactive pupil. What should you suspect?

bacterial endocarditis the lesions described are Osler's nodes, Janeway lesions, and Roth's spots

A patient presents with fever and nonspecific complaints (malaise, arthralgia, headache, weight loss, night sweats). They have a new murmur, splenomegaly, hematuria, splinter hemorrhages, retinal hemorrhages, and round white spots in their retina. They also have small raised pink lesions on their palms and small erythematous hemorrhagic lesions on their palms and soles. What is the diagnosis?

*Infectious mononucleosis:* commonly caused by EBV. The postantibiotic rash after administration or ampicillin or amoxicillin is not well understood, but not considered a true drug allergy as patients can get the same antibiotic in the future without reaction.

A patient presents with fever, malaise, exudative pharyngitis, cervical lymphadenopathy, and hepatosplenomegaly. They also developed a polymorphous rash after taking old amoxicillin. What is the likely diagnosis?

cystic fibrosis

A patient presents with reccurent sinopulmonary infections, nasal polyps, and digital clubbing. There is also vitamin D deficiency, bruising (indicative of vitamin K deficiency), and poor growth. What is the likley diagnosis?

*Bronchiolitis* is a very common viral infection most often caused by respiratory syncytial virus. It is most often seen in the winter months with symptoms of wheezing, hypoxia, and respiratory distress seen in younger children; often an older sibling has milder, upper respiratory symptoms. Premature infants, infants with congenital heart disease, infants with a variety of lung disorders, and infants with immune system defects are at higher risk of severe complications. Diagnosis is made by clinical history and/or detection of the viral antigen in nasal secretions; treatment is supportive.

A preterm infant is now 7 weeks old. She was intubated for 2 weeks and was weaned off oxygen at 3 weeks of age. You are about to leave your office for the Thanksgiving holiday when the emergency room calls to tell you she has recent onset of hypoxia, respiratory distress, wheezes, and runny nose. A chest radiograph reveals patchy infiltrates and hyperexpansion in both lung fields. The newborn's 2-year-old sibling has an upper respiratory infection.

*Encourage adequate fluid and salt intake.* This is likely vasovagal syncope.

A previously healthy 16-year-old girl presents to the emergency center with the complaint of "falling out." She was with her friends at a local fast food restaurant when she felt faint and, according to her friends, lost consciousness for about a minute. There was no seizure activity noted, but the friends did notice her arms twitching irregularly. She is now acting normally. She denies chest pain or palpitations, and her electrocardiogram (ECG) is normal. What should you do?

This child has evidence of a stroke. Emergency imaging of the brain (preferably MRI but CT with contrast, if necessary), urgent transfusion of blood to reduce the number of circulating sickled cells, and hospitalization (likely in the intensive care unit) to observe for further neurologic deterioration are indicated. As recovery begins, physical therapy is instituted and the patient is enrolled in a chronic blood transfusion program to reduce the risk of recurrence.

A previously healthy 2-year-old child is known to have sickle cell disease; she now has a 1-hour history of left-sided weakness and ataxia. What is the appropriate management for this child?

botulism

A previously healthy 3-week-old boy was seen in your office 5 days ago. This is the family's third child, was the product of an uneventful pregnancy, and they have no concerns other than feeling that he does not now stool so often as their other breast-fed children. Your examination is normal. Over the next several days, he has worsening of his "constipation" (despite the family's trying a number of home remedies recommended by the grandmother), he is increasingly lethargic, he has developed a weak cry, and he seems to have decreased spontaneous movement. Overall, the family reports that his eating has decreased and he seems to be gagging when he eats. You admit him to the hospital for a "rule out sepsis" evaluation. You find on your examination a poor gag and suck reflex with drooling from his mouth, diminished corneal reflexes, ptosis, and loss of head control. His lumbar puncture attempt must be aborted after the collection of the first tube of fluid is gathered due to his developing respiratory distress while curled for the procedure. What diagnosis do you suspect?

The abrupt onset of a hemisyndrome, especially with the eyes looking away from the paralyzed side, strongly indicates a diagnosis of *acute infantile hemiplegia.* Most frequently, this represents a thromboembolic occlusion of the middle cerebral artery or one of its major branches. The diagnosis has also been used to describe an acute syndrome of fever and partial seizure with resulting hemiparesis. Childhood stroke can result from trauma, infection, a hyper-coagulable state, arteritis, and congenital structural or metabolic disorders.

A previously healthy 7-year-old child suddenly complains of a headache and falls to the floor. When examined in the emergency room (ER), he is lethargic and has a left central facial weakness and left hemiparesis with conjugate ocular deviation to the right. What is the likely diagnosis?

infective endocarditis

A previously healthy 8-year-old boy has a 3-week history of low-grade fever of unknown source, fatigue, weight loss, myalgia, and headaches. On repeated examinations during this time, he is found to have developed a heart murmur, petechiae, and mild splenomegaly. What is the likely diagnosis?

Small children frequently introduce any number of small objects into their noses, ranging from food to small toys. Initially, only local irritation occurs. Later, as prolonged obstruction is seen, symptoms increase to include worsening of pain, and a purulent, malodorous, bloody discharge can be seen. Unilateral nasal discharge in the presence of obstruction suggests the need to examine the patient for a *nasal foreign body.*

A previously healthy, active, 18-month-old African American child presents with unilateral nasal obstruction and foul-smelling discharge. The child's examination is otherwise unremarkable. What is the most likely diagnosis?

*Bronchiolitis:* the most likely cause of the illness is infection by respiratory syncytial virus, which causes outbreaks of bronchiolitis of varying severity, usually in the winter and spring. Other viruses, such as parainfluenza and the adeno-viruses, have also been implicated in producing bronchiolitis. Treatment is generally supportive in this usually self-limited condition.

A previously well 1-year-old infant has had a runny nose and has been sneezing and coughing for 2 days. Two other members of the family had similar symptoms. Four hours ago, his cough became much worse. On physical examination in the ED, he is in moderate respiratory distress with tachypnea and nasal flaring. Upon auscultation, he has easily audible wheezing with scattered crackles bilaterally. His arterial blood gas on room air revealed a pH of 7.46, a PaCO2 of 34 mm Hg, and a PaO2 of 75 mm Hg. His chest radiographs show patchy infiltrates with flat diaphragms. What is the likely diagnosis and treatment?

eczema herpeticum

A primary herpes simplex virus infection associated with atopic dermatitis. Painful vesicles, "punched-out" erosions, and hemorrhagic crusting along with fever and lymphadenpathy are typical.

Blount's disease (tibia vara)

A progressive angulation at the proximal tibia. Thought to a result of overload injury to the medial tibial growth plate causing inhibited growth only on the medial side. Presents with lateral thrust with gait. Should be suspected in any child with progressive bowing, unilateral bowing, or persistent bowing of the legs after 2 years of age. Treat with bracing or surgical osteotomy.

biliary atresia

A progressive fibrosclerotic disease that affects the extrahepatic biliary tree. Etiology unknown. Patients usually present between 4-6 weeks of age with jaundice, dark urine, and pale or acholic stools. Hepatosplenomegaly, ascites, poor growth, steatorrhea, peripheral edema, and coagulopathy can also occur. Bilirubin levels are moderately elevated. Progression of rapid, with bile duct obliteration and cirrhosis by 4 months.

allergic conjunctivitis

An IgE mediated acute hypersensitivity to environmental allergens. Episodes may be due to a novel exposure (eg a new cat) but more commonly are seasonal (pollens) or perennial (mold, dust). Ocular pruritus is typical, and edema of the conjunctivae and eyelids and clear discharge are also seen. Patients may have very mild crusting of the eyes. Patients commonly have other atopic disease like asthma or allergic rhinitis.Treat with topical antihistamines and allergen avoidance.

Allergic Conjunctivitis

An IgE mediated inflammation of the eyes. Look for eye discharge, conjunctival injection and swelling, and "allergic shiners." Treatment involves allergen avoidance, artificial tears, and combination eye drops (mast cell stabilizers plus antihistamines).

*Shigella:* clinical manifestations of shigellosis range from watery stools for several days to severe infection with high fever, abdominal pain, and generalized seizures. In addition, there is a rare and fatal "toxic encephalopathy" seen with Shigella infection known as Ekiri syndrome. In general, about 50% of infected children have emesis, greater than two-thirds have fever, 10% to 35% have seizures, and 40% have blood in their stool. Often, the seizure precedes diarrhea and is the complaint that brings the family to the physician. Fever usually lasts about 72 hours, and the diarrhea resolves within 1 to 2 weeks. Presumptive diagnosis can be made on the clinical history; confirmation is through stool culture. Supportive care, including adequate fluid and electrolyte support, is the mainstay of therapy. Antibiotic treatment is problematic; resistance to trimethoprimsulfamethoxazole and ampicillin is common, necessitating therapy with third-generation cephalosporins in many cases. As always, knowledge of the susceptibility patterns of the bacteria in your area is the key to using the right antibiotic.

An 18-month-old child presents to the emergency center having had a brief, generalized tonic-clonic seizure. He is now postictal and has a temperature of 40°C (104°F). During the lumbar puncture (which ultimately proves to be normal), he has a large, watery stool that has both blood and mucus in it. What is the likely diagnosis?

*Wilson disease* is an autosomal recessive disorder characterized by liver disease (usually seen in childhood), neurologic and behavioral disturbances (seen by adolescence), renal tubular dysfunction (Fanconi syndrome), and eye findings (Kayser-Fleischer rings). Its multisystem manifestations are caused by the deposition of copper in various tissues (resulting in low serum levels), and therapy is aimed at the prevention of accumulation of copper. Defective metabolism of the copper-binding protein ceruloplasmin (usually reduced) has been demonstrated by some.

An 18-year-old girl has hepatosplenomegaly, an intention tremor, dysarthria, dystonia, and deterioration in her school performance. She also developed abnormal urine with excess glucose, protein, and uric acid. She has a several-year history of elevated liver enzymes of unknown etiology. What is the diagnosis?

rocky mountain spotted fever oral or IV doxycycline

An 8-year-old Cub Scout who returned from an outing 9 days ago is brought to the clinic with the rapid onset of fever, headache, muscle pain, and rash. The maculopapular rash began on the flexor surfaces of the wrist and has become petechial as it spread inward to his trunk. What is the diagnosis and treatment?

*pancreatitis* secondary to blunt trauma to the abdomen

An 8-year-old boy is accidentally hit in the abdomen by a baseball bat. After several minutes of discomfort, he seems to be fine. Over the ensuing 24 hours, however, he develops a fever, abdominal pain radiating to the back, and persistent vomiting. On examination, the child appears quite uncomfortable. The abdomen is tender, with decreased bowel sounds throughout, but especially painful in the midepigastric region with guarding. What is the likely diagnosis?

*Idiopathic hypercalciuria* causes recurrent gross hematuria, persistent microscopic hematuria, and complaints of dysuria or abdominal pain without initial stone formation. Over time, however, stones may form in 15% of cases.

An 8-year-old boy with the intermittent complaint of "burning" when he urinates and who has trace blood on his urine dip test. What is the likely diagnosis?

dystonia

An 8-year-old hospitalized boy with unusual "spasms" of his neck and arms shortly after receiving Phenergan for nausea caused by his chemotherapy. What is the diagnosis?

One to seven days following the ingestion of pork or other improperly cooked meat infected with *T spiralis*, symptoms develop, including abdominal pain, nausea, vomiting, and malaise. During the second week, muscle invasion occurs, which causes edema of eyelids, myalgia, weakness, fever, and eosinophilia. The muscle organisms can become encysted and remain viable for years. Therapy is with *mebendazole or albendazole.*

An 8-year-old immigrant from rural Central America presents with complaints of weakness, facial swelling, muscle pain, and fever. A CBC shows his white blood count to be 8,500/mm3 with 30% eosinophils. What is the diagnosis and treatment?

*Parvovirus B19:* fifth disease (erythema infectiosum), long recognized as a benign mild exanthem of school-age children, is now known to be caused by human parvovirus B19. Replication of the virus occurs in the erythroid progenitor cells resulting in a decrease in red cell production for about a week in infected patients. While this transient drop in reticulocytes is not noticeable in normal children, patients with hemolytic conditions (such as sicklecell anemia) develop a transient aplastic crisis. A poorly functioning bone marrow (for a week or more) in a patient with a reduced red-cell life span (about 30 days) can result in profound anemia. Other problems can result in patients infected with parvovirus B19. In patients with immunodeficiency, the B19 infection can be persistent and lead to life-threatening chronic anemia. Infection in a pregnant woman can result in severe anemia in the infected fetus, with secondary hydrops fetalis and death.

An 8-year-old sickle-cell patient arrives at the emergency room (ER) in respiratory distress. Over the previous several days, the child has become progressively tired and pale. The child's hemoglobin concentration in the ER is 3.1 mg/dL. What is the likely infectious agent?

tricuspid atresia

An ASD or PFO must always be present for an infant to survive with this congenital cardiac defect, wherein there is a plate of tissue in the location of the tricuspid valve.

viral nasal cold

An ENT infection caused by rhinovirus and transmitted between people by large droplets. Presents with rhinorrhea, congestion, and low grade fever. If it lasts less than 10 days, it is likely viral (if over 10 days, likely sinusitis, a bacterial infection). No treatment necessary, it will resolve spontaneously.

Henoch Schonlein purpura

An IgA mediated vasculitis characterized by nonthrombocytopenic palpable purpura on the buttocks and thighs, abdominal pain, arthritis or arthralgias, and gross or microscopic hematuria. Usually self-limited with complete recovery within 3 months.

Henoch Schonlein purpura

An IgA mediated vasculitis that presents with a palpable purpuric rash on the buttocks and lower extremities, large joint arthralgias, renal involvement, and GI bleeding from complications such as intussusception and bowel perforation.

Henoch Schonlein purpura

An IgA mediated vasculitis, which presents with palpable purpura on the lower extremities and buttocks, renal insufficiency, arthritis, and abdominal pain. Platelet count is normal.

infective endocarditis

Antibiotic prophylaxis for this disease should be provided to all patients with structural heart diseases undergoing dental work likely to produce bleeding, surgery, or invasive GI procedures. It should also be given to postop cardiac surgery patients for up to 6 months.

physiologic jaundice

Benign and self-limited indirect hyperbilirubinemia that typically resolves by the end of the first week of life and requires no treatment. Due to increased bilirubin load on hepatocytes and delayed activity of the hepatic enzyme glucuronyl transferase. Peaks on days 3-4 of life.

genu varum

Bowing of the legs in children under 2; normal until over age 2. OFten presents with a "cowboy stance" and normal gait.

1% silver nitrate, which is an eyedrop used as prophylaxis against N gonorrhoeae.

Chemical conjunctivitis is most often secondary to what?

HCC

Chronic HBV infection may result in increased risk for what?

recurrent cystitis (UTIs)

Chronic constipation in toddlers can lead to *urinary stasis* due to compression of the bladder from rectal distention. This puts the infant at risk for what?

eosinophilic esophagitis

Chronic, immune-mediated esophageal inflammation which presents with dysphagia, chest pain, reflux, vomiting, food impaction, and associated atopic dermatitis. Diagnosed via endoscopy and esophageal biopsy, which will reveal eosinophils. Treat with dietary modification and topical glucocorticoids

Bacterial: Increased WBCs, PMNs predominate; high protein, low glucose. Viral: PMNs early, then monocytes and lymphocytes; RBCs present in HSV encephalitis. Protein normal to high, glucose normal. Tb: Lymphocytes predominate, protein very high, glucose low. Fungal: Lymphocytes predominate, protein normal to high, glucose low.

Compare CSF analysis in bacterial, viral, Tb, and fungal meningitis.

*Niemann-Pick:* sphingomyelinase deficiency which results in loss of motor milestones, hypotonia, feeding difficulties, cherry red spot, hepatosplenomegaly, and areflexia. Autosomal recessive, seen in Ashkenazi Jews, onset 2-6 months of age. *Tay-Sachs:* beta-hexosaminidase A deficiency which results in loss of motor milestones, hypotonia, feeding difficulties, cheery red spot, and hyperreflexia. Autosomal recessive, seen in Ashkenazi Jews, onset 2-6 months of age.

Compare and contrast Niemann-Pick and Tay-Sachs disease

*Hodgkin's:* slow indolent course, cervical and supraclavicular involvement, systemic symptoms common, abdominal findings rare, SVC syndrome and airway compression rare, painless adenopathy common. *NHL:* rapid course, abdominal, mediastinal ,and supraclavicular involvement, systemic symptoms uncommon, but painless adenopathy, SVC syndrome, and airways compression common.

Compare and ocntrast Hodgkin's disease vs non-Hodgkin's lymphoma

persistent urachus

Complete failure of the urachal duct to close, resulting in a fistula between the bladder and the umbilicus. May present with urine draining from the umbilicus, especially when pressure is applied over the bladder.

cardiac tamponade

Critically impaired left ventricular output as a result of pericarditis, where inflammation of parietal and visceral pericardial layers leads to exudation or transudation of fluid and impairment of venous return and cardiac filling.

colic

Crying that occurs >3 hours/day and >3 days/week. (Note that normal crying usually lasts up to 2 hours/day at 2 weeks of age and increases to 3 hours/day at 3 months.) Occurs in about 10% of newborns. Begins at 2-4 weeks of age and resolves by 3-4 months.

*Brain abscess,* due to hematogenous spread of bacteria. Brain abscess in children frequently presents with headache, fever, focal neurologic deficits, and seizure.

Cyanotic congenital heart diseases (such as Tetralogy of Fallot) are a risk factor for what CNS pathology?

*Congenital hypothyroidism:* associated with neurodevelopmental injury if not recognized and treated early. *Thyroid dysgenesis* is the most common cause.

Decreased activity, hoarse cry, large tongue, and jaundice are commonly associated with what congenital condition in newborns?

zinc

Deficiency of this mineral results in skin lesions, poor wound healing, immune dysfunction, diarrhea, and growth failure

Vitamin E

Deficiency of this vitamin results in anemia (hemolysis), neurologic deficits, altered prostaglandin synthesis

vitamin B1 (thiamine) *Beriberi*

Deficiency of this vitamin results in cardiac failure, peripheral neuropathy, hoarseness or aphonia, and Wernicke's encephalopathy

vitamin K

Deficiency of this vitamin results in coagulopathy with increased prothrombin time, abnormal bone matrix synthesis

vitamin B6 (pyridoxine)

Deficiency of this vitamin results in dermatitis, cheilosis, glossitis, microcytic anemia, and peripheral neuritis

niacin *pellagra*

Deficiency of this vitamin results in diarrhea, dermatitis, dementia, glossitis, and stomatitis

vitamin C *scurvy*

Deficiency of this vitamin results in hematologic abnormalities, edema, spongy swelling of the gums, poor wound healing, and impaired collagen synthesis

vitamin A

Deficiency of this vitamin results in night blindness and xerophthlamia (dry conjunctiva and cornea)

vitamin D

Deficiency of this vitamin results in rickets, dental caries, hypcalcemia, and hypophosphatemia

Daytime Sx 2+ times/week Nighttime Sx 2+ times/month Give albuterol PRN

Define intermittent asthma and its management

Daytime Sx btwn 2X/week and 1X/day Nighttime Sx >2 times/month FEV1 > 80% predicted (normal) Albuterol for symptom relief Low-dose inhaled steroids daily

Define mild persistent asthma and its treatment

ataxia

The inability to coordinate muscle activity during voluntary movement. It can involve the trunk or limbs and is caused by cerebellar or proprioceptive dysfunction.

premature adrenarche

Early onset of pubic or axillary hair which occurs without development of breast tissue or enlarged testes; more common in females. Classically occurs after age 5, with onset of pubic hair growth, axillary hair growth, and apocrine odor. No breast tissue is noted, and there is no clitoromegaly. Growth is normal without advancement of bone age. No treatment is indicated.

Alport syndrome

Electron microscopy of a renal biopsy specimen showing alternating areas of thinned and thickened capillary loops with splitting of the glomerular basement membrane would suggest what diagnosis?

nephrotic syndrome

Elevated levels of cholesterol and triglycerides found in a 6-year-old boy whose mother reports that he has been awakening with puffy eyes each morning. On your physical examination, you determine that he has had unexpected weight gain and has scrotal edema. What is the likely diagnosis?

acute pancreatitis

Elevated serum amylase and lipase are specific for what disease?

type 1 DM

Enuresis in the setting of polyuria, polydipsia, and weight loss is suggestive of what disease?

*extrophy of the bladder:* keep covered with plastic barrier to prevent drying out and correct surgically.

Extruded viscera in the midline of an infant that is red, shining, and wet with urine is most likely what?

Tics are commonly seen in a pediatric practice. All have in common the nonrhythmic, spasmodic, involuntary, stereotypical behaviors that involve any muscle group. *Transient tic disorder* is the most common and is seen more often in boys; a family history is often noted. In this condition, the patient has eye blinking, facial movements, or throat clearing lasting for weeks to about a year. No medications are needed. Chronic motor tics persist throughout life and can incorporate motor movements involving up to three muscle groups.

Eye blinking or throat-clearing noises in an otherwise healthy 8-year-old boy would most likely be attributable to what?

blepharitis

Eyelid inflammation; one of the most common causes of red eye. Usually due to Staph aureus infection. Features include burning, crusting, and scales at the eyelash base, thickened and hyperemic eyelid margins, broken or absent eyelashes, and a history of awakening in the morning with eyelashes stuck together. Treat with eyelid hygiene, in which eyelids are scrubbed twice daily with baby shampoo. Topical erythromycin ointment is also applied.

desmopressin

First-line pharmacotherapy for nocturnal enuresis after behavioral modifications and alarm therapy have failed. Unfortunately, it has a high rate of relapse on discontinuation of therapy.

*Hereditary angioedema,* transmitted as an autosomal dominant trait, is a result of inadequate function (owing to either deficient quantity or quality) of C1 esterase inhibitor (C1-INH), involved in the first step in the complement cascade, which results in the excessive production of a vasoactive kinin. In addition to otherwise asymptomatic subcutaneous edema, edema can occur in the gastrointestinal tract and produce the symptoms mentioned in the question. Laryngeal edema with airway obstruction can also occur; a change in the tone of the voice and difficulty swallowing secretions are symptoms of impending airway compromise and require emergent medical care.

For the past year, a 12-year-old boy has had recurrent episodes of swelling of his hands and feet, which has been getting worse recently. These episodes occur following exercise and emotional stress, last for 2 to 3 days, and resolve spontaneously. The last episode was accompanied by abdominal pain, vomiting, and diarrhea. The results of routine laboratory workup are normal. An older sister and a maternal uncle have had similar episodes, but they were not given a diagnosis. He presents today with another episode. What is the likely diagnosis?

For drugs with an enterohepatic circulation (eg, phenobarbital and tricyclic antidepressants), or those with prolonged absorption (eg, sustained-release theophylline), the use of multiple-dose activated charcoal can be effective in decreasing the half-life and increasing the total body clearance of the toxic substance

For what toxins can activated charcoal be used to decrease absorption?

cough hoarsenss inspiratory stridor

Foreign bodies lodged in the larynx or trachea (extrathoracic) typically present with what symptoms?

MPGN

Glomerulonephritis characterized by lobular mesangial hypercellularity and thickening of the glomerular basement membrane. aAtients typically present with nephritis or nephrotic syndrome with hematuria; HTN is common, as is low serum complement. Patients may respond to corticosteroids, and ACE inhibitors may slow disease progression.

folate and iron

Goat's milk, by itself, is not an ideal source of infant nutrition as it contains inadequate amounts of what nutrients?

congenital dermal melanocytosis (Mongolian spots)

Gray-blue macules most commonly seen on the sacrum and buttocks during the first days of life. They usually fade spontaneously during childhood.

cystic fibrosis

Growth failure and recurrent respiratory infections, along with steatorrhea, should raise concern for what disease in infants?

Culture Bordetella pertussis from nasopharyngeal secretions plated on Regan-Lower or Bordet-Gengou meida, or by positive direct fluorescent antibody tests of nasopharyngeal secretions.

How do you definitely diagnosis pertussis?

*Ultrasound* is the method of choice in detecting intussusception; the positive finding of the *target sign* should prompt immediate enema reduction. *Air enemas* are the preferred treatment.

How do you detect and treat suspected intussusception?

concussion

Head trauma with a loss of consciousness without any bleeding seen on CT scan. Can be mild (observe only) or severe (admit). Often occurs during athletics, and athletes must stop playing in their current game/match and have a slow stepwise return to play (sleep, go to school, do homework, practice, play in that order).

hemophilia

Hemarthrosis after minor trauma should prompt suspicion of what disease?

*Bone marrow biopsy* demonstrating marrow replacement by lymphoblasts. Other normal marrow elements are decreased or absent.

How do you confirm a diagnosis of ALL?

developmental dysplasia of the hip

Hip pathology where the hip is insufficiently deep, so the femur head constantly pops out. Diagnosed with a clear clunk during Barlow and Ortolani maneuvers. Confirm diagnosis with an ultrasound at 4-6 weeks as there can be physiologic laxity initially around time of birth which may resolve. Once diagnosed, put child in a harness to keep femur approximated to the joint as the joint grows out.

Causes include drugs (*sulfonamides, anticonvulsants, chloramphenicol*), infection (*HIV, EBV, CMV*), chemicals, and radiation.

How can you get an acquired aplastic anemia (pancytopenia)?

PVR decreases SVR increases

How do PVR and SVR change after birth?

By default, the gonads become ovaries; because there is no testicular tissue, there is no testosterone or anti-mullerian hormone, and so the Mullerian ducts develop and the Wolffian ducts regress, leading to female internal genitalia. Since there is also no DHT, the external genitalia develops into the labia, clitoris, and lower two thirds of the vagina.

How do the gonads develop in the absence of the SRY gene?

Developmental age divdided by chronologic age times 100. Over 85 is normal, under 70 is abnormal, and anything in between warrants close follow-up.

How do you calculate a developmental quotient?

ANC = WBC X 10 X (%neutr. + %bands)

How do you calculate the absolute neutrophil count, the percentage of WBCs that are neutrophils, bands, and immature myeloid cells?

*Chemical conjunctivitis* is a self-limited condition that presents within *6 to 12 hours* of birth and lasts for the first day or so of life; it is a consequence of ocular silver nitrate (no longer available in the United States). As most nurseries use erythromycin prophylaxis now, chemical conjunctivitis is less common. Both of these occular medications prevent gonococcal (GC) conjunctivitis. *GC conjunctivitis* has its onset within *2 to 5 days* after birth and is the most serious of the bacterial infections. Prompt and aggressive topical treatment and systemic antibiotics are indicated to prevent serious complications such as corneal ulceration, perforation, and resulting blindness. Parents should be treated to avoid the risk to the child of reinfection. *Chlamydial conjunctivitis* occurs *5 to 14 days* after birth; to avoid the risk of chlamydial pneumonia, treatment in an infant with conjunctivitis is with systemic antibiotics (parents, too, require treatment). However, asymptomatic infants born to chlamydia-positive mothers are not routinely treated with oral antibiotics at birth as prophylaxis, but rather watched closely for signs of infection, due to an increased incidence of hypertrophic pyloric stenosis among neonates having received erythromycin.

How do you differentiate between chemical, gonococcal, and chlamydial conjunctivitis in the newborn?

breast feeding = first week breast milk = after first week

How do you differentiate breast feeding jaundice from breast milk jaundice based on time course?

*Preseptal cellulitis* features eyelid erythema and swelling with chemosis; oral antibiotics are used to treat it. *Orbital cellulitis* features the symptoms of preseptal cellulitis plus pain with EOM, proptosis, and ophthalmoplegia with diplopia; IV antibiotics are used to treat.

How do you differentiate preseptal and orbital cellulitis?

*Sinus tachy:* heart rate <230, heart rate variation present, P waves normal, associated with fever, infection, and anemia, response to adenosine intervention gradual. *SVT:* heart rate >250, heart rate variation absent, P waves absent or abnormal axis, no predisposing factors, rapid response to adenosine

How do you differentiate sinus tachycardia from SVT?

1) Symptoms > 10 days 2) Symptoms severe (fever + drainage) >3 days 3) Symptoms worsening following initial improvement (ie biphasic illness)

How do you distinguish bacterial rhinosinusitis from viral infection?

Evaluate with an abdominal ultrasound, which may show absent or abnormal gallbladder. Early treatment with a Kasai procedure dramatically improves outcomes.

How do you evaluate for and treat biliary atresia?

*Low reticulocyte count* reflects bone marrow suppression or failure and is consistent with red cell aplasias, pancytopenia, and malignancy. *High reticulocyte count* reflects high bone marrow production of RBCs as seen in hemolytic anemias and sickle cell anemia.

How do you interpret and abnormal reticulocyte count?

Monthly *IVIG* replacement, aggressive management of infections with antibiotics, and chronic diarrhe amanagement including nutritional support.

How do you manage CVID?

The concentration of hemoglobin A2 is increased in *β-thalassemia trait* (also called β-thalassemia minor). Patients have a single abnormal gene for the β-globin component of hemoglobin; they do not typically have problems aside from a mild microcytic anemia, and *family counseling* is the only necessary intervention. It is important to distinguish between β-thalassemia trait and the more common iron-deficiency anemia, as iron is not useful to patients with β-thalassemia trait.

How do you manage a pediatric patient with increased hemoglobin A2?

high dose OCP, which will stabilize the endometrium and stop the acute bleeding

How do you manage heavy vaginal bleeding in hemodynamically stable teenage patients in whom the immature hypothalamic pituitary ovarian axis can cause anovulation with heavy, irregular menstrual bleeding?

Treat with transfusions; splenectomy is curative as the spleen lyses spherocytes. To decrease incidence of invasive disease by encapsulated organisms, splenectomy is generally delayed until after age 5.

How do you manage hereditary spherocytosis?

Incarceration is common; elective surgical repair is often considered

How do you manage indirect inguinal hernias, which often appear in premature infants?

*Common cold:* viral, short duration (less than 10 days), low grade fever, mild symptoms *Sinusitis:* bacterial, long duration (over 10 days), high fever, worsening symptoms

How do you tell the difference between common cold and sinusitis?

Transfusion will only provide transient benefit; *Corticosteroids* are often used until hemolysis diminishes. The acute form responds well to steroids.

How do you treat AIHA?

Resect the affected area and connect (pull-through procedure). Severe cases (perforation, full colon involvement) requires colostomy.

How do you treat Hirschprung's disease?

Although the disease is self-limited, treatment with *IVIG* should be started within 10 days of fever onset to help prevent coronary artery aneurysms. *Aspirin* is also indicated for its anti-inflammatory and antiplatelet effects.

How do you treat Kawasaki disease?

doxycycline (>9 years) amoxicillin (<9 years) Note that this is treatment for early localized disease or late disease with arthritis only. Carditis and meningitis often require IV ceftriaxone or penicillin.

How do you treat Lyme disease?

NSADIs are only useful in treating minor inflammatory symptoms like arthralgias. *Glucocorticoids* are the mainstay of therapy for children. In severe SLE, *cyclophosphamide* is useful, but it is cytotoxic. Other agents include azathioprine, methotrexate, and cyclosporine.

How do you treat SLE?

The preferred treatment is closed reduction by applying pressure on the radial head and *hyperpronating the forearm*. Also common is forearm supination with elbow flexion, though this may be less successful and more painful.

How do you treat a radial head subluxation (nursemaid's elbow)?

folate deficiency

Hypersegmented neutrophils are an indication of what?

serial PFTs (eg spirometry) of FVC and negative inspiratory force should be obtained to monitor for impending respiratory failure

If you suspect Guillain-Barre syndrome in a patient with ascending weakness and areflexia, what is an important part of the workup?

immediate bronchoscopy

If you suspect foreign body aspiration in a child, how should you proceed?

allergic rhinitis

IgE related inflammation of the *nasal mucosa*. Precipitating factors can either be seasonal (grasses, weeds, outdoor mold) or perennial (pets, dust mites, indoor mold). Presenting symptoms include rhinorrhea, sneezing, and nasal itching. Diagnosis is clinical and diagnostic testing usually isn't needed as an environmental history can uncover causes. Treatment includes *allergen avoidance*, intranasal meds (steroids, antihistamines), oral meds (antihistamines, leukotriene antagonists), and immunotherapy in severe or refractory cases.

Actions such as *crying and tachycardia* which increase resistance through the RVOT, or actions that decrease SVR like *exercise, vasodilation, or volume depletion,* increase right to left shunt and thus *increase cyanosis.* In contast, actions that increase SVR or reduce resistance through the RVOT (volume infusion, systmeic HTN, Valsalva, bradycardia) reduce right to left shunt and thus reduce cyanosis.

In Tet of Fallot, what action increase and decrease cyanosis?

*Orbital cellulitis:* the most common risk factor is *sinusitis*, particularly of the ethmoid or maxillary sinuses. The proximity of the sinuses to the orbital space allows the contiguous spread of bacteria (Step species, Staph aureus) during episodes of severe sinusitis.

In a child with painful eye movements, proptosis, ophthalmoplegia, and visual changes, what should you suspect?

necrotizing enterocolitis

In a newborn with temperature instability, feeding intolerance, abdominal distention, and bloody stools, who is premature, hypotensive, and has congenital heart disease, and has an abdominal x-ray with air within the bowel wall, what is the diagnosis?

*Above 100* is desirable; between 60-100, ventilation should be appropriate, and below 60 means chest compressions are needed (these are done in 3:1 ratio of compressions to breaths). If chest compressions are done, about 45-60 seconds should be completed before the underlying pulse is checked. IF there's no improvement, epi is administered via the umbilical vein catheter and the process should be continued.

In a newly born child, what heart rate is appropriate?

PEP consists of wound cleansing (if there is a wound), rabies immuno globulin (RIG), and the five injection rabies vaccine series. Modifications may be made for patients who have received PEP in the past.

In a patient with a bate bite with possible exposure to rabies, what is the correct postexpsoure orphylaxis?

A *renal and bladder ultrasound* should be ordered to evaluate for abnormalities that lead to recurrent UTIs.

In addition to 1-2 weeks of antibiotics, what is an important step in treating children under 2 years old with a first febrile UTI?

atopic dermatitis

In infancy, this condition presents as pruritus and scaly erythematous lesions on the face, chest, and extensor surfaces of the extremities. In older children and adults, flexural involvement is common. The first line treatment is topical emollients.

Staph aureus

In infant and young children patients with cystic fibrosis, what is the most common pathogen that causes pneumonia?

Trichuris trichuria (whipworm)

Infection with this helminth presents with *rectal prolapse*, abdominal pain, tenesmus, bloody diarrhea. Treat wtih albendazole and screen all close contacts.

Ascaris lumbricoides (roundworms)

Infection with this large intestinal helminth occurs via fecal oral transmission. *Loffler syndrome* can occur, which is a transient pneumonitis as larvae migrate through lungs causing fever, cough, wheezing, and eosinophilia. *Small bowel obstruction* is also common. Treat with albendazole and screen all close contacts.

Trichomonas vaginalis

Infection with this organism will present with malodorous, profuse, yellow-green vaginal discharge and a cervix which is friable and covered in petechiae. Vulvar inflammation and itching and dyspareunia are also common. Diagnosis is made via wet-mount saline microscopy demonstrating the organism, or by positive culture and vaginal pH > 4.5. Treat with oral metronidazole. Partner should also be treated.

croup (laryngotracheobronchitis)

Inflammation and edema of the subglottic larynx, trachea, and bronchi. The most common form is viral, occurring in children 3 months to 3 years in the late fall and winter; a spasmodic form occurs year round in preschool aged children and is likely the result of a hypersensitivity reaction.

bronchiolitis

Inflammation of the bronchioles, most commonly from a viral infection (RSV) that causes inflammatory bronchiolar obstruction. The most common lower respiratory tract infection in the *first 2 years of life*; epidemics occur from November to April. Risk of infection increases with day care attendance, multiple siblings, exposure to smoke, and lack of breastfeeding.

bacterial meningitis

Inflammation of the meninges due to bacterial infection; occurs most often during the *first month of life*, although overall incidence has declined due to Hib vaccine. Risk factors include young age, immunodeficiency, and anatomic defects (basilar skull fracture, shunt).

aseptic meningitis

Inflammation of the meninges with a CSF lymphocytic pleocytosis; usually of viral origin, with normal CSF glucose and protein.

cervicitis

Inflammation of the mucous membranes of the endocervix, most commonly caused by Chlamydia trachomatis or Neisseria gonorrhoeae. Can also be caused by herpes simplex and syphilis. Findings include purulent endocervical discharge, friable edematous erythematous cervix, dysuria, and urinary frequency. Complications include PID, infertility, ectopic pregnancy, Fitz-Hugh-Curtis syndrome (perihepatitis), and neonatal conjunctivitis. Diagnose with culture of the endocervix (gold standard) or use rapid antigen detection or enzyme immunoassay, which are very sensitive but have high false-positive rates.

myocarditis

Inflammation of the myocardium, characterized by cellular infiltrate and myocardial cell death; one of several common causes of sudden death in young athletes. Causes range from viral to bacterial to fungal to autoimmune. Kawasaki disease can cause this sequelae. It often follows a viral or flu-like illness and will present with dyspnea, malaise, and resting tachycardia on physical exam with muffled heart sounds, gallop, hepatomegaly, tachypnea, and pulmonary rales. ESR, CK-MB, CRP will all be elevated in most but not all cases. The causative organism may be found by PCR of endomyocardial biopsy specimens.

Most common: Staph aureus Penetrating injury: coag-negative staph Animal bite: Pasteurella Sickle cell: Salmonella Tb expsoure: Mycobacterium tuberculosis

Name the organisms which can cause osteomyelitis and their associated risk factor

dilated cardiomyopathy hypertrophic cardiomyopathy restrictive cardiomyopathy

Name the three types of cardiomyopathy

*Acute hemorrhagic cystitis,* which may result from bacterial infections, viral infections (eg adenovirus), or chemotherapeutic agents (eg cyclophosphamide).

Large numbers of RBCs in the urine, especially in the presence of dysuria, may indicate what?

metatarsus adductus

Medial curvature of the midfoot; occurs in children under one. Usually caused by intrauterine constraint. Clinical features include a C-shaped foot that can be straightened by gentle manipulation.

corticosteroids

Medication which should be given before or with the first dose of antibiotics in cases of bacterial meningitis, as they have been shown to be effective at reducing the incidence of hearing loss in Hib meningitis. Note that efficacy in other causes of bacterial meningitis has not been demonstrated.

folate deficiency

Megaloblastic anemia, glossitis, pharyngeal ulcers, and impaired immunity are symptoms of what vitamin deficiency?

B12

Megaloblastic anemia, growth failure, paresthesias, sensory defects, developmental regression, weakness, and fatigue are symptoms of what vitamin deficiency?

beta thalassemia

Microcytic anemia caused by deletion of the beta globin gene. Since there are two beta globin genes per cell, a single deletion results in minor anemia and two deletions results in major anemia.

Pierre Robin syndrome

Micrognathia, cleft palate, glossoptosis (downward displacement or retraction of the tongue) and obstruction of the upper airway are found in what syndrome?

basilar artery migraine

Migraine headache associated with vertigo, tinnitus, ataxia, or dysarthria.

West Nile virus

Paralysis and encephalitis is most likely caused by what organism?

ANA = sensitive anti-dsDNA = specific anti-Sm = most specific

Name one antibody makrer that is sensitive for SLE and another two that are specific

Breast hypertrophy Swollen labia Whitish vaginal discharge (leukorrhea) Uterine withdrawal bleeding

Name some *benign* transient findings commonly seen in newborns as responses to transplacental maternal estrogen exposure; work-up is unnecessary with these findings and reassurance should be given.

Urushiol (poison ivy, oak) Nickel (jewelry, buckles) Neomycin (topical antibiotic)

Name some examples of allergic contact dermatitis, a T cell mediated type IV hypersensitivity reaction, which requires prior exposure to the allergen, allowing antigen-specific T cells to create a delayed inflammatory response

Precocious sexual knowledge, preoccupation with masturbation, excessive or aggressive talk about sexuality, and simulating genital-genital, oral-genital, or anal-genital contact (as in on dolls). Anogenital trauma, coercive sex play, or repeated UTIs are also concerning.

Name some examples of behavior in a small child that would be concerning for sexual abuse.

For an adolescent, at presentation, reassurance and education are appropriate. However, reduced sperm counts are possible with this condition; surgery may ultimately be indicated for infertility problems.

Name the correct management of varicocele, a common condition seen after 10 years of age which results from the dilatation of the pampiniform venous plexus (usually on the left side) due to valvular incompetence of the spermatic vein. Typically, this condition is not painful but can become tender with strenuous exercise. Its typical "bag of worms" appearance on palpation makes its diagnosis apparent in most cases.

*Digoxin*, a cardiac glycoside, increases efficiency of myocardial contractions and relieves tachycardia. *Furosemide* and other loop diuretics reduce volume by maximizing sodium loss, which in turn leads to diminished ventricular dilation and improved function. *Dobutamine or dopamine* are inotropic medications given IV to treat severe CHF.

Name the drugs used in medical management of CHF

*Appearance:* blue, bluish, pink *Pulse:* none, <100, >100 *Grimace:* none, w/high stim, w/stim *Activity:* none, flexion, resist extension *Respiration:* none, irregular, strong

Name the five categories scored on a scale from 0 to 2 on the Apgar Score, which is taken at 1 and 5 minutes of life. Recall that scores of 7 and up are generally considered acceptable, and max score is 10.

renal papillary necrosis

Painless hematuria is most likely due to what?

*Meckel's diverticulum:* an outpouching of the bowel in the terminal ileum that occurs in 2% of infants. The diverticulum contains ectopic gastric mucosa that produces acid, causing adjacent intestinal mucosal damage.

Painless, acute rectal bleeding in an otherwise healthy child should always prompt the inclusion of what in your differential?

central precocious puberty

Precocious puberty due to early activation of the hypothalamic-pituitary-gonadal axis, which presents with increased levels of FSH and LH.

peripheral precocious puberty

Precocious puberty independent of the HPGA (i.e. caused by peripheral production of male or female sex steroids, and not FSH or LH mediated). The hallmark is a flat response on GnRH stimulation testing because the HPGA has not been activated. Not ethat in males, there is usually no testicular enlargement, as increase in testicle size is stimulated by FSH.

craniosynostosis

Premature closure of one or more cranial sutures. Etiology often unknown or sporadic. Affects head shape, based on which suture prematurely closes

*HBsAb* (surface antibody)

Presence of this factor is protective against hepatitis B infection and can result from either natural infections or vaccinations

HBcAb (core antibody)

Presence of this factor results from natural infection (not vaccination) of hepatitis B, and persists lifelong; its presence is an indication of past or present hep B infection.

*CGD:* a primary immunodeficiency syndrome caused by a mutation that prevents phagocytic exodative burst (formation of H2O2) and therefore impairs intracellular killing by phagocytes. Staph aureus and Aspergillus are catalase positive organisms which often invade CGD patients.

Recurrent cutaneous abscesses with organism-filled neutrophils (*Staph aureus, Aspergillus*) are consistent with what diagnosis?

trichotillomania

Recurrent hair pulling resulting in hair loss, psychological distress, and functional impairment. There are often attempts to stop pulling hair. Commonly affected sites include scalp, eyebrows, and eyelids.

sequestration crisis

Rapid accumulation of blood in the spleen of sickle cell patients. Occurs in patients under 6 and presents with abdominal distention, abdominal pain, SOB, tachycardia, pallor, fatigue, and shock. Treat with supportive care, transfusion of RBCs, and splenectomy to prevetn recurrence.

Potter syndrome

Severe oligohydramnios can cause this disorder, which results in lung hypoplasia and fetal compression with limb abnormalities and characteristic facial features. Severe oligohydramnios may be a result of a chronic amniotic fluid leak or intrauterine renal failure caused by bilateral renal agenesis, polycystic kidneys, or obstructive uropathy.

pharyngitis

Sore throat and pain on swallowing indicates what likely diagnosis?

intussusception

Telescoping of a proximal portion of intestine into a more distal portion. Male predominance, peak incidence at 5-9 months of age. Most common location is ileocolic. A lead point such as Meckel's, polyps, Peyer's, or lymphoma may be present, but is only identified in 5% of cases and is more common in older children. Causes bowel wall edema and hemorrhage and may lead to bowel ischemia and infarction.

*Primary amenorrhea:* can result from either functional or anatomic problems of the hypothalamus, pituitary, ovaries, uterus, or vagina. The best first step in evaluation is a *pelvic ultrasound* to exclude anatomic abnormalities of the ovaries, uterus, or vagina.

The absence of menarche at age >13 in girls with no secondary sexual characteristics (eg no breast development) or in those age >15 with breast development.

*HHV6:* roseola (exanthema subitum) is a common acute illness of young children (such as the 9-month-old child with fever and transient rash), characterized by several days of high fever followed by a rapid defervescence and the appearance of an evanescent, erythematous, maculopapular rash. Human herpesvirus 6 (HHV-6) has been identified as its main cause.

The appearance of an evanescent, erythematous, maculopapular rash following the rapid defervescence of several days of high fever in a 9month-old boy is likely due to which etiologic agent?

*AML:* diagnosis suggested by these clinical features plus blood smear showing leukemic myeloblasts; confirmed by bone marrow biopsy

The clinical presentation of this childhood cancer is similar to ALL; however, CNS involvement occurs more commonly in this cancer. Symptoms and signs include fever, hepatosplenomegaly, bruising and bleeding, gingival hypertrophy, and bone pain. Lymphadenopathy and testicular involvement are uncommon. Labs may show pancytopenia or leukocytosis, and DIC.

ASD

The congenital heart defect causes a *left to right shunt* (that is, blood flows from the left heart to the right heart, from high resistance to low resistance). This leads to an increase in size of the right atrium and ventricle, and increased pulmonary blood flow. Symptoms are minimal, but physical exam may reveal systolic ejection murmur and a fixed-split second heart sound.

*Sudden infant death syndrome:* this is a diagnosis you make after death. All the emphasis is on SIDS prevention. Place infant on their back, on hard (not fluffy) cribs, don't share a bed, and stop smoking. Don't do prophylactic assessment or interventions (no ECG, apnea monitors, pulse ox, x-rays, etc); just practice good prevention techniques for SIDS

The death of an infant with no reason found on autopsy or during review of the scene

pyridostigmine (cholinesterase inhibitor)

The drug of choice to treat myasthenia gravis

The child in the question appears to have *myotonic muscular dystrophy.* An elevated creatinine kinase (especially in the preclinical phase) often is found, and psychomotor retardation can be the presenting complaint (but may be identified only in retrospect). Ptosis, baldness, hypogonadism, facial immobility with distal muscle wasting (in older children), and neonatal respiratory distress (in the newborn period) are major features of this disorder. Cataracts are commonly seen, presenting either congenitally or at any point during childhood. The prominence of distal muscle weakness in this disease is in contrast to the proximal muscle weakness seen in most other forms of myopathies. The diagnosis is confirmed by a molecular blood test.

The family of a 4-year-old boy has just moved into your area. The child was recently brought to the emergency department (ED) for an evaluation of abdominal pain. Although appendicitis was ruled out in the ED and the child's abdominal pain has resolved, the ED physician requested that the family follow up in your office to evaluate an incidental finding of an elevated creatine kinase. The family notes that he was a late walker (began walking independently at about 18 months of age), that he is more clumsy than their daughter was at the same age (especially when trying to hold onto small objects), and that he seems to be somewhat sluggish when he runs, climbs stairs, rises from the ground after he sits, and rides his tricycle. What is the likely diagnosis?

1) *spit up:* small volume, nonprojectile, formula colored; totally normal 2) *bilious vomiting:* green vomit, indicative of an obsturction distal to the ampulla of Vater 3) *non-bilious vomiting:* not green in color, caused by obstruction proximal to the biliary tree

The first step of evaluating emesis in babies is dividing into what three categories?

*Macrolides:* Azithromycin Clarithromycin

The gold standard for treating pertussis; should be initiated based on clinical suspicion without waiting for confirmatory diagnosis

Reed Sternberg cell

The hallmark histologic feature of Hodgkin's lymphoma found on lymph node biopsy; a large multinucleated cell with abundant cytoplasm

hyposthenuria

The inability of the kidneys to concentrate urine. Often found in patients with sickle cell trait or sickle cell disease. Presents with polyuria. The urine specific gravity will be low but all other labs will be normal. Serum sodium will be normal due to intact ADH.

posterior urethral valves

The most common cause of urinary tract obstruction in newborn boys. Abnormal folds in the distal prostatic urethra obstruct urinary flow, resulting in progressive dilation of the bladder, ureters, and kidneys. Bilateral hydronephrosis is highly suggestive. It leads to oligohydramnios and Potter sequence as a result of poor urine output.

*Maternal withdrawal of estrogen:* the bleeding is self-limited and no treatment is required.

The most common cause of vaginal bleeding and discharge in the neonatal period.

Bacterial vaginosis

The most common cause of vaginitis in adolescents. Presents with gray-white thin vaginal discharge, fishy odor, and little vaginal or vulvar inflammation. Diagnosis can be made by positive "whiff test" or by presence of "clue cells" on wet-mount. Vaginal pH will be > 4.5. Treat with oral metronidazole. Partner does not require treatment.

Staph aureus Strep pyogenes

The most common causes of acute, unilateral lymphadenitis in children

Staph aureus Strep pyogenes

The most common causes of impetigo, which can be treated with topical antibiotics

acute lymphoblastic leukemia

The most common childhood cancer; represents 80-85% of childhood leukemias. Peak incidence at 2-6 years of age. More common in males and caucasians.

Wilms' tumor (nephroblastoma)

The most common childhood renal tumor; often occurs in children young than 5. Associated with genetic findings or syndromes such as Beckwith-Wiedemann and WAGR syndrome.

bacterial penumonia

The most common complication of measles and the most common cause of mortality

Most patients with sickle cell trait lead normal, healthy lives; *painless hematuria* is the most common complication. Sickle cell disease is autosomal recessive, so being a carrier (sickle cell trait) is usually not overtly pathologic.

The most common complication of sickle cell trait

*Transposition of the great vessels:* if suspected, give prostaglandins immediately to keep the ductus arteriosus patent.

The most common congenital cyanotic heart defect in the neonatal period. It generally presents in the first few hours of life with cyanosis and a single loud second heart sound. A narrow mediastinum will be seen on CXR.

metatarsus adductus

The most common congenital foot deformity. Characterized by medial deviation of the forefoot with a normal neutral position of the hindfoot. The deformity is usually bilateral and occurs most frequently in first-born infants, likely due to the crowded positioning in a smaller, primigravid uterus. OVer 90% of cases are characterized by flexible feet that overcorrect both passively and actively into lateral deviation. Treatment is normally not necessary and it corrects spontaneously.

*Complete AV canal septal defect:* failure of the endocardial cushions to merge, resulting in both a VSD and an ASD. Heart failure results. Auscultation will reveal a fixed split S2, a systolic ejection murmur, or a holosystolic murmur, or any combination of the above.

The most common congenital heart defect in patients with Down syndrome

ventricular septal defect

The most common congenital heart disease. There will be a harsh systolic murmur. Some may close spontaneously and do not require intervention; children that have evidence of right-sided hypertrophy, increased right-sided pressures, failure to thrive, or heart failure need immediate repair.

vasooclusive crisis (painful bone crisis)

The most common crisis in sickle cell patients. Due to ischemia/infarction of bone or marrow. Presents with deep, gnawing, or throbbing pain lasting 3-7 days. Sometimes presents with acute dactylitis. Can be confused for esteomyelitis. Manage with fluids, pain control.

Tetralogy of Fallot

The most common cyanotic defect of *children* (TGA is more common but most babies either get fixed or die). Caused by endocardial cushion defect. Defined by overriding aorta, pulmonary stenosis, right ventricle hypertrophy, and ventricular septal defect. If severe, requires immediate intervention. Presents in a toddler with cyanosis relieved by squatting, because squatting causes an increase in SVR, pushing more right ventricular blood to the lungs. Look for a "boot shaped heart" on CXR. Associated with Down and DiGeorge. Surgery is definitive therapy.

transposition of the great arteries

The most common cyanotic defect of the *newborn.* Embryologically, the heart doesn't "twist" correctly and the right ventricle releases blood into the aorta while the left ventricle releases blood into the pulmonary artery (normally the reverse). This means blood pumped to the periphery isn't oxygenated and oxygenated blood is pumped back through the lungs, which is useless. Pregestational diabetes is a risk factor. Without a PDA, this is fatal, so give prostaglandins. Correct with surgery ASAP.

poststreptococcal glomerulonephritis

The most common form of acute glomerulonephritis in school-age children, but rare before age 2. Develops 8-14 days after an infection of the skin or pharynx with a strain of GABHS. Presents with hematuria, proteinuria, and HTN with edema. Low C3 is present but transient, and normalizes within 8-12 weeks, as does renal function.

oral levonorgestrel (Plan B)

The most readily available emergency contraception that prevents pregnancy by delaying ovulation. It should be given as soon as possible as efficacy decreases over time.

congenital rubella syndrome

The most serious complication of rubella. Occurs after primary maternal infection udring the first trimester. Presenting clinical features include thrombocytopenia, hepatosplenomegaly, jaundice, and purpura ("blueberry muffin baby"). Structural abnormalities include congenital cataracts and patent ductus arteriosus. Other findings include sensorineural hearing loss and meningoencephalitis. LAte complications include mental retardation, HTN, DM Type I, and autoimmune thyroid disease.

Hurler syndrome

The most severe mucopolysaccharidosis; autosomal recessive. Features begin after 1 year of age with kyphosis, hepatosplenomegaly, and developmental delay. There are coarsened facial features, frontal bossing, hydrocephalus, corneal clouding, and stiff contracted joints. Caused by alpha-L-iduronidase deficiency and diagnosed by finding dermatan and heparan sulfates in the urine. Manage with early bone marrow transplant.

The symptoms listed are those of *vulvovaginitis*, with nonspecific (or chemical) vulvovaginitis accounting for 70% of all pediatric vulvovaginitis cases. The discharge in nonspecific vulvovaginitis is usually brown or green and with a fetid odor. The burning with urination occurs because of contact between raw skin and urine. Further history in this case might reveal use of tight-fitting clothing (including swimsuits or dance outfits), nylon undergarments, prolonged bubble baths with contamination of the vagina with soap products, use of perfumed lotions in the vaginal area, or improper toilet habits (wiping of fecal material toward rather than away from vagina). Attention to these causative conditions usually results in resolution of the symptoms.

The mother of a 2-year-old girl reports that her daughter complains of burning when she urinates and that she has foul-smelling discharge from her vagina. She has some slight staining on the front of her underwear, but denies fever, nausea, vomiting, or other constitutional signs. The child does not attend day care, and she has demonstrated no change in behavior. The physical examination is normal with an intact hymen, but the child's vulva is reddened and with a malodorous scent noted. Her urinalysis and culture are normal. What is the likely diagnosis?

*Screen the parents for total cholesterol.* Identification of those with a genetic predisposition to hypercholesterolemia and of the factors that increase the risk of the condition is recommended so that dietary and other measures to reduce serum lipids can be introduced if indicated. Children with a first- or second-degree relative with early onset of coronary heart disease should be evaluated early in life but after 2 years of age. Other known risk factors include obesity, diabetes, hypertension, and smoking. No change in current infant dietary practice is recommended for children under 2 years of age.

The parents of a 2-month-old baby boy are concerned about his risk of coronary artery disease because of the recent death of his 40-yearold maternal uncle from a myocardial infarction. What should you do?

The patient has classic findings of *galactosemia.* Galactose is a component of lactose, found in breast milk and most infant formulas. Symptoms of galactosemia occur in the first weeks of life. While screening for classic galactosemia typically is part of the newborn metabolic panel, patients fitting the clinical presentation as outlined in the question must be evaluated promptly. Signs and symptoms in addition to those presented in the vignette include cataracts and ascites. While three different errors in galactose metabolism are known, most cases result from the deficiency in galactose-1-phosphate uridyl transferase. Urine-reducing substances can be positive, but a routine urinalysis will be negative, as the urine strips do not react with galactose. Patients are at increased risk for E coli sepsis, and this infection may precede the diagnosis of galactosemia. Prompt removal of galactose from the diet usually reverses the symptoms, including cataracts.

The parents of a 2-week-old infant bring him to the emergency center in your local hospital for emesis and listlessness. Both of his parents wanted a natural birth, so he was born at home and has not yet been to see a physician. On examination, you find a dehydrated, listless, and irritable infant. Although you do not have a birth weight, the parents do not feel that he has gained much weight. He has significant jaundice. His abdominal examination is significant for both hepatomegaly and splenomegaly. Laboratory values include a total bilirubin of 15.8 mg/dL and a direct bilirubin of 5.5 mg/dL. His liver function tests are elevated and his serum glucose is 38 mg/dL. His admit urinalysis is negative for glucose but positive for Gram-negative rods; his urine and his blood ultimately grow E coli. What is the likely diagnosis?

tension headaches

These headaches are bifrontal or diffuse, dull, and aching, often associated with muscle contraction. They are unusual in children and thus other diagnoses should be preferentially considered. There is no vomiting, visual changes, or paresthesias.

cluster headaches

These headaches, rare during childhood, are characterized by unilateral frontal or facial pain, accompanied by conjunctival erythema, lacrimation, and nasal ocngestion.

*Dacryocystitis* is an infection of the nasolacrimal sac. In newborns it is associated with congenital nasolacrimal duct obstruction, which is seen in about 6% of normal infants. Nasolacrimal duct obstruction is thought to be caused by the failure of epithelial cells forming the duct to canalize. Treatment of this benign, usually self-limited condition involves nasolacrimal massage and cleaning the area with warm washcloths; failure to open the duct by 6 months usually results in a referral to ophthalmology for surgical opening. When the lacrimal sac is infected, as in dacryocystitis, the patient requires a course of antibiotics to clear the infection.

The parents of a 7-day-old infant bring her to your office for a swollen eye. Her temperature has been normal, but for the last 2 days she has had progressive erythema and swelling over the medial aspect of the right lower lid near the punctum. Her sclera and conjunctiva are clear. Gentle pressure extrudes a whitish material from the punctum. What is the diagnosis?

The patient described has *Wiskott-Aldrich syndrome*, an X-linked recessive combined immunodeficiency characterized by thrombocytopenia, eczema, and increased susceptibility to infection. Problems occur early, and prolonged bleeding from the circumcision site may be the first clue. The thrombocytopenia also manifests as bloody diarrhea and easy bruising. Patients have impaired humoral immunity with a low serum IgM and a normal or slightly low IgG; they also have cellular immunity problems, with decreased T cells and depressed lymphocyte response. Few live past their teens, frequently succumbing to malignancy caused by EBV infection.

The parents of a 7-month-old boy arrive in your office with the child and a stack of medical records for a second opinion. The boy first started having problems after his circumcision in the nursery when he had prolonged bleeding. Studies were sent at the time for hemophilia, but factor VIII and IX activity were normal. At 2 months, he developed bloody diarrhea, which his doctor assumed was a milk protein allergy and changed him to soy; his parents note he still has occasional bloody diarrhea. He has seen a dermatologist several times for eczema, and he has been admitted to the hospital twice for pneumococcal bacteremia. During both admissions, the parents were told that the infant's platelet count was low, but they have yet to attend the hematology appointment arranged for them. The child's WBC count and differential were normal. What is the diagnosis?

*HBsAg* (surface antigen)

The presence of this is pathopnomonic for active hepatitis B infection. For this reason, it is used in the hepatitis B vaccine

medulloblastoma

The second most common posterior fossa tumor in children, after cerebellar astrocytoma. They usually occur in the cerebllar vermis, which is important for balance and gait coordination. Symptoms include truncal or gait instability. Less commonly, they occur in the lateral cerebellar hemispheres and cause intention tremor. They can also cause obstructive hydrocephalus, due to the proximity to the fourth ventricle.

Trisomy 18 (Edwards Syndrome)

The second most common trisomy; most common in females. Features mental retardation, hypertonia with scissoring of the legs, delicate small facial features, clenched hands with overlapping digits, and rocker bottom feet.

*Urticaria:* to treat, use second gen H1 antihistamines (*cetirizine, loratidine, fexofenadine*) and remove/avoid the offending agent if possible. First gen antihistamines can be used but have a sedating side effect.

The skin manifestation of allergic reactions; usually IgE-mediated, but can also come from non-immunologic mast cell degranulation (contrast, opiates, Red man syndrome from vancomycin). The skin will have erythema and wheals which are often pruritic and limited to the superficial layers of the dermis. Always check for symptoms of anaphylaxis (dyspnea, wheezing, diarrhea).

aplastic crisis

This phenomenon occurs in sickle cell patients. It is characterized by temporary cessation of RBC production, often caused by parvovirus B19 or other infectious agents. Presents with pallor, fatigue, and tachycardia. Treat supportively with RBC transfusion.

pericardiocentesis

This procedure is both diagnostic of and therapeutic for pericarditis.

reactive attachment disorder

This psychiatric disorder may develop in young children when abuse, neglect, or inconsistent care (moving between foster homes) disrupts development of healthy, secure attachment to caregivers. These children seldom seek comfort and do not respond to attempts to comfort them. Other symptoms include lack of social responsiveness, lack of positive emotions, and episodes of unexpected irritability or sadness in response to nonthreatening encounters.

Streptococcal perianal dermatitis

This rash often presents in young children as an erythematous, sharply demarcated perianal rash associated with pruritus and pain. There are often close contacts with cellulitis or pharyngitis. Diagnosis can be confirmed with a perianal bacterial culture. Treat with beta lactams (penicillin, amoxicillin).

epiglottitis

This respiratory disease presents with abrupt onset or rapidly progressive upper airway obstruction without prodrome. High fever, toxic appearance, muffled speech, dysphagia, drooling, and quiet stridor may occur. Patients often sit forward in *tripod position" with neck hyperextension. On a lateral radiograph of the neck, a "thumbprint" sign is visible. The epiglottis will be cherry red and swollen.

ASO titers

This serologic marker is abnormally elevated in most patients with rheumatic fever; it is evidence of a recent GABHS infection.

AFP

This serum protein is elevated in the amniotic fluid and maternal serum in open neural tube defects; it it used to detect spinal defects.

EBV pharyngitis

This specific pharyngitis may present with tonsillar exudates, enlarged posterior cervical lymph nodes, malaise, and hepatosplenomegaly

thymus

This structure in the thorax is normally visible on CXRs in children *under 3 years.* On frontal views, it is commonly seen as the *"sail sign"* due to its triangular shape, scalloped border, and uniform density.

In children, *ITP* is the most common form of thrombocytopenic purpura. It is a diagnosis of exclusion. In most cases, a preceding viral infection can be noted. In this disease, the platelet count is frequently less than 20,000/μL, but other laboratory tests yield essentially normal results, including the bone marrow aspiration (if done). Complications are uncommon; significant bleeding occurs in only 5% of cases and intracranial hemorrhage is even rarer. The treatment of childhood ITP is controversial. Patients with mild symptoms such as bruising and self-limited epistaxis may be observed, while patients with significant bleeding should be treated.

Two weeks after a viral syndrome, a 2-year-old child develops bruising and generalized petechiae, more prominent over the legs. No hepatosplenomegaly or lymph node enlargement is noted. The examination is otherwise unremarkable. Laboratory testing shows the patient to have a normal hemoglobin, hematocrit, and white blood cell (WBC) count and differential. The platelet count is 15,000/μL. What is the likely diagnosis?

Cryptosporidium has become an important cause of diarrhea in immunocompromised patients, particularly those with AIDS who are not on the highly active antiretroviral therapy (HAART). It can also affect patients who are immunocompetent, and has been recognized as an agent responsible for epidemics of diarrhea in day-care centers. Although the disease is selflimited in immunocompetent individuals, treatment with the antiparasitic nitazoxanide can improve symptoms quickly. Persistent, nonsuppurative diarrhea as described in the question can be caused by such organisms as *amebas, whipworms (trichuriasis), Cryptosporidium, or Giardia lamblia.*

Two weeks ago, a 5-year-old boy developed diarrhea, which has persisted to the present time despite dietary management. His stools have been watery, pale, and frothy. He has been afebrile. What is the likely causative parasite?

DR3 and DR4

Type 1 DM is associated with which HLA haplotypes?

vesicoureteral reflux

Urine flowing backwards from the bladder into the ureters and the renal collecting system. Caused by abnormalities of the ureterovesical junction, most commonly a short submucosal tunnel in which the ureter inserts through the bladder wall. Autosomal dominant. Most children have spontaneous resolution. It predisposes to pyelonephritis which leads to renal scarring. Diagnosed via voiding cystourethrogram. Manage with low-dose prophylactic antibiotics.

pallor pulselessness paralysis pain paresthesias

What "Five Ps" are late signs of compartment syndrome?

low MCV and MCHC elevated RDW

What CBC values are associated with iron deficiency anemia?

1) pleocytosis with neutrophils predominating 2) low CSF glucose 3) ratio of CSF to serum glucose <0.40 4) increased CSF protein 4) positive Gram stain and culture

What CSF analysis findings from an LP would indicate likely bacterial meningitis?

Meconium ileus Distal intestinal obstruction syndrome Rectal prolapse Pancreatic insufficiency Recurrent pancreatitis Chronic hepatic disease

What GI abnormalities are seen in cystic fibrosis?

A course of zidovudine for the infant

What action can decrease transmission of HIV from mother to infant?

As a physician, you are obligated to report abuse to CPS. Separate the abuser from the child. Hospitalize the patient if no safe alternative exists.

What actions should you take if you suspect child abuse or neglect?

Treat the patient with *isoniazid, rifampin, and pyrazinamide.* Immediate family and close contacts should be tested with tuberculin skin tests and chest radiographs and treated appropriately when indicated. Three to eight weeks is required after exposure before hypersensitivity to tuberculin develops. This means that the tuberculin test must be repeated in exposed persons if there is a negative reaction at the time that contact with the source of infection is broken.

What actions should you take once you have diagnosed a pediatric patient with tuberculosis?

Antidopaminergic agents are the most effective medications; these include dopamine-depleting tetrabenazine as well as antisychotics (risperidone, haloperidol). Alpha-2 adrenergic agonists (clonidine, guanfacine) are also used, but the evidence for them is not as robust

What agents are effective for treating Tourette disorder? Recall that the most effective nonpharmacologic method is habit reversal training.

*azithromycin/erythromycin* Antibiotics do not alter the patient's clinical course unless they are administered during the catarrhal phase or very early in the paroxysmal phase. Make sure whooping cough patients are on respiratory isolation until antibiotics have been given for at least 5 days.

What antibiotics are used to treat whooping cough?

amp plus aminoglycoside or third-gen ceph; IV acyclovir for possible HSV infection should also be considered for ill neonates, especially those presenting with apnea, seizures, or cutaneous vesicles.

What antibiotics should be given for *newborns* (0-28 days) with suspected bacterial meningitis?

third-gen cephalosporin add vanc if index of suspicion high

What antibiotics should be given for *older children* (over 3 months) with suspected bacterial meningitis?

ampicillin plus third-gen ceph; add vanc if index of suspicion is high

What antibiotics should be given for *young infants* (1-3 months) with suspected bacterial meningitis?

*Cefepime* covers MSSA and Pseudomonas; *IV vanc* covers MRSA.

What antibiotics should you use to treat bacterial lower respiratory infections in CF patients?

The child in the question likely has *PKU;* the clinical picture described was entirely preventable with universal newborn metabolic screening and presymptomatic dietary intervention. PKU is a disease that affects about 1 in 15,000 people in the United States, being more common in Whites and Native Americans than in Blacks, Hispanics, and Asians. It is inherited as an autosomal recessive manner. In addition to the features in the question, children born to women with PKU and who do not follow a low-PKU prenatal diet are at risk for a syndrome similar to fetal-alcohol exposure including microcephaly, mental retardation, and malformations of the heart and great vessels. Infants that are not diagnosed as part of the newborn screening program or for whom post-natal dietary manipulation is not achieved have additional signs and symptoms to those in the question that might include seizures.

You see in consultation a 2-year-old child from a local institution for chronically ill children. He was born normal but had significant developmental delay noted in the first months of life. Early in his life he had significant vomiting and was once evaluated for pyloric stenosis although the testing for this was negative. He is now hyperactive, and has much purposeless movements with rhythmic rocking and athetosis noted. The family notes that his only other illnesses have been mild seborrhea and eczema, which have got better over time. As you compare him to his 2 older siblings in the room, you note that his skin is of lighter color. You note a distinctive, unpleasant "mousy" smell in the room. What is the most likely diagnosis?

iron deficiency anemia

Young children with this condition are often pale and may be underweight. Older children with this condition may have spoon-shaped nails and diminished attention and ability to learn.

pustular melanosis

a benign transient rash characterized by small, dry, superficial vesicles over a dark macular base. This rash is more frequently seen in African American infants. This must be differentiated from viral infections, such as herpes simples, and from bacterial infections, such as impetigo.

vernix caseosa

a thick, white, creamy material found in term infants; it covers large areas of the skin in preterm infants but is usually absent in postterm infants.


Conjuntos de estudio relacionados

Legal Concepts of Life Insurance

View Set

Chapter 1 Problem Set, Ten Principles of Economics Homework

View Set

Combo with "APUSH Chapters 1-35 Review" and 10 others

View Set